X



トップページ数学
1002コメント461KB
分からない問題はここに書いてね462
レス数が1000を超えています。これ以上書き込みはできません。
0272132人目の素数さん
垢版 |
2020/08/11(火) 15:47:14.80ID:cs2e13nz
87 71
85 70 55
83 68 54 41
81 66 53 40 29
79 64 51 39 28 19
77 62 49 38 27 18 11
75 60 47 36 26 17 10 5
73 58 45 34 25 16 9 4 1


規則性を見つけてくれ〜(^_^)ノ
0274132人目の素数さん
垢版 |
2020/08/11(火) 17:48:36.34ID:cs2e13nz
>>272

nを1〜44まで変化させた2n−1の出力に
4を頂点としてその周りを1小さな数で
取り囲んでいったものをプラスしたもの

0 0
0 1 0
0 1 1 0
0 1 2 1 0
0 1 2 2 1 0
0 1 2 3 2 1 0
0 1 2 3 3 2 1 0
0 1 2 3 4 3 2 1 0

このような数列を表す数式を
知っている人はいますか?
0275132人目の素数さん
垢版 |
2020/08/11(火) 18:12:30.51ID:RcW8WFhM
2. 事象 A と事象 Bが起こる確率はP(A) = 0.6, P(B) = 0.7 である。条件
付確率 Pa(B) = 0.8であるという。Pb(A) を求めよ。
事象 A と事象 Bが起こる確率は P(A) = 0.8, P(B) = 0.7 である。P(A∪B) = 0.94がわかっている。このとき事象 A と事象Bが独立であるか
否かを説明しなさい。
答を教えて頂けたら幸いです
0279132人目の素数さん
垢版 |
2020/08/11(火) 18:52:11.01ID:RoFM6jYQ
ある鉱石に含まれる鉄分含有率 (%) を調べたところ,次のデータを得た
7.5, 5.5, 4, 7, 8.5, 5, 9, 9.5
鉄分含有率は正規分布に従うとする.そのとき,以下の問いに答えよ
1. 標本平均 x の値を以下の中から選択せよ
(1) 1.870829, (2) 2, (3) 3.5, (4) 4, (5) 7
2. (不偏)標本分散 s2 の値を以下の中から選択せよ
(1) 1.870829, (2) 2, (3) 3.5, (4) 4, (5) 7
3. (不偏)標本標準偏差 s の値を以下の中から選択せよ
(1) 1.870829, (2) 2, (3) 3.5, (4) 4, (5) 7
4. 自由度 7 のティー分布の両側 0.1 点 t7 (0.1) の値を以下の中から選択せよ
(1) 1.8595, (2) 1.8946, (3) 2.3060, (4) 2.3646, (5) 3.4995
5. 母平均 µ の 90% 信頼区間をを以下の中から選択せよ
(1) [5.685135,8.314865], (2) [5.660315,8.339685], (3) [5.369412,8.630588],
(4) [5.327975,8.672025], (5) [4.52548,9.47452]


1番は3.5
2番は4
3番は2になりました
4番以降がわからないです
0280132人目の素数さん
垢版 |
2020/08/11(火) 19:30:15.93ID:M3m7YlSp
出来そうでできません。

n番目の素数は2^nより小さいことを証明せよ。
0281132人目の素数さん
垢版 |
2020/08/11(火) 19:37:23.98ID:FoWZrPf+
>>280
ベルトラン=チェビシェフの定理を使えば
p_n≦2^nならば
2^n<p≦2^(n+1)なる素数pが1つは存在するので
p_(n+1)≦2^(n+1)もわかり
帰納法で示せたことになる
0282132人目の素数さん
垢版 |
2020/08/11(火) 20:25:09.38ID:7zYwSct8
>>274
数列:
a(n) = {
i = ceil( (-3 + sqrt(8*n+9))/2 ) ; \\ row
j = n - (i-1)*(i+2)/2; \\ column
return( -1 + min( i- (2<j)*(j-2) , j ) )
}

a(1)〜a(65)
0, 0,
0, 1, 0,
0, 1, 1, 0,
0, 1, 2, 1, 0,
0, 1, 2, 2, 1, 0,
0, 1, 2, 3, 2, 1, 0,
0, 1, 2, 3, 3, 2, 1, 0,
0, 1, 2, 3, 4, 3, 2, 1, 0,
0, 1, 2, 3, 4, 4, 3, 2, 1, 0,
0, 1, 2, 3, 4, 5, 4, 3, 2, 1, 0,
(i<j の時に改行)
0283132人目の素数さん
垢版 |
2020/08/11(火) 21:13:15.60ID:cs2e13nz
プログラムコードじゃなくて
wolfram 入力可能な数式化したい
0284132人目の素数さん
垢版 |
2020/08/11(火) 21:47:48.94ID:3FSqzbPO
>>281
証明を与えていただきありがとうございます。
証明する事柄が自明にしか見えないので、高校数学程度の知識で示せると直感的に考えたのですが、そうでもないのでしょうか。
0285132人目の素数さん
垢版 |
2020/08/11(火) 21:52:05.06ID:cs2e13nz
17 11
15 10 5
13 8 4 1

Table[2n-1+C(0,(21mod n)-1),{n,1,9}]

nが小さい時の式はつくれる
nが大きくなると破綻する
0286132人目の素数さん
垢版 |
2020/08/11(火) 22:06:22.77ID:dwVOjOlW
>>279
R使って
x=c(7.5, 5.5, 4, 7, 8.5, 5, 9, 9.5)
mean(x)
var(x)
sd(x)
qt(1-0.1/2,df=7)
t.test(x,conf.level = 0.9)
で終了。
0287132人目の素数さん
垢版 |
2020/08/11(火) 22:34:35.71ID:dwVOjOlW
>>282
慣れた言語に移植して続きを出力してみました。

0 0
0 1 0
0 1 1 0
0 1 2 1 0
0 1 2 2 1 0
0 1 2 3 2 1 0
0 1 2 3 3 2 1 0
0 1 2 3 4 3 2 1 0
0 1 2 3 4 4 3 2 1 0
0 1 2 3 4 5 4 3 2 1 0
0 1 2 3 4 5 5 4 3 2 1 0
0 1 2 3 4 5 6 5 4 3 2 1 0
0 1 2 3 4 5 6 6 5 4 3 2 1 0
0 1 2 3 4 5 6 7 6 5 4 3 2 1 0
0 1 2 3 4 5 6 7 7 6 5 4 3 2 1 0
0 1 2 3 4 5 6 7 8 7 6 5 4 3 2 1 0
0 1 2 3 4 5 6 7 8 8 7 6 5 4 3 2 1 0
0 1 2 3 4 5 6 7 8 9 8 7 6 5 4 3 2 1 0
0 1 2 3 4 5 6 7 8 9 9 8 7 6 5 4 3 2 1 0
0 1 2 3 4 5 6 7 8 9 10 9 8 7 6 5 4 3 2 1 0

あっているかな?
0288132人目の素数さん
垢版 |
2020/08/11(火) 22:36:23.41ID:FoWZrPf+
>>284
素数に関することは自明そうでも難しいことが多いと思う
これに関しては何か上手い別の方法で示せるかもしれないけどね
0289132人目の素数さん
垢版 |
2020/08/11(火) 23:32:05.94ID:cs2e13nz
>>282 は総和が95

>>287 は総和が715 ならあっている
0290132人目の素数さん
垢版 |
2020/08/11(火) 23:33:42.39ID:cs2e13nz
総和を出力する関数

(4n^3-6n^2-4n-3(-1)^n+3)/48 
0292132人目の素数さん
垢版 |
2020/08/12(水) 06:15:40.39ID:hrLsE2UP
f(x) = 1+a*sin(x)+b*cos(x)に対し、
g(x) = ∫[0,x] (x-t)f(t) dt
とおく。
このとき任意の実数x,yについて
g(x+y)+g(x-y) ≧ 2g(x)
が成り立つような実数a,bが満たす条件を求めよ。
0293132人目の素数さん
垢版 |
2020/08/12(水) 07:13:59.33ID:VuTZnt5m
>>292
要は常に下に凸ってことだから恒等的に g''(x)≧0 となるようにすればいいんじゃね?
0296132人目の素数さん
垢版 |
2020/08/12(水) 10:18:33.70ID:mYtbyTE6
体積問題は単純な計算問題にしかならないんで
丁寧に式変形しても 外野から「Wolfram先生に頼ったんだろw」とケチを付けられて終わる。
体積半分条件も大抵は数値計算に帰着するしかないんで数学的な面白みは薄い。
0297132人目の素数さん
垢版 |
2020/08/12(水) 11:04:37.57ID:StcyJuzq
積分問題は積分可能性がかなりアルゴリズム化されてるから、ほとんどのケースで手計算でやる意味はあんまりないと言えばないからな
積分がexplicitにできるかどうか不明であるケースはかなり少なくなってきてる
まぁ素人が適当に作った問題なんか高校生でもできるか、explicitには計算不能のどっちかにしかならない
0298イナ ◆/7jUdUKiSM
垢版 |
2020/08/12(水) 12:13:37.10ID:VaAaef6o
>>277
>>294左←審判が白の選手を殴って制したのはわかる。
青の選手を蹴ったのは青の選手がなにか暴言を吐いたんだろうか?
0299132人目の素数さん
垢版 |
2020/08/12(水) 13:30:09.82ID:erjZX/DD
>>286
Rって何ですか?
0302132人目の素数さん
垢版 |
2020/08/12(水) 14:25:59.44ID:v8BOhpZv
nを自然数の定数とする。
xy平面において、極方程式r=1+(2^n)*cosθで表される曲線をCとする。
C上の格子点の個数をnで表せ。
0304132人目の素数さん
垢版 |
2020/08/12(水) 16:52:51.95ID:erjZX/DD
>>301
Rの使い方がわかりません
0305132人目の素数さん
垢版 |
2020/08/12(水) 19:47:46.27ID:UO/+XUZI
あの普通の中学生が質問しても大丈夫ですか?
0306132人目の素数さん
垢版 |
2020/08/12(水) 19:55:00.44ID:UO/+XUZI
3axy-2axをカッコで括るとき
ax(3y-2)とa(3y-2)xはどっちが正しいのですか?

あとカッコの外に出した文字はカッコの前後どちらに付けるのか決まりがあるんでしょうか?
0311132人目の素数さん
垢版 |
2020/08/12(水) 21:59:52.63ID:so8oBh6O
>>282 の演算子を取り除いて
数式に変換してくれ〜(^_^)ノ
0312132人目の素数さん
垢版 |
2020/08/12(水) 23:02:10.56ID:mYtbyTE6
>>311
これくらい推測できるだろ...と思ったが一応書いとく。
sqrt(x) = √x
ceil(x) = ⌈ x ⌉  ( ceiling function, 天井関数 )
min(x, y) = if (x ≦ y) then x else y
(x<y) = if (x < y) then 1 else 0
0313132人目の素数さん
垢版 |
2020/08/12(水) 23:06:26.50ID:mYtbyTE6
最後のは ブール値である False, True と 0, 1 を区別しない言語に特有の記法
そうでない言語も多い。
0314132人目の素数さん
垢版 |
2020/08/12(水) 23:40:33.06ID:fvT6HFIC
幅10cmの正五角形の中心点から頂点までの長さは何cm何mmですか?
0317132人目の素数さん
垢版 |
2020/08/13(木) 00:10:25.20ID:J8kLqyHu
a n i = div (n-(abs $ n-2*i)) 2

main = do
mapM_ print $ take 10 $ [[a n i|i<-[0..n]] | n<-[0..]]

[0]
[0,0]
[0,1,0]
[0,1,1,0]
[0,1,2,1,0]
[0,1,2,2,1,0]
[0,1,2,3,2,1,0]
[0,1,2,3,3,2,1,0]
[0,1,2,3,4,3,2,1,0]
[0,1,2,3,4,4,3,2,1,0]
0318132人目の素数さん
垢版 |
2020/08/13(木) 00:11:22.16ID:sbSWJEQc
>>312
wolfram で出力可能な数式化という意味
0322132人目の素数さん
垢版 |
2020/08/13(木) 00:17:37.58ID:gpqh/Hd2
>>316
正五角形だった。

複素平面に作図して計測
p=ngon(5)
o=mean(p[1:5]);pt(o)
seg(o,p[1]);seg(p[2],p[4])
10*abs(o-p[1])/abs(p[2]-p[4])
[1] 5.257311

約5cm26mm
0323132人目の素数さん
垢版 |
2020/08/13(木) 00:29:57.96ID:sbSWJEQc
sqrtとceilingはwolframで出力できる
columnは無理
0324132人目の素数さん
垢版 |
2020/08/13(木) 00:31:12.37ID:gpqh/Hd2
>>322
半径1の円に内接する5角形の対角線の長さは2sin(2π/5)
対角線の長さが10なら、半径は
> 10 / ( 2*sin(2*pi/5))
[1] 5.257311
0326132人目の素数さん
垢版 |
2020/08/13(木) 01:22:21.90ID:LK1yH0ga
ある三角形の内心I、傍心I‘として
線分II’の中点が線分II‘と三角形の外接円との交点になるのは何故ですか?
0327132人目の素数さん
垢版 |
2020/08/13(木) 02:24:57.15ID:sbSWJEQc
>>312
◆sqrtとceilはwolfram形式にできた

Table[n-(ceil((-3+sqrt(8*n+9))/2)-1)*(ceil((-3+sqrt(8*n+9))/2)+2)/2,{n,1,40}]


式の判定部分をwolfram入力形式に変形してくれ〜(^_^)ノ

return(-1+min(i-(2<j)*(j-2),j))
0330132人目の素数さん
垢版 |
2020/08/13(木) 05:14:24.98ID:GNxQ5zOj
>>326
内心 傍心 外接円 でgoogle先生にお願いしたら一番上に知恵袋の解答が出てきた。
0331132人目の素数さん
垢版 |
2020/08/13(木) 10:36:05.71ID:I56KrEx3
(x-y)e^(-x^2-y^2)の極値って求められますか?
∂xf=∂yf=0の(x,y)がうまくいかなかったのですが
0332132人目の素数さん
垢版 |
2020/08/13(木) 11:14:17.06ID:LP9xEpjl
積分するのが難しい関数はあるのに微分するのが難しい関数がないのはなぜ?
0335132人目の素数さん
垢版 |
2020/08/13(木) 12:39:48.06ID:GNxQ5zOj
>>9
素直に読んだら後攻必勝ではないから、(1)が証明できるためには相当ひねくれた無茶苦茶なルール解釈が必要となる。

例えば
>まずS君がサイコロを振り、駒Xを動かす(先攻)。
>続けてT君がサイコロを振り、駒Yを動かす(後攻)。
の部分について、サイコロを“何回”振ると書いていないので3の倍数が出てしまったら連続で何回でも振りなおすと解釈すると
先攻は絶対に勝利しない、後攻が勝つことはある、永久に勝利者が現れないこともあるのでこの状態を後攻必勝と表現できなくもない。

この解釈だと(2)はp=(1/2)^nとなるのでn=1のときの|p-(1/2)|=0が最小である。
0336女子中学生
垢版 |
2020/08/13(木) 13:45:04.75ID:c43mBXX1
>>309
ありがとうございます!
夏休み明けにテストがあったんで不安でしたが
やれる気がして来ました!
0337132人目の素数さん
垢版 |
2020/08/13(木) 13:55:48.17ID:LK1yH0ga
>>330
自分の検索能力カスすぎて草
0338イナ ◆/7jUdUKiSM
垢版 |
2020/08/13(木) 14:17:47.38ID:bwxcosiq
>>277
>>314
求める長さをxcmとし、
正五角形の一辺の長さをycmとおくと、
対角線が10cmで、ほかの対角線によって10:y:10に分割され、この3つのパーツ10+y+10のうち10+yが一辺の長さと等しい。
∵一つの対角線と一つの辺が平行だから。
10(10+y)/(10+y+10)=y
100+10y=20y+y^2
y^2+10y-100=0
y=5√5-5
以下再検討。
y^2-5^2={x-√(x^2-5^2)}^2
125-50√5+25-25=x^2-2x√(x^2-25)+x^2-25
125-50√5=2x^2-2x√(x^2-25)-25
x^2-x√(x^2-25)-75+25√2=0
0339132人目の素数さん
垢版 |
2020/08/13(木) 15:02:01.85ID:OcaeKpVf
tを実数とし、f(t)=sin(t)+cos(t)+√2とする。

(1)f(t)≧0 を示せ。

(2)xy平面において、極方程式f(r)=sinθcosθにより定まる曲線をCとする。C上を点A(a,b)が動くとき、g(a)=(a+1)(b+1)を最大にするAの位置を求めよ。

(3)g(a)を最大にするaをpとおくとき、定積分∫[0,p] e^(-x^2) dxは有理数か。
0340132人目の素数さん
垢版 |
2020/08/13(木) 15:15:38.10ID:Uuox2URt
Table[Floor[Abs[n-(Floor[(Floor[Sqrt[2n+1]]+1)^2/2]-1/2-Floor[(Floor[Sqrt[2n+1]]+1)/2])]],{n,1,100}]

{0, 0, 0, 1, 0, 0, 1, 1, 0, 0, 1, 2, 1, 0, 0, 1, 2, 2, 1, 0, 0, 1, 2, 3, 2,
1, 0, 0, 1, 2, 3, 3, 2, 1, 0, 0, 1, 2, 3, 4, 3, 2, 1, 0, 0, 1, 2, 3, 4, 4,
3, 2, 1, 0, 0, 1, 2, 3, 4, 5, 4, 3, 2, 1, 0, 0, 1, 2, 3, 4, 5, 5, 4, 3, 2,
1, 0, 0, 1, 2, 3, 4, 5, 6, 5, 4, 3, 2, 1, 0, 0, 1, 2, 3, 4, 5, 6, 6, 5, 4}
0343イナ ◆/7jUdUKiSM
垢版 |
2020/08/13(木) 15:51:46.74ID:bwxcosiq
>>338
>>314
求める長さをxcmとし、
正五角形の一辺の長さをycmとおくと、
対角線が10cmで、ほかの対角線によって10:y:10に分割され、この3つのパーツ10+y+10のうち10+yが一辺の長さと等しい。
∵一つの対角線と一つの辺が平行だから。
10(10+y)/(10+y+10)=y
100+10y=20y+y^2
y^2+10y-100=0
y=5√5-5
次に正五角形の中心と頂点を斜辺xとする直角三角形においてピタゴラスの定理より、
(5√5-5)^2/4=x^2-(10-x)^2
150-50√5=4(20x-100)
75-25√5=40x-200
40x=275-25√5
8x=55-5√5
x=(55-5√5)/8
=5.47745751406……
∴約5cm5mm
0344132人目の素数さん
垢版 |
2020/08/13(木) 15:53:01.50ID:EuD8qnzl
>>541
x=0のときy=0 or z=0
(x,y)=(0,0)なら(z,w)=(±1,0)
xyz≠0のときw≠0
x,y,z,wのうち負であるものは偶数個であり
(x,y,z,w), (-x,-y,-z,-w),
(-x,-y,z,w), (-x,y,-z,w), (-x,y,z,-w),
(x,y,-z,-w), (x,-y,z,-w), (x,-y,-z,w),
のいずれかは全て正
よってx,y,z,w>0の場合を考えれば良い
xy/z=zx/y=yz/xよりx^2=y^2=z^2
∴ x=y=z=2w=1/√3
以下ry
0346132人目の素数さん
垢版 |
2020/08/13(木) 16:29:36.62ID:sbSWJEQc
>>340
数えはじめの修正が必要

87 71
85 70 55
83 68 54 41
81 66 53 40 29
79 64 51 39 28 19
77 62 49 38 27 18 11
75 60 47 36 26 17 10 5
73 58 45 34 25 16 9 4 1


惜しい(^_^)ノ
0347132人目の素数さん
垢版 |
2020/08/13(木) 16:54:10.76ID:Vk3erFID
xy平面上に、一辺の長さが2√3の△ABCと動点Pがある。
tをt>2√3の実数とする。

(1)動点PがPB+PC=tを満たしながら平面上を動く。Pの描く軌跡と辺ABが交点を持つときの、tの取りうる値の上限を求めよ。

以下、tは(1)の上限を超えないとする。

(2)(1)において、Pの描く軌跡と辺ABの交点をTとする。BTをtで表せ。

(3)(1)において、Pの描く軌跡上でAから最も近い点をSとする。ASをtで表せ。

(4)点Pは以下の条件を満たす。
・PA+PB+PC=r, r>0
・Pの描く軌跡は△ABCの内部にある
rの取りうる値の範囲を求めよ。
0348132人目の素数さん
垢版 |
2020/08/13(木) 17:16:23.55ID:aEgkMNEo
>>343
> 次に正五角形の中心と頂点を斜辺xとする直角三角形においてピタゴラスの定理より、
> (5√5-5)^2/4=x^2-(10-x)^2
10-xって何?
0349132人目の素数さん
垢版 |
2020/08/13(木) 17:24:59.62ID:KCc316ag
>>314
正五角形の
外接円の半径と1辺の長さの比はかなりややこしい
対角線と1辺の長さの比はいわゆる黄金比
いずれもググると見つかるからそこから外接円の半径と対角線の比を求めて計算したほうが早い
イナはすぐに思い込みで適当やことをやって間違えるから信用しちゃダメだよ
0351132人目の素数さん
垢版 |
2020/08/13(木) 18:49:12.53ID:KhggCoPs
>>331
∂f/∂x = {1-2x(x-y)}e^(-xx-yy)
∂f/∂y = {-1-2y(x-y)}e^(-xx-yy)
これらを0とおくと
 x+y = 0,
 (x,y) = (1/2,-1/2) (-1/2,1/2)

あるいは 軸を45°回して
 u = (x+y)/√2,
 v = (x-y)/√2,
とおくと
 f = (√2)e^(-uu)・v・e^(-vv) = g(u)・h(v),
 g '(u) = -2(√2)u e^(-uu),
 h'(v) = (1-2vv)e^(-vv),
 (u,v) = (0,±1/√2)
∴ (x,y) = (1/2,-1/2) (-1/2,1/2)
0352132人目の素数さん
垢版 |
2020/08/13(木) 19:10:08.66ID:Uuox2URt
>>346
じゃ、これとの和と言うことで
Table[{n,89-2 Floor[(-1+Sqrt[1+8n])/2]+(n-(Floor[(-1+Sqrt[1+8n])/2]+1)(Floor[(-1+Sqrt[1+8n])/2])/2)^2-17(n-(Floor[(-1+Sqrt[1+8n])/2]+1)(Floor[(-1+Sqrt[1+8n])/2])/2)},{n,1,44}]
0353132人目の素数さん
垢版 |
2020/08/13(木) 19:15:33.71ID:KhggCoPs
>>345
(w,x,y,z) = (0,±1,0,0) (0,0,±1,0) (0,0,0,±1) … 6
    (±1/(2√3), ±1/√3, ±1/√3, ±1/√3) … 8 {(2w)xyz = 1/9}
計 14とおり
0354132人目の素数さん
垢版 |
2020/08/13(木) 19:35:33.47ID:GNxQ5zOj
>>347
(1)2√3<t<4√3
(2)BT=xとする。△BCTで余弦定理
(t-x)^2=(2√3)^2+x^2-4(√3)xcos60°
x=(t^2-12)/(2t-2√3)
(3)AS=3-√{(t^2/4)-3}
0355132人目の素数さん
垢版 |
2020/08/13(木) 19:43:32.85ID:ErCiafhA
I=[-a,a]としてf,f'がI上連続であるとき
∫_[-a,a]xf(x)dx = (2/3)a^3 f'(b)
となるb∈Iが存在することを示して下さい
0356132人目の素数さん
垢版 |
2020/08/13(木) 21:08:29.97ID:xAD4HTpt
>>350
a[n] = (n^2 + 1)
b[n] = (5n^2 + 9)
と置く。 a[n]b[n] が平方数でないことを示せばよい。
a[n] と b[n] の最大公約数 gcd(a[n], b[n]) を考える。
-5a[n] + b[n] = 4
より、 gcd(a[n], b[n]) は 4 の約数である。
a[n] が 4 で割り切れることはないので、 gcd(a[n], b[n]) は 1 か 2 のいずれかである。
したがって a[n] と b[n] の偶奇を調べれば、
n が偶数のとき、 gcd(a[n], b[n]) = 1
n が奇数のとき、 gcd(a[n], b[n]) = 2
となることがわかる。
以下、 a[n]b[n] が平方数でないことを n の偶奇に分けて示す。

【 n が偶数のとき】 gcd(a[n], b[n]) = 1 より、
もし a[n]b[n] が平方数ならば、 a[n] および b[n] も平方数である。
しかし、 n > 0 のとき n^2 < a[n] < (n+1)^2 より a[n] は平方数ではないので矛盾する。

【 n が奇数のとき】 gcd(a[n], b[n]) = 2 より、
もし a[n]b[n] が平方数ならば、 a[n]/2 および b[n]/2 も平方数である。
ここで n = 2k - 1 と置くと、 b[n] = 20k^2 - 20k + 14 より
b[n]/2 = 10k^2 - 10k + 7
となる。
すると b[n]/2 を 5 で割った余りは 2 となるが、平方数を 5 で割った余りは 2 にはならないので矛盾する。
0357132人目の素数さん
垢版 |
2020/08/13(木) 21:18:53.58ID:lLMkMecQ
ある工場で生産される精密部品を 25 個無作為抽出して長さを測ったら,平均値 x は x = 30 (mm) であった.過去の製造データの蓄積により, 製品の長さは標準偏差が 4 mmの正規分布に 従うことが分かっている.
区間の幅を 2.0=2×1.0 以下としたい.少なくとも何個の標本が必要か
61.46334 という値が出たとき答えは61ことしませんよね?
62個としますよね?
0358132人目の素数さん
垢版 |
2020/08/13(木) 21:28:09.71ID:xh5b0X6b
n^2+1=Nとおけば
N(5N+4)が平方数でないことを示せば良い
Nは平方数ではなく、非平方因子は5N+4の素因子にもなる必要があるからそれは2のみである
ところがN=2k^2とおいて代入すると5k^2+2が平方数であることになり矛盾
0359イナ ◆/7jUdUKiSM
垢版 |
2020/08/13(木) 23:24:00.80ID:bwxcosiq
>>343訂正。
>>314
求める長さをxcmとし、
正五角形の一辺の長さをycmとおくと、
対角線が10cmで、ほかの対角線によって10:y:10に分割され、この3つのパーツ10+y+10のうち10+yが一辺の長さと等しい。
∵一つの対角線と一つの辺が平行だから。
10(10+y)/(10+y+10)=y
100+10y=20y+y^2
y^2+10y-100=0
y=5√5-5
次に正五角形の中心と頂点を斜辺xとする直角三角形においてピタゴラスの定理より、
x+√[x^2-{(5√5-5)/2}^2]=√[10^2-{(5√5-5)/2}^2]
あとは計算。
5.2ぐらいかな?
0360イナ ◆/7jUdUKiSM
垢版 |
2020/08/14(金) 00:05:37.30ID:KtYwWebs
>>359
x+√[x^2-{(5√5-5)/2}^2]=√[10^2-{(5√5-5)/2}^2]
x^2+2x√[x^2-{(5√5-5)/2}^2]+x^2-{(5√5-5)/2}^2=[10^2-{(5√5-5)/2}^2]
2x^2+2x√[x^2-{(5√5-5)/2}^2]-75/2+25√5/2=[10^2-{(5√5-5)/2}^2]
0361イナ ◆/7jUdUKiSM
垢版 |
2020/08/14(金) 00:05:37.95ID:KtYwWebs
>>359
x+√[x^2-{(5√5-5)/2}^2]=√[10^2-{(5√5-5)/2}^2]
x^2+2x√[x^2-{(5√5-5)/2}^2]+x^2-{(5√5-5)/2}^2=[10^2-{(5√5-5)/2}^2]
2x^2+2x√[x^2-{(5√5-5)/2}^2]-75/2+25√5/2=[10^2-{(5√5-5)/2}^2]
0364132人目の素数さん
垢版 |
2020/08/14(金) 00:49:27.96ID:uDoX/Qiy
>>362
e^(iπ)=-1で計算するだけでは
nが奇数のときはeのとこ全部1になって-2と打ち消す
nが偶数のときはeのとこ全部-1になって-2と合わせて-4が出てくる
0365132人目の素数さん
垢版 |
2020/08/14(金) 01:02:33.93ID:OvK50PPf
>>364
それを知りませんでした
これもオイラーの公式なんですね…
ありがとうございました
0366132人目の素数さん
垢版 |
2020/08/14(金) 01:16:44.25ID:vL2Z3oJP
>>365 e^(iπ)+ 1 = 0はオイラーの等式っていう有名な等式なんですよ。
0368132人目の素数さん
垢版 |
2020/08/14(金) 04:09:09.55ID:N0QFEg7X
iを虚数単位とする。数列{a[n]}は
a[1]=1,a[2]=i
a[n+2]=a[n+1]+a[n]
を満たす。

また数列{b[n]}を
b[n]=a[n+1]/a[n]
で定める。

複素平面上で、b[x]が表す点をP[x]とする。j,k,lを相異なる自然数とし、3点P[j],P[k],P[l]が三角形となる場合を考える。
その面積S[j,k,l]について以下の問に答えよ。

(1)△P[j]P[k]P[l]の辺の長さをa,b,c(a≦b≦c)とするとき、ab/S[j,k,l]を求めよ。

(2)S[j,k,l]の取りうる値の範囲を求めよ。
0369132人目の素数さん
垢版 |
2020/08/14(金) 04:21:39.18ID:tUmyx/yQ
∫sin(x^k)dx[0→∞]は収束するか発散するか考察せよ。なおkは実数定数。
0370132人目の素数さん
垢版 |
2020/08/14(金) 04:43:27.85ID:Vqud894y
∫[0,∞] sin(x^k) dx = sign(k) ∫[0,∞] sin(y) (1/k)y^(1/k-1) dy

|k|>1 のとき収束 Γ(1+1/k) sin(π/2|k|),
|k|≦1 のとき発散
0371132人目の素数さん
垢版 |
2020/08/14(金) 04:51:06.15ID:Vqud894y
>>355
平均値の定理より
f(x) - f(-x) = 2x f '(ξ)  (-x<ξ<x)

∫[-a,a] xf(x) dx = ∫[0,a] x{f(x) - f(-x)} dx
 = ∫[0,a] 2xx f '(ξ) dx  (-x<ξ<x)
 = ∫[0,a] 2xx dx f '(b)
 = (2/3)a^3 f '(b).
0372132人目の素数さん
垢版 |
2020/08/14(金) 05:01:20.16ID:tUmyx/yQ
>>370
signとはなんですか??
0373132人目の素数さん
垢版 |
2020/08/14(金) 05:33:55.50ID:Vqud894y
>>368
題意より
 a[n] = F[n-2] + F[n-1]i,
 | a[n] |^2 = F[2n-3],
 b[n] = a[n+1]/a[n]
 = (F[n-1] + F[n]i) / (F[n-2] + F[n-1]i)
 = {F[n-1](F[n-2]+F[n]) - (-1)^n・i} / | a[n] |^2,
0376132人目の素数さん
垢版 |
2020/08/14(金) 10:08:25.62ID:cIdouH6q
>>349
プログラムで作図して計測して検証してみました。


> DOPs(5,T) # 五角形の1辺と対角線の長さ(外接円の半径=1)
$side
[1] 1.175570504584946

$diagonal
[1] 1.902113032590307

> DOPs(5)$diagonal/DOPs(5)$side # 対角線/辺長
[1] 1.618033988749895

> (1+sqrt(5))/2 # 黄金比
[1] 1.618033988749895
0377132人目の素数さん
垢版 |
2020/08/14(金) 10:16:33.43ID:cIdouH6q
>>376
# 対角線/辺長

> (2*sin(2*pi/5)) / (2*sin(pi/5))
[1] 1.618033988749895

> (1+sqrt(5))/2 # 黄金比
[1] 1.618033988749895
0378132人目の素数さん
垢版 |
2020/08/14(金) 10:29:48.89ID:Y2RmGzuY
>>352
出力が全然合わない
やはり難易度が違う
0379132人目の素数さん
垢版 |
2020/08/14(金) 10:30:46.24ID:cIdouH6q
>>345
-1,0,1で81通りを探索させたら、

w x y z
0 0 0 -1
0 0 -1 0
0 -1 0 0
0 1 0 0
0 0 1 0
0 0 0 1

6通りがひっかかった。
0380132人目の素数さん
垢版 |
2020/08/14(金) 10:55:19.63ID:Y2RmGzuY
Table[{n,41-2 Floor[(-1+Sqrt[1+8n])/2]+(n-(Floor[(-1+Sqrt[1+8n])/2]+1)(Floor[(-1+Sqrt[1+8n])/2])/2)^2-17(n-(Floor[(-1+Sqrt[1+8n])/2]+1)(Floor[(-1+Sqrt[1+8n])/2])/2)},{n,1,20}]


nを変化させると
まったく対応できない
0381132人目の素数さん
垢版 |
2020/08/14(金) 12:17:51.36ID:8Is1Irgf
>>371
ξ の値は当然 x に依存するわけですが、そのあたりはどうやって処理しているんでしょうか?
ξ = ξ(x)
0382132人目の素数さん
垢版 |
2020/08/14(金) 12:26:07.82ID:O2eyEomh
>>314
こういうのは丁寧に図を書いて平行線やら対称性を見ていけば解ける。

相似三角形より x/1 = 1/(x-1)
xx -x -1 = 0 ∴ x = ( 1 + √5 )/2   {黄金比}

sin(α/2) = (1/2) / x = ... = ( -1 + √5)/ 4
cos(α/2) = √(1- sin(α/2)^2 ) = ... = √(10 + 2√5) /4

x/2 = r * cos(α/2)
∴ r / x = 1/2cos(α/2) = ... = √{(5 - √5)/ 10} = 0.5257..

よって 10cm * r/x ≒ 5cm 3mm

右図は 検算?用に GeoGebraで描いた。
0384132人目の素数さん
垢版 |
2020/08/14(金) 12:48:55.39ID:GJ+vKVSe
座標上に作図して頂点の座標を計算で出せばどの対角線の長さも計算できる。
最も原始的だが汎用のある方法。
手計算だと大変だが一度、プログラムを組めば何角形になっても使える。
例:外接円の半径が1の正17角形の一辺の長さと対角線の長さを全て求めよ。
0385132人目の素数さん
垢版 |
2020/08/14(金) 13:00:56.30ID:O2eyEomh
正n角形に 一般化するとこうなる
半径: r = 1
最大幅: w=|e^{i2π/n *⌊n/2⌋} - 1|  {複素座標で描いた}
r/w = 1/√( {1-cos(2π⌊n/2⌋/n)}^2 + sin(2π⌊n/2⌋/n)^2 ) = 1/√( 2 -2cos(2π⌊n/2⌋/n) )
 nが偶数 ⇒ r/w = 1/√( 2 -2cos(π) ) = 1/2
 nが奇数 ⇒ r/w = 1/√( 2 -2cos(π - π/n) ) = 1/√( 2 +2cos(π/n) ) = 1/2cos(π/2n)
0386132人目の素数さん
垢版 |
2020/08/14(金) 13:13:05.60ID:SqBzt3dT
>>378

0 0 0 1 2 ...
みたいな数が並んでいるものが、何度も現れているので、それが目的かと思い、それを表現する式を >>340 で書いた
すると、>>346 で修正が必要と指摘され、87 71 85 70 55.... こそが目的で、惜しいと言われた。
そこで、>>352 で「修正」に当たる部分を表現した。

本当は、合わせた式にしたかったが、Wolframが「長すぎる」ことが原因だと思うが、理解してくれなかったから、
補正部分のみを書いた。改めて書く。

Table[Floor[Abs[n-(Floor[(Floor[Sqrt[2n+1]]+1)^2/2]-1/2-Floor[(Floor[Sqrt[2n+1]]+1)/2])]],{n,1,44}]
{0, 0, 0, 1, 0, 0, 1, 1, 0, 0, 1, 2, 1, 0, 0, 1, 2, 2, 1, 0, 0, 1, 2, 3, 2, 1, 0, 0, 1, 2, 3, 3, 2, 1, 0, 0, 1, 2, 3, 4, 3, 2, 1, 0}

Table[89-2 Floor[(-1+Sqrt[1+8n])/2]+(n-(Floor[(-1+Sqrt[1+8n])/2]+1)(Floor[(-1+Sqrt[1+8n])/2])/2)^2-17(n-(Floor[(-1+Sqrt[1+8n])/2]+1)(Floor[(-1+Sqrt[1+8n])/2])/2),{n,1,44}]
{87, 71, 85, 69, 55, 83, 67, 53, 41, 81, 65, 51, 39, 29, 79, 63, 49, 37, 27, 19, 77, 61, 47, 35, 25, 17, 11, 75, 59, 45, 33, 23, 15, 9, 5, 73, 57, 43, 31, 21, 13, 7, 3, 1}
の和
難易度は、そんなに高くない。面倒くさいだけ。
87 71 85 70 ...に当たる数字が、何段目の何列目の数字か、そしてそれが第何項に当たるかを求め、それら組み合わせて、
上のように分離された数を表す式を出すだけ。
例えば、一番左の数字は、1,3,6,10,...と三角数に当たる項数だけが来ているが、このようなものに注目して、逆算すればよい。
0387132人目の素数さん
垢版 |
2020/08/14(金) 15:05:44.98ID:TtI/FrBY
iを虚数単位とする。数列{a[n]}は
a[1]=1,a[2]=i
a[n+2]=a[n+1]+a[n]
を満たす。

また数列{b[n]}を
b[n]=a[n+1]/a[n]
で定める。

複素平面上で、複素数b[x]が表す点をP[x]とする。

(1)j,kを相異なる自然数とする。j,kを変化させるとき、線分P[j]P[k]の長さには最大値が存在することを示し、その値を求めよ。

(2)j<kとする。(1)の最大値を与える自然数の組(j,k)を全て決定せよ。
0388132人目の素数さん
垢版 |
2020/08/14(金) 15:07:30.28ID:8Is1Irgf
(1) 方程式 y^2 = x + √(x+1) の整数解 (x, y) を全て求めよ。

(2) (1)の一般化として、方程式
y^2 = x + √(x+n)
を考える。
任意の整数 n に対し、少なくとも一つは整数解 (x, y) が存在することを示せ。
また、整数解 (x, y) が無数に存在するような整数 n は存在するか?
0390132人目の素数さん
垢版 |
2020/08/14(金) 16:10:19.09ID:uDoX/Qiy
>>389
それn<0のときはダメじゃね

>>388
とりあえず(1)だけ

√(x+1)が整数なのでx=m^2-1(m∈Z)と書ける
y=|m|-k(k∈Z)とおいて代入すると
|m|=(k^2+1)/(2k+1)∈N、2k+1>0を得る
4|m|-2k+1=5/(2k+1)∈Zより
k=0,2の可能性しかない
このとき、それぞれ(x,y)=(0,1),(0,-1)でこれで全て
0391132人目の素数さん
垢版 |
2020/08/14(金) 16:20:48.02ID:uDoX/Qiy
(2)の後半
√(x+n)が整数なのでx=m^2-n(m∈Z)と書ける
y=|m|-k(k∈Z)とおいて代入すると
|m|=(k^2+n)/(2k+1)∈Zを得る
4|m|-2k+1=(4n+1)/(2k+1)∈Zより
2k+1は4n+1の約数でなければならず
kは有限個の可能性しかない
よって(x,y)も有限個の可能性しかない
0392132人目の素数さん
垢版 |
2020/08/14(金) 16:38:00.88ID:uDoX/Qiy
(2)の前半
n≧0のとき(x,y)=(n^2-n,±n)
n≦-1のとき(x,y)=(n^2+n+1,±n)
を解として持つ
0393132人目の素数さん
垢版 |
2020/08/14(金) 17:08:26.58ID:QALWwjXy
>>387
b1=i , b[n+1]=1+1/b[n]
(1)
n≧2のとき常に -π/4≦arg(1/b[n])≦π/4 かつ |1/b[n]|≦1 が成り立つ。
この扇形領域内の2点間の距離は√2が上限であるので、3以上の任意のj,kに対して|1/b[j-1]-1/b[k-1]|≦√2
ここで P[j]P[k]=|b[j]-b[k]|=|1/b[j-1]-1/b[k-1]| であるから P[1]P[2]=√5 が最大である。
(2) (j,k)=(1,2)
0394388
垢版 |
2020/08/14(金) 17:47:11.36ID:8Is1Irgf
>>390-392
ありがとうございます
やはり方程式 E[n] : y^2 = x + √(x+n) の整数解は有限個なんですね

整数 n に対し、>>392の整数解

> n≧0のとき(x,y)=(n^2-n,±n)
> n≦-1のとき(x,y)=(n^2+n+1,±n)

を方程式 E[n] の「自明な解」と呼び、
もし他の整数解をもつならば「非自明な解をもつ」と呼ぶことにします
例えば、>>388の(1)より、 E[1] は非自明な解をもたないことがわかります
一方、 E[2] は非自明な解 (x, y) = (-1, 0) をもちます
そこで次の問題を提出します

整数 n に対し、方程式
E[n] : y^2 = x + √(x+n)
が非自明な解 (x, y) をもつような n を全て決定せよ。
また、もし可能ならばそれらの解を全て求めよ。
0396132人目の素数さん
垢版 |
2020/08/14(金) 18:15:19.93ID:uDoX/Qiy
>>394
それはyも√(x+n)も整数であることから
一般にn=f(x,y)=(y^2-x)^2-xの形のとき(x,y)が解である
という当たり前の話になるのでは?
自明解と定義した値以外の整数組(x,y)に対してfの像は非自明解を持つnとなる
0397132人目の素数さん
垢版 |
2020/08/14(金) 18:27:05.63ID:cVTIbRA2
>>393
ありがとうございます
n=1,2,3...ですべてのP[n]が同一円周上にあることを発見できたので、まず円の直径を求めて、次にP[i]P[j]が直径の長さと等しくなるi,jを探そうとしました
そこで行き詰まったのですが、i=1,j=2だけだということで、読み返してもう一度解き直してみます
0398132人目の素数さん
垢版 |
2020/08/14(金) 18:37:47.62ID:8Is1Irgf
>>396
確かにそうですね
その言い換えで考えると、問題は
n=f(x,y)=(y^2-x)^2-x
が( y の符号の違いを除いて) 2 通り以上の (x, y) で表せる n はどのような数か?
ということになります
例えば、
1 = f(0, ±1)
2 = f(2, ±2) = f(-1, 0)
もう少し自明でない例を挙げると、
方程式 E[11] は (x, y) = (-2, ±1), (5, ±3), (110, ±11) を解にもつので、
11 = f(-2, ±1) = f(5, ±3) = f(110, ±11)
という 3 通りの表示をもつことがわかります
このような非自明な表示をもつ n はどのような数か?ということが知りたいです
0399132人目の素数さん
垢版 |
2020/08/14(金) 19:02:53.40ID:8Is1Irgf
>>398
どうやらこの言い換えは不完全なようです
(x, y) = (5, ±1) は E[11] の解ではありませんが、
11 = f(5, ±1)
と書けるので
何か条件が抜け落ちてしまったようです
0400132人目の素数さん
垢版 |
2020/08/14(金) 19:50:36.57ID:uDoX/Qiy
>>399
y^2-x≧0という条件がいる

>>398
元の証明に戻れば
|m|=(k^2+n)/(2k+1)∈Z
4|m|-2k+1=(4n+1)/(2k+1)∈Z
で、自明解とは2k+1=±1のときだから
非自明解を持つのは4n+1が(±素数)の形のとき、と言えそう
0402132人目の素数さん
垢版 |
2020/08/14(金) 20:22:13.27ID:oMJDqT0U
f(x,y)=(x-1)^2+(y-1)^2が
条件x^3+y^3-3xy=0のもとで最小値が存在することを示してそれを求めよという問題なのですが

未定乗数法を使うのは分かるのですが最小値の存在をどう示したらいいのか分かりません

条件が有界閉集合なら存在するみたいな感じですか?そもそも条件が有界閉集合なのかもわからず…

どなたかお願いします
0403132人目の素数さん
垢版 |
2020/08/14(金) 21:09:34.70ID:CE6P3k1H
あらかじめ最小値をとることがわかってなくとも、未定乗数法で最小値の候補を出して後から縁付きヘッセ行列を確認すればおk
0404132人目の素数さん
垢版 |
2020/08/14(金) 22:25:20.25ID:8Is1Irgf
>>400-401
> y^2-x≧0という条件がいる

なるほど
そうすると、整数 n, x, y に対し、
(x, y) が E[n] の解 ⇔ n = f(x, y) かつ y^2 ≧ x
となりますね
非自明な解をもつ場合よりも、非自明な解をもたない場合のほうが面白いかもしれません
上の同値から、特に y ≧ 0 の場合に限れば
E[n] は非自明な解をもたない ⇔ n = f(x, y) かつ y^2 ≧ x となる (x, y) は一組しかない
となります

>非自明解を持つのは4n+1が合成数のとき、か

なるほど! |4n+1| が合成数ならば、
4n+1 = pq, |p| > 1, |q| > 1 かつ p+q-2 ≧ 0
となるように p, q を選び、 2k+1 = p によって整数 k を定めると、
|m| = (p+q-2)/4 ≧ 0 であり、このとき
y = (q-p)/4 ≠ ±n
であるので、これによって非自明な解が得られますね

逆に、 |4n+1| が素数ならば、 |2k+1| = 1, |4n+1| より k = 0, -1, 2n, -(2n+1)
に限られるので、>>391の式から自明な解に限られることもわかりますね

以上より、整数 n, x, y および y ≧ 0 において、
E[n] は非自明な解をもたない ⇔ n = f(x, y) かつ y^2 ≧ x となる (x, y) は一組しかない
⇔ |4n+1| が素数
が成り立つ。

また、 |4n+1| が合成数のとき、
非自明な解は |4n+1| の素因数分解によって定まることもわかる。
( n < 0 のときは符号の制限に注意が必要)
0405132人目の素数さん
垢版 |
2020/08/14(金) 23:17:27.92ID:DH/eJ8n6
高校範囲での極限の難問とのことですが、初期条件の黒板の枚数・位置に関わらず1に収束するという結論が理解できずにいます。
時刻t=nでの黒板の枚数は計算できず、評価の仕方も分かりません。
よろしくお願いします。


【問題】
平面が合同な正三角形の板で隙間なく敷き詰められている。どの板も白色である。

時刻t=1において、1つの板を黒く塗る。
その後、各時刻t=2,3,...において、その時刻に存在する黒い板と辺を共有する白い板をすべて黒く塗る。時刻t=nにおける黒い板の枚数をa[n]とおく。

さて、時刻t=1において、平面上の任意のk枚(k≧2)の板を黒く塗り、上記と同様の操作で板を塗っていくことを考える。
t=1での板の塗り方(位置)によって、kが同じでも時刻t=nにおける黒い板の枚数は変化する。その最小値をm[n,k]、その最大値をM[n,k]とする。
このとき以下の極限がいずれも1に収束することを証明せよ。

lim[n→∞] m[n,k]/a[n]
lim[n→∞] M[n,k]/a[n]
0406351
垢版 |
2020/08/14(金) 23:41:48.86ID:Vqud894y
>>395
e^(-xx-yy) > 0 だから
 1 - 2x(x-y) = 0,  … (1)
 -1 -2y(x-y) = 0,  … (2)

(1)*y - (2)*x より
 y + x = 0,   … (3)
これを (1) に入れて
 1 -2x・2x = 0,
 x = ±1/2,
(3)を(2)に入れて
 -1 -2y(-2y) = -1 + (2y)^2 = 0,
 y = 干1/2,
このうち (3) を満たす組合せは
f(-1/2,1/2) = -e^(-1/2) = -1/√e = -0.60653 (最小)
f(1/2,-1/2) = e^(-1/2) = 1/√e = 0.60653  (最大)
の2つだけ。
0407132人目の素数さん
垢版 |
2020/08/14(金) 23:56:00.30ID:Vqud894y
なお、最大値・最小値だけでよければ
 f(x,y)^2 = e^(-2uu) (2vv)/e^(2vv) ≦ 1/e,

 e^(-2uu) ≦ 1,    (等号は u=0)
 e^(2vv) ≧ e(2vv),  (等号は 2vv=1)
0408132人目の素数さん
垢版 |
2020/08/15(土) 00:14:18.17ID:3iIf4ygs
>>405
とりあえず a[n] = 1 + 3n(n-1)/2 かな?
1 ≦ m[n,k]/a[n] ≦ M[n,k]/a[n]
だから、 lim[n→∞] M[n,k]/a[n] = 1 を示せば十分だということはわかる
直観的には、どんな初期条件であっても n が十分大きくなれば
黒い板は全部繋がってしまうから a[n] と M[n, k] は大差なくなる
ってことなんだろうか
0409132人目の素数さん
垢版 |
2020/08/15(土) 00:26:37.14ID:3iIf4ygs
>>408
あれ、違うかな?

>平面が合同な正三角形の板で隙間なく敷き詰められている。
>黒い板と辺を共有する白い板をすべて黒く塗る。

と書いてあるからアイゼンシュタイン整数みたいに敷き詰められていると想定したけど、
こういうふうに敷き詰められている可能性もあるな
https://upload.wikimedia.org/wikipedia/ja/thumb/7/7d/Shikitume02.gif/180px-Shikitume02.gif
この場合はどうやって塗っていくんだろ
もしこの場合は横にしか塗ってはいけないルールなら、
縦に k 個置けば明らかに M[n, k] = ka[n] だが
少しでも辺が触れていたら塗っていくルールなのかな
0410 【大吉】
垢版 |
2020/08/15(土) 00:27:33.54ID:+S/JbsGk
>>375
>>314
黄金比を既知としたら解いたことにならない。
対角線が一辺に対して黄金比であることは未知として解かないとなんにもなんないだろうが。
そんな答案不正解だからな。
0411132人目の素数さん
垢版 |
2020/08/15(土) 00:34:01.80ID:M0oPDaM8
>>405
何かが変だな

初期配置を固定したどんな2つの面積比も極限的には一致するだろうけど、各n時刻で最大値をとってきて比べてしまったら極限は変わってくる
特に最小配置と最大配置の比はkになって一致しない
0412132人目の素数さん
垢版 |
2020/08/15(土) 00:55:55.27ID:/A9LjrPH
問題自体がおかしいようですみません。問題の元となった原題を張ります。東大後期1997の第1問です。
この通りに文章で表現できたつもりが、浅はかでした。
正三角形での敷き詰めを文章で表現するのが難しいと思いました。

http://server-test.net/math/php.php?name=tokyo&;v1=1&v2=1997&v3=2&v4=1&y=1997&n=7
0413132人目の素数さん
垢版 |
2020/08/15(土) 01:06:17.30ID:M0oPDaM8
>>412
おいおい、話が全然違うやんけ

これなら十分大きなNをとれば
どんなk個の初期配置も単一配置のNステップ後の状態に覆われてる
よってa_n<b_n<a_(n+N)

a_n=1+3n(n+1)/2だから
a_n/b_n→1が挟み討ちの定理からわかる
0416イナ ◆/7jUdUKiSM
垢版 |
2020/08/15(土) 09:57:27.69ID:+S/JbsGk
>>410
>>314
求める長さをxcmとし、
正五角形の一辺の長さをycmとおくと、
対角線が10cmで、ほかの対角線によって10:y:10に分割され、この3つのパーツ10+y+10のうち10+yが一辺の長さと等しい。
∵一つの対角線と一つの辺が平行だから。
10(10+y)/(10+y+10)=y
100+10y=20y+y^2
y^2+10y-100=0
y=5√5-5
次に正五角形の中心と頂点を斜辺xとする直角三角形においてピタゴラスの定理より、
x+√[x^2-{(5√5-5)/2}^2]=√[10^2-{(5√5-5)/2}^2]
x^2+2x√[x^2-{(5√5-5)/2}^2]+x^2-{(5√5-5)/2}^2=10^2-{(5√5-5)/2}^2
2x^2+2x √[x^2-{(5√5-5)/2}^2]=100
x^2+x √[x^2-{(5√5-5)/2}^2]=50
x^2[x^2-{(5√5-5)/2}^2]=(50-x^2)^2
100x^2-{(5√5-5)/2}^2]x^2=2500
{40-(√5-1)^2}x^2=1000
(34+2√5)x^2=1000
(17+√5)x^2=500
(289-5)x^2=500(17-√5)
71x^2=125(17-√5)
71x^2=5^2(105-5√5)
x=5√{5(17-√5)/71}
=5.09831717999……
5.1もないね。妥当な値だ。
0418132人目の素数さん
垢版 |
2020/08/15(土) 10:06:49.26ID:icX1mGke
>>415
n^k が奇数だから n は奇数。
k≧2 のとき {左辺}≡2 (mod 4)
n≧2 のとき {右辺}≡0 (mod 4)
したがって k=1 または n=1
n=1のとき任意のkについて成り立つ。
k=1 かつ n≧2 のとき
2^n=(1+1)^n=Σ[r=0~n]nCr>n+1 であるから成り立たない。
0420132人目の素数さん
垢版 |
2020/08/15(土) 10:52:56.25ID:b0vYiwvB
s_p := n (if p = 0)
s_p := Σ_{k=1}^{n-1} k^p (if p ≧ 1)

s_pがnの多項式になることって自明ですか?自明じゃないですか?

ちなみに次数はp+1次になります。
0421132人目の素数さん
垢版 |
2020/08/15(土) 11:01:46.09ID:b0vYiwvB
自明であるようなそうじゃないような気がするので質問しました。
5秒でs_pがnの多項式か否か答えないといけないとするとnの多項式になると答えます。
0422132人目の素数さん
垢版 |
2020/08/15(土) 11:02:51.74ID:b0vYiwvB
p = 2の場合でいえば、

1^2 + 2^2 + … + (n - 1)^2がnの多項式かどうかということになります。
0424132人目の素数さん
垢版 |
2020/08/15(土) 11:12:17.41ID:VHOC4kxf
>>403
未定乗数法を使うには条件が正則でなければいけないみたいですがこれが正則であるかはどう示したら良いのでしょうか
0426132人目の素数さん
垢版 |
2020/08/15(土) 12:19:23.14ID:c5SrH6ui
鋭角三角形△ABCに内接する3つの異なる正方形と、その内部領域D1,D2,D3を考える。
ここで△ABCに正方形Sが内接するとは、Sの一辺が△ABCのいずれか一辺に含まれ、その辺上にない2頂点が残りの二辺上にあることを指す。
D1∩D2∩D3は空でないことを示せ。
0428132人目の素数さん
垢版 |
2020/08/15(土) 16:24:42.11ID:icX1mGke
>>427
例えば座標平面上で A(-1,0),B(100,0),C(0,100)とすると
BCの一部を辺とする正方形は垂心を含まない。
0429132人目の素数さん
垢版 |
2020/08/15(土) 16:26:21.74ID:66Dxz5iE
人間は知り合いの死による精神的動揺で寿命が縮む。
直接の知り合いのことを1次知り合いと呼ぶ。また「知り合いの知り合い」を2次知り合い、「知り合いの知り合いの知り合い」を3次知り合い、…とし、任意の2人は必ず6次以内の知り合いである。
一般にn次知り合いの相手が死んだとき、人間は寿命が[{32/2^(n-1)}-1]時間縮む。
渡哲也が死んだことで日本国民の寿命の総和がどれほど減少するか推定したい。
0430132人目の素数さん
垢版 |
2020/08/15(土) 17:53:07.49ID:6GTusgWt
>>387
プログラム組んで100までの自然数でやってみた。

a <- function(n){
x=complex(n)
x[1]=1
x[2]=1i
if(n<3) return(x[n])
for(i in 1:(n-2)){
x[i+2]=x[i+1]+x[i]
}
return(x[n])
}
b <- function(n) a(n+1)/a(n)
P <- function(j,k) abs(b(j)-b(k))
P=Vectorize(P)
N=100
j=k=1:N
z=outer(j,k,P)
max(z) ; sqrt(5)

idx=which(z==sqrt(5))
for(i in idx){
print(c(i%%N,i%/%N+1))
}

最大値は√5
> max(z) ; sqrt(5)
[1] 2.2360679774997898
[1] 2.2360679774997898

それを与える値は
> for(i in idx){
+ print(c(i%%N,i%/%N+1))
+ }
[1] 2 1
[1] 1 2
0431132人目の素数さん
垢版 |
2020/08/15(土) 19:20:54.33ID:icX1mGke
>>426
内心を含むから空でない。

辺AB上の点E、辺BC上の点F,G、辺CA上の点Hを頂点とする内接正方形EFGHが内心を含むことを示す。辺AB,辺ACの一部を一辺とする内接正方形についても同様。
点Hを中心とする半径GHの円をOとする。直線BEは円Oと交わるから∠EBH<∠GBH。ゆえに∠GBH>(1/2)∠ABCであるから、∠Bの二等分線は線分GHと交わる。
同様に∠Cの二等分線は線分EFと交わる。したがって内心は正方形EFGHの内部にある。
0432イナ ◆/7jUdUKiSM
垢版 |
2020/08/15(土) 19:43:07.05ID:+S/JbsGk
>>417
>>429
うちに渡哲也さんのサイン色紙があります。
渡哲也さんが亡くなったからといって自分のような一兵卒の寿命が縮むなんておそれ多いです。
0433132人目の素数さん
垢版 |
2020/08/15(土) 21:09:36.19ID:R1A01FWT
>>402
この問題、解析解って求められるんでしょうか?
自分も未定乗数法でやってみようと、しばらく式をコネ回してみたけど
簡単な形式には持っていけそうにありません。

Wolframも数値解しか出しません。
Minimize[ {(x-1)^2+(y-1)^2, x^3+y^3 -3x y == 0}, {x,y} ]
→ ...
0438132人目の素数さん
垢版 |
2020/08/16(日) 00:49:58.40ID:N4DeS3nO
∫sin(x^k)dx[0→∞]は収束するか発散するか。なおkは実数定数。
これについて詳しく教えてもらいたいです
0439132人目の素数さん
垢版 |
2020/08/16(日) 01:13:44.40ID:JAwBuHm3
>>438
kの値を変えてプログラムで実験してみたけど
> # ∫sin(x^k)dx[0∫sin(x^k)dx[0→∞]

> f <- function(k) integrate(function(x)sin(x^k),0,Inf)
> f(1)
Error in integrate(function(x) sin(x^k), 0, Inf) :
roundoff error is detected in the extrapolation table
> f(runif(1,1,10))
Error in integrate(function(x) sin(x^k), 0, Inf) :
maximum number of subdivisions reached
> f(runif(1,0,1))
Error in integrate(function(x) sin(x^k), 0, Inf) :
roundoff error is detected in the extrapolation table
> f(runif(1,-1,0))
Error in integrate(function(x) sin(x^k), 0, Inf) :
the integral is probably divergent
> f(runif(1,-10,-1))
Error in integrate(function(x) sin(x^k), 0, Inf) :
the integral is probably divergent

収束はしないみたい。
0440132人目の素数さん
垢版 |
2020/08/16(日) 01:16:29.79ID:N4DeS3nO
>>439
なんかすごいですね笑 これどうやって数学的に証明すればいいんですかね…
0442132人目の素数さん
垢版 |
2020/08/16(日) 01:30:07.26ID:pRhbC/47
フレネル積分を知っていれば k の値によって収束したり収束しなかったりすることは明らか
|x| > 1 なら収束する
|x| ≦ 1 なら収束しない
と予想
0444132人目の素数さん
垢版 |
2020/08/16(日) 01:35:46.88ID:N4DeS3nO
>>443
フレネル積分をどう応用したらいいんですかね…交項級数とかに繋がってくる感じですか??
0445132人目の素数さん
垢版 |
2020/08/16(日) 01:39:40.34ID:N4DeS3nO
フレネル積分∫sin(x^2)dx[0→∞]の収束性を示すのってどうやるんですか??
0449132人目の素数さん
垢版 |
2020/08/16(日) 02:12:48.08ID:N4DeS3nO
>>448
えええ、、どんな感じでつなげるのですか??宜しければ詳しく知りたいです
0451132人目の素数さん
垢版 |
2020/08/16(日) 03:02:14.88ID:940pKOPL
鋭角三角形△ABCの内部に点Pがあり、
∠PAB,∠PBC,∠PCAの値はそれぞれ分かっている。
この条件のみで、点Pの位置をただ1箇所に特定できるか。
0454132人目の素数さん
垢版 |
2020/08/16(日) 14:37:04.28ID:T/d8M/lm
二条を教えてください
0457132人目の素数さん
垢版 |
2020/08/16(日) 18:27:43.42ID:/uc0uymM
>>402
デカルトの葉線でござるか。与式から
1 = x^3 + y^3 + 1^3 - 3xy
 = (x+y+1)(xx+yy-xy-x-y+1),

f(x,y) = (x-1)^2 + (y-1)^2
 = {2(xx+yy-xy-x-y+1) + (x+y-2)^2}/3
 = {2/(x+y+1) + (x+y-2)^2}/3
 = {2/(s+1) + (s-2)^2}/3   (s=x+y)
 = (s^3 -3ss +6)/{3(s+1)},

df/ds = {2(s^3 -3s -3)}/{3(s+1)^2} = 0 より
 s = φ^(2/3) + φ^(-2/3) = 2.1038034 で最小
 {x,y} = {φ^(2/3), φ^(-2/3)},  t = xy = 1,

 f(x,y) ≧ 2 - φ^(5/3) + φ^(-5/3) = 0.21838195
 φ = (1+√5)/2 = 1.618034
0458132人目の素数さん
垢版 |
2020/08/16(日) 18:44:51.74ID:/uc0uymM
0 = x^3 + y^3 - 3xy
 = (x+y)(xx-xy+yy) - 3xy
 = s(ss-3t) - 3t,
より
 t = (s^3)/{3(s+1)},
0459132人目の素数さん
垢版 |
2020/08/16(日) 19:11:55.84ID:7pdtccLH
鋭角三角形△ABCの内部に点Pが与えられており、∠APB=x°,∠BPC=y°,∠CPA=z°である。
△ABCの内部の点Qで、∠AQB,∠BQC,∠CQAのいずれもx°またはy°またはz°に等しいものを考える。
以下の場合に、QをPとは異なる点にとることはできるか。

(1)x>y≧z
(2)x≧y>z
(3)x>y>z
0461132人目の素数さん
垢版 |
2020/08/16(日) 20:41:50.93ID:XfR5KAjl
しかし最近とくに、ここを自作問題投下スレと勘違いしてるんじゃないかと思うことが増えている気がする。
一応、ここで聞いているからには分からない問題なのだろうと解釈して回答をしているけれども。
自作問題を投下したいけど適切なスレが分からないということなら、素直にそう聞けばいいのに。
それともなんらかの意図を持った確信犯なのだろうか。
0462132人目の素数さん
垢版 |
2020/08/16(日) 21:11:30.57ID:lbhPh1Wz
自信がないからここで確認してるんじゃないの
もっと酷い場合は答えの用意もなくここで面白い問題になるか確認してることもありそう
0464132人目の素数さん
垢版 |
2020/08/16(日) 21:57:47.63ID:XfR5KAjl
>>462
それをするために適切なスレが他にあるのにわざわざここでやるってのがいまいちわからなくてね。
0466132人目の素数さん
垢版 |
2020/08/16(日) 23:13:48.65ID:pRhbC/47
自作ですが分からないので投下します

自然数 n に対し、一変数多項式 f[n](x) を以下のように再帰的に定める。
f[0](x) = x
f[n](x) = (f[n-1](x))^2 - n

各 n に対し、 f[n](x) の実数根のうち最大のものを a[n] とする。
定義より明らかに a[n] は実数の代数的整数である。

(1) n > 1 のとき、 a[n] は無理数か?
(2) lim[n→∞] a[n] は存在するか?
 存在するならば、それは無理数か?超越数か?
(3) g[n](x) ∊ Z[x] を a[n] の最小多項式とする。
  g[n](x) = f[n](x) となる n はどのような数か?そうでない n はどのような数か?
 また、そのような n および、そうでない n はそれぞれ無数に存在するか?
0467132人目の素数さん
垢版 |
2020/08/16(日) 23:31:11.41ID:f0CPlVcj
低次で何か面白そうな傾向は出たのかね?
それがなきゃ興味ないな
0468132人目の素数さん
垢版 |
2020/08/16(日) 23:37:41.54ID:pRhbC/47
>>467
以下に予想を書いておきますね

【予想(>>466)】
(1) a[n] は n > 1 で常に無理数
(2) lim[n→∞] a[n] は存在し、恐らく超越数
(3) n > 1 がsquare-freeかつそのときに限り g[n](x) = f[n](x)
 したがってどちらも無数に存在する
0471132人目の素数さん
垢版 |
2020/08/16(日) 23:48:12.56ID:pRhbC/47
>>469
やっぱりそうなるんですかね?
実はその数を根にもつ多項式を構成しようとして考えたのが f[n](x) です
したがってその数は高々 2^n 次の代数的整数であることがわかります
ただ、その数が f[n](x) の根の中で最大になるかどうかがわかりません
0473132人目の素数さん
垢版 |
2020/08/17(月) 00:12:22.94ID:SfLQkZLB
>>457
>df/ds = {2(s^3 -3s -3)}/{3(s+1)^2} = 0 より
> s = φ^(2/3) + φ^(-2/3) ...

ここでサクっと解が求まるのは、裏でヴィエトの解法を使ってますか?
それとも黄金比を使った王道パターンがあるのでしょうか?

ヴィエトの解法 (参考: 前スレ >>432)
s^3 -3s -3 = 0 の実解を求める
a = ... = 2
θ = ... = arccos( 3/2 ) / 3 { θは虚数となる }
... ∴ e^(+3θ*I) = √{ (3+√5)/2 }
s= a*cos(θ) = 2*cos(1/3 * ln((3+√(5))/2) * I ) {他の2解は複素数となる}
 = 2*cosh( 2/3 * ln( √{(6+2√5)/ 4} ) )
 = 2*cosh( 2/3 * ln( (1+√5)/ 2 ) )
 = e^{+2/3 * lnφ} + e^{-2/3 * lnφ}
 = φ^(2/3) + φ^(-2/3)
0475132人目の素数さん
垢版 |
2020/08/17(月) 00:19:46.53ID:IxYiNKnI
>>472
そうだよ
そして全て+を選択する場合が一番大きい
-を選択して途中で複素数になることもあるが絶対値は常に+を選択したものの方が勝ってるから大丈夫そう
0476132人目の素数さん
垢版 |
2020/08/17(月) 00:24:37.20ID:IxYiNKnI
てか一度複素数になる符号選択をしたらその先実数には帰って来ないことが示せるか
0479132人目の素数さん
垢版 |
2020/08/17(月) 01:03:08.50ID:ENNOwnx+
n次多項式f(x)で
f(1)f(2)...f(n)=f(n+1)
であるものを全て求めたいのですが分かりません。
1以上n以下のある整数mに対しf(m)=0、かつf(n+1)=0であれば成立するのは分かりました
0481132人目の素数さん
垢版 |
2020/08/17(月) 01:17:56.25ID:IxYiNKnI
>>479
一般にはニュートンの補間公式使って出せるはず
f(1)〜f(n)をまず任意に与えて、最後に(n+1,f(1)f(2)…f(n))を通るように設定する
0482132人目の素数さん
垢版 |
2020/08/17(月) 01:23:54.04ID:IxYiNKnI
ニュートンってかラグランジュか

具体的には
f(x)=Π[i=1,n](x-i)f(i)/n!+Σ[j=1,n]Π[i≠j](x-i)f(j)/(j-i)
0483132人目の素数さん
垢版 |
2020/08/17(月) 01:27:16.54ID:U7f6nYy/
 f(x) = n+1-x + (x-1)(x-2)・・・・(x-n),
のとき
 f(1)=n, f(2)=n-1, ・・・・, (n-1)=2, f(n)=1, f(n+1) = n!
0485132人目の素数さん
垢版 |
2020/08/17(月) 01:40:22.41ID:IxYiNKnI
>>482
例えば
n=2のとき
f(1)=a、f(2)=b、f(3)=abとして
f(x)=(x-1)(x-2)ab/2+(x-2)(x-3)a/2-(x-1)(x-3)b

n=3のとき
f(1)=a、f(2)=b、f(3)=c、f(4)=abcとして
f(x)=(x-1)(x-2)(x-3)abc/6-(x-2)(x-3)(x-4)a/6+(x-1)(x-3)(x-4)b/2-(x-1)(x-2)(x-4)c/2

このようにして一般の場合が求まる
0487132人目の素数さん
垢版 |
2020/08/17(月) 01:47:11.18ID:IxYiNKnI
>>486
上の例のn=2のときのaとbに9/4と9を代入すると出る
(x-1)(x-2)81/8+(x-2)(x-3)9/8-(x-1)(x-3)9=9/4x^2
0488132人目の素数さん
垢版 |
2020/08/17(月) 01:50:45.98ID:U7f6nYy/
>>481
 g(x) はn次以下の多項式
 f(x) = g(x) + {g(1)g(2)・・・・g(n) - g(n+1)}(x-1)(x-2)・・・・(x-n)/n!
かな?
0492132人目の素数さん
垢版 |
2020/08/17(月) 05:34:50.62ID:hHpaTy/b
画像の微分方程式の一般解の求め方を教えていただきたいです
院試が近いのですが高校レベルの数学すらあやふやなため全然分かりません
よろしくお願いします

https://i.imgur.com/zWFat3W.jpg
0493132人目の素数さん
垢版 |
2020/08/17(月) 10:44:30.40ID:d0IFupQZ
笠原の微分積分学にf(x) - f(0) 〜 g(x) - g(0)ならばf(x)〜g(x)と書いてありますが、なぜでしょうか?
φ(x)〜ψ(x)は両関数とも無限小でlim_{x->+0}φ(x)/ψ(x)=1を意味する記号です。
0494132人目の素数さん
垢版 |
2020/08/17(月) 10:49:22.70ID:d0IFupQZ
笠原の本持っている人はp.85の一番上に書いてあるので参照してください。
0497132人目の素数さん
垢版 |
2020/08/17(月) 11:28:32.40ID:d0IFupQZ
笠原の本ですが、φ(x)=(1/x)*log(x)+1/x^2+o(1/x^2)のときo(φ(x)^2)=o(1/x^2)であると書いてありますが、これはなぜですか?(p.96一番上のあたり)
0498132人目の素数さん
垢版 |
2020/08/17(月) 11:49:27.61ID:A0lhg88c
>>496
あ、失礼しました、そうですね
結局言えるのは
f(x)-f(0)〜g(x)-g(0) ‥@
かつ
(0)=g(0)=0‥A
ならば
fx)〜g(x)‥B
であつて@⇒Bはもちろん言えませんね
(反例はf(x)=1+x, g(x)=x など)
ホントに@⇒Bと読めるなら筆滑りの類いではないかと
0499132人目の素数さん
垢版 |
2020/08/17(月) 12:47:23.25ID:d0IFupQZ
笠原の本ですが、φ(x)=(1/x)*log(x)+1/x^2+o(1/x^2) (x->∞)のときo(φ(x)^2)=o(1/x^2)(x->∞)であると書いてありますが、これはなぜですか?(p.96一番上のあたり)
0500132人目の素数さん
垢版 |
2020/08/17(月) 13:12:07.37ID:9U9sXAI2
>>499
成り立たないと思う
f(x) = (φ(x)^2)/(log(x)) とすれば f(x) = o(φ(x)^2) だが、
x^2 f(x) = x^2 φ(x)^2 /(log(x)) = O(log(x)) → ∞
0501132人目の素数さん
垢版 |
2020/08/17(月) 13:34:48.91ID:d0IFupQZ
>>498
ありがとうございます。
>>500
ありがとうございます。
ランダウの記号関係の話はあまり他の本にも載っていない内容ですので、丁寧に書いてあったとしたら良かったのですが、非常に雑なんです。
ちなみに、>>499は途中の計算で出てくるんですが、最終的な結果(以下に書きます)は間違っていないでしょうか?(pp.95-96)
(1+x)^(1/x) = 1 + (1/x)*log(x) + (1/(2*x^2))*(log(x))^2 + 1/x^2 + o(1/x^2) (x->∞)
0502132人目の素数さん
垢版 |
2020/08/17(月) 13:41:44.61ID:d0IFupQZ
杉浦光夫の解析入門1のp.117問題1(iv)がまさに>>501の問題でした。
答えは、 1 + (1/x)*log(x) + (1/(2*x^2))*(log(x))^2 + o(|log(x)|^2/x^2) でした。
0503132人目の素数さん
垢版 |
2020/08/17(月) 13:43:04.42ID:d0IFupQZ
どちらがあっているのか、あるいは両方あっているのか、あるいは両方間違っているのか?どなたかわかる方いらっしゃいますか?
0504132人目の素数さん
垢版 |
2020/08/17(月) 14:27:26.15ID:U7f6nYy/
(1+x)^h = x^h・(1+1/x)^h

マクローリンで
 x^h = e^{h log(x)} = Σ[k=0,∞] (1/k!){h log(x)}^k,
一般二項公式で
 (1+1/x)^h = 1 + h/x + h(h-1)/(2x^2) + ・・・・,

ここで h→1/x とおく。

>>499
 φ(x)^2 = O({log(x)/x}^2) = o(1/x^2) (x→∞)

>>501 は正しい。

>>502 は x^(1/x) と思われる。
0505132人目の素数さん
垢版 |
2020/08/17(月) 14:37:27.75ID:A0lhg88c
(1+x)^(1/x)
=exp( (1/x)log(1+x) )
= 1 + (1/x)log(1+x)
. + (1/2)( (1/x)log(1+x) )^2
. +o( (1/x)log(1+x) )^2

(1/x)log(1+x)=(1/x)log(x)+1/x^2+o(1/x^2)
を代入したらできそう
0507132人目の素数さん
垢版 |
2020/08/17(月) 14:59:14.00ID:SfLQkZLB
>>492
(1) (x^2-y^2) y' = 2xy
∂F/∂x = -2xy q(x,y), ∂F/∂y = (x^2-y^2) q(x,y)
となる関数 F(x,y) を求めてみる。
問の微分方程式は dF = (∂F/∂x) dx + (∂F/∂y) dy = 0 と等価(※)である。(q(x,y)は積分因子)
∂∂F/∂x∂y = ∂∂F/∂y∂x を満たす必要 (連続条件とか積分可能条件とか) があるので、
-2xq -2xy ∂q/∂y = 2xq + (x^2-y^2) ∂q/∂x
(x^2-y^2)∂q/∂x + 2xy ∂q/∂y = -4xq {一見簡単になる気がしないが...}
q = 1/y^2 が条件を満たす。
よって ∂F/∂x= -2x/y,  ∂F/∂y= x^2/y^2 - 1
∴ F(x,y) = -x^2/y - y = 2R {積分定数}
x^2 + (y - R)^2 = R^2 つまり陰関数解は円である。
陽解は y = R ± √{R^2 -x^2}  (-|R| ≦ x ≦ +|R|)
傾きが無限になる点 (x=±|R|) は極限点として許容されるだろう。
積分因子を考慮すると y = 0 (定数解)もまた解である。これは円族の極限でもある。
(※等価が言えるのは方程式が有意味な範囲のみ)
0508132人目の素数さん
垢版 |
2020/08/17(月) 14:59:54.40ID:SfLQkZLB
>>492
(2) y' +2y = a*cos(2x)
Aを定数とすると
(Ae^{±i2x})’ + 2*Ae^{±i2x} = (2±2i)Ae^{±i2x}
よって特解は
y = a/2 * ( e^{+i2x}/(2+2i) + e^{-i2x}/(2-2i) )
 = a/2 * Re{ e^{+i2x}/(1+i) }
 = a/4 * Re{ (1-i)e^{+i2x} }
 = a/4 * ( cos(2x) + sin(2x) )
斉次一般解と合わせて
y = C * e^{-2x} + a/4 * ( cos(2x) + sin(2x) )
が一般解である。
0509132人目の素数さん
垢版 |
2020/08/17(月) 15:06:52.34ID:SfLQkZLB
>>492
(3) y'+y/x = x^m y^m
これはWolframでカンニング
 DSolve[ y'[x]+y[x]/x == x^m y[x]^m, y[x],x ]
 ... {{y[x]→(x^(1 + m)/2 - 1/2 m x^(1 + m) + x^(-1 + m) C[1])^(1/(1 - m))}}
よく分からんが ○^(1/(1 - m)) が肝なのだろう。 (こんなの思いつくわけがない...※)
f(x) = y^{1-m} と置いて
f' = (1-m) f/y * y' = (1-m) f * (x^m /f - 1/x )
∴ f' -(m-1)f/x -(1-m)x^m = 0
f' -p(x) f - q(x) = 0 タイプの方程式解 ((2)でも使えるパターン) ...
f(x) = e^{+∫dx p} ∫dx( q e^{-∫dx p} )  {p=(m-1)/x, q=(1-m)x^m}
= x^{m-1} ∫dx ( (1-m)x^m * x^{1-m} )
= (1-m) x^{m-1} ( 1/2 * (x^2 - C^2) )  {積分定数は後知恵で整えた}
よって
y = { (m-1)/2 * x^{m-1} ( C^2 - x^2 ) }^{1/(1-m)}  ( -|C| ≦ x ≦ |C| )
(もちろんWolfram解と等価である)

※手持ちの本をよく見たら、これはベルヌイの方程式のパターンだそうだ。
 たぶん見た事あるけど忘れてた。
0511132人目の素数さん
垢版 |
2020/08/17(月) 16:59:37.39ID:U7f6nYy/
>>492
@ (xx-yy)(dy/dx) = 2xy,

 x = r cosθ, y = r sinθ,
とおくと、与式は
 (dy/dx) = tan(2θ),
すなわち
 (動径OPと Pでの接線のなす角) = θ = (動径OPと x軸のなす角),
x軸が点Oでの接点だとすると、
2点O, Pにおける交角が相等しいことになる。
円周角の定理の逆により、Pの軌跡は O を通る円周。

同次形なので u = y/x とおく。
 x(du/dx) + u = (dy/dx) = 2xy/(xx-yy) = 2u/(1-uu),
より
 (1/x)dx = {(1-uu)/u(1+uu)}du = {1/u - 2u/(1+uu)}du,
 x = 2R u/(1+uu) = 2R xy/(xx-yy), {2R:積分定数}
 x^2 + (y-R)^2 = R^2,
0512132人目の素数さん
垢版 |
2020/08/17(月) 17:15:59.75ID:U7f6nYy/
>>492
B (dy/dx) + y/x = x^m y^m,  (mは2以上の定数)

 (1/x)(xy) ' = (xy)^m,
xy=v とおくと
 (1/x)v ' = v^m,
 v^(-m) v' = x,
積分して
 -{1/(m-1)}v^(1-m) = -(CC-xx)/2,
 y = v/x = (1/x){(m-1)(CC-xx)/2}^{1/(1-m)},
0513132人目の素数さん
垢版 |
2020/08/17(月) 17:27:47.63ID:9U9sXAI2
>>511
> x = 2R u/(1+uu) = 2R xy/(xx-yy), {2R:積分定数}
> x^2 + (y-R)^2 = R^2,

違くね
上は双曲線
下は円
0514132人目の素数さん
垢版 |
2020/08/17(月) 17:30:26.65ID:SfLQkZLB
>>509
(3)の解はやや不正確なので少し訂正

m: 偶数 ⇒ y = + { (m-1)/2 * x^{m-1} ( C - x^2 ) }^{1/(1-m)}  
定義域: C>0 ⇒ { x<-C, 0<x<+C },  C≦0 ⇒ { x<0 },

m: 奇数 ⇒ y = ± { (m-1)/2 * x^{m-1} ( C - x^2 ) }^{1/(1-m)}  
定義域: C>0 ⇒ { |x|<C },  C≦0 ⇒ 実解なし

とにかく 根号内は正で y^(1-m) が 〜 になれば良い。
0515132人目の素数さん
垢版 |
2020/08/17(月) 17:43:29.81ID:SfLQkZLB
>>514 (再訂正)
m: 偶数 ⇒
 y = + { +(m-1)/2 * x^{m-1} ( C - x^2 ) }^{1/(1-m)}  
 定義域: C >0 ⇒ { x<-C, 0<x<+C },  C≦0 ⇒ { x<0 }

 y = - { -(m-1)/2 * x^{m-1} ( C - x^2 ) }^{1/(1-m)}  
 定義域: C >0 ⇒ { -C<x<0, +C< x},  C≦0 ⇒ { x>0 }

yが負の領域に解は無いんか?と気付くべきだった。
0517132人目の素数さん
垢版 |
2020/08/17(月) 18:52:36.21ID:8hGqtSan
>>386
wolfram入力可能に短縮した

Table[2n-1+C(0,n-2)+3C(1,(10mod n)-2)+5C(1,n-11)+3C(0,C(0,C(3,n-16)))+11C(0,n-22)+C(0,C(0,C(6,n-29))),{n,1,44}]
0518132人目の素数さん
垢版 |
2020/08/17(月) 18:56:04.87ID:8hGqtSan
わからない問題は

わからないままにしておくことによって

人々は幸せになる
0520132人目の素数さん
垢版 |
2020/08/17(月) 21:04:44.80ID:d0IFupQZ
>>499

>>499
笠原のをほぼ書き写しました。誰か解読してください。

f(x) = (1 + x)^(1/x)
log(f(x)) = (1/x)*log(1 + x) = (1/x)*log(x) + (1/x)*log(1 + 1/x) = (1/x)*log(x) + (1/x)*(1/x + o(1/x))
= (1/x)*log(x) + 1/x^2 + o(1/x^2) (x->∞)

f(x) = exp((1/x)*log(x) + 1/x^2 + o(1/x^2))
log(f(x))の各項はx->∞のときすべて無限小だからテイラー公式から
f(x) = 1 + ((1/x)*log(x) + 1/x^2 + o(1/x^2)) + (1/2)*((1/x)*log(x) + 1/x^2 + o(1/x^2))^2 + o(1/x^2) ← これがなぜ成り立つのか分からない。
= 1 + (1/x)*log(x) + (1/(2*x^2))*(log(x))^2 + 1/x^2 + o(1/x^2)) (x->∞)
0521132人目の素数さん
垢版 |
2020/08/17(月) 21:09:21.88ID:d0IFupQZ
>>520
こういう漸近展開は何がうれしくてこんな風に展開しているんですか?
xが大きい時にf(x)と1 + (1/x)*log(x) + (1/(2*x^2))*(log(x))^2 + 1/x^2の値が近いということが言えて嬉しいということですか?
でもどれくらいxが大きいときに、誤差はどれくらいといった情報は得られませんから意味あるのかな?って思いました。
0522132人目の素数さん
垢版 |
2020/08/17(月) 21:35:29.73ID:A0lhg88c
>>521
まぁ言ってる事にそもそも意味が感じられないし面白くもないというなら無理して読まなくてもいいんではない?
ただ
f(x) = 1 + ((1/x)*log(x) + 1/x^2 + o(1/x^2)) + (1/2)*((1/x)*log(x) + 1/x^2 + o(1/x^2))^2 + o(1/x^2)
すなわち

f(x) = 1 + ((1/x)*log(x) + 1/x^2 + p(x)) + (1/2)*((1/x)*log(x) + 1/x^2 + q(x))^2 + r(x)
for some
p(x), q(x), r(x)
such that
lim p(x)x^2 = lim q(x)x^2 = lim r(x)x^2 = 0

くらいは自分で示せないとこの先何勉強してても行き詰まるよ
0523132人目の素数さん
垢版 |
2020/08/17(月) 21:59:59.60ID:IxYiNKnI
確かに言われてみればオーダー計算の正当性を示すことってあんま無いかもね
expくらい行儀のいい関数なら大丈夫だろうとやってしまいがちだけど特に物理とか
0524132人目の素数さん
垢版 |
2020/08/17(月) 23:06:44.14ID:9U9sXAI2
成り立たないんじゃないの?
誤差項付きのテイラー展開
exp(h) = 1 + h + h^2/2 + o(h^2) as (h→0)
において
h = (1/x)*log(x) + 1/x^2 + o(1/x^2) as (x→∞)
と置いたように見えるけど
h^2 には (log(x))^2 / x^2 の項が含まれるから x^2 を掛けたら発散するはず
o(h^2) = o((log(x))^2 / x^2) as (x→∞)
だと思うが
例えば
log(x)/x^2 = o((log(x))^2 / x^2) as (x→∞)
だが、 log(x)/x^2 = o(1/x^2) as (x→∞) ではない
0525132人目の素数さん
垢版 |
2020/08/17(月) 23:12:02.48ID:hHpaTy/b
>>507ID:SfLQkZLBさんと>>511ID:U7f6nYy/さん丁寧に解説ありがとうございます
あとでじっくり勉強させていただきます

>>510
ありがとうございます
教科書とノートで解法を確認してみます
0526132人目の素数さん
垢版 |
2020/08/17(月) 23:23:55.21ID:IxYiNKnI
>>524
ここでo(h^k)はo(h ^k)/h^k→0(h→0)の意味じゃないの?
oのとこに入ってるのは(logx)^3/x^3
>>520の式でも(logx)^2/x^2の項はoの外にいるから問題なさそうに見える
0527132人目の素数さん
垢版 |
2020/08/17(月) 23:37:43.30ID:A0lhg88c
まぁランダウのoとかOは
exp(x)=1+x+x^2/2+O(x^3)
を実際に
exp(x)=1+x+x^2/2+p(x)
for some p(x)
such that
|p(x)/x^3| is bdd.
と書き直してみてみる事を何回かやらないと勘が身につかない
理屈だけわかってもそういう経験に裏打ちされた地力は身につかないからな
数学やるような奴はそこそこ地頭はいいのでそういう修練で身についてくる力を軽視しがち
0528132人目の素数さん
垢版 |
2020/08/17(月) 23:44:44.45ID:9U9sXAI2
>>526
>>522と同じように書けば、
g(h) = o(h^2) as (h→0) の定義は lim[h→0] g(h)/h^2 = 0 でしょ?
h^2 = (log(x))^2 / x^2 + o(1) as (x→∞)
だから誤差項に (log(x))^2 / x^2 が残ってしまうと思う

> oのとこに入ってるのは(logx)^3/x^3

3乗はどこから出てきた?
O (ビッグオー)じゃなくて o (スモールオー)の話では?
0529132人目の素数さん
垢版 |
2020/08/17(月) 23:50:21.29ID:A0lhg88c
>>628
exp(t)=1+t+t^2/2+O(t^3)

t=(1/x)log(x) + 1/x^2 + o(1/x^2)
を“代入”できるならわかる
“代入”できないなら
exp(t)=1+t+t^2/2+p(t)
t= (1/x)log(x) + 1/x^2 + q(x)
とキチンとp(x), q(x)と定数
|p(x)|<M|t^3|
とおいてみてホントに代入してみればわかる
0531132人目の素数さん
垢版 |
2020/08/17(月) 23:53:41.61ID:A0lhg88c
おっと
|p(t)|<M|t^3|
ね。
代入すると
|p((1/x)log(x)+1/x^2+q(x))|
<M |((1/x)log(x)+1/x^2+q(x))^3|
となる
コレが何を意味するか
0532132人目の素数さん
垢版 |
2020/08/17(月) 23:56:51.92ID:9U9sXAI2
>>529
>exp(t)=1+t+t^2/2+O(t^3)

ああ、そっちを使うのか
それなら確かに高々
O((log(x))^3 / x^3) だから x^2 O((log(x))^3 / x^3) = o(1) as (x→∞)
だね
0534132人目の素数さん
垢版 |
2020/08/18(火) 00:37:28.47ID:ymr8iYI5
>>499
意味不明な
 o(φ(x)^2) = o(1/x^2)   (x→∞)
が混乱の元か?

>>516
 x^2 f(x) = O(log(x)) は、じゅうぶん大きいxについて
 m < | x^2 f(x)/log(x) | < M   (0<m<M<∞)
の意味とすれば、証明になってる。

>>520

log(x+1) = log(x) + log(1 + 1/x)
 = log(x) + 1/x - 1/(2x^2) + 1/(3x^3) - ・・・,     (マクローリン)

log(f(x)) = (1/x)log(x+1)
 = (1/x){log(x) + 1/x - 1/(2x^2) + 1/(3x^3) - ・・・・ }

f(x) = exp[(1/x){log(x) + 1/x - 1/(2x^2) + 1/(3x^3) - ・・・・ }]
 = 1 + (1/x){log(x) + 1/x - 1/(2x^2) + 1/(3x^3) - ・・・・ }
  + {1/(2x^2)}{log(x) + 1/x - 1/(2x^2) + ・・・・ }^2
  + {1/(6x^3)}{log(x) + 1/x - ・・・・ }^3
  + {1/(24x^4)}{log(x) + ・・・・ }^4
  + ・・・・               (マクローリン)
となる。
1/x^2 の項まで残し 1/x^3, 1/x^4, ・・・・ の掛かった項を無視すれば
 1 + {log(x) + 0}/x + {(log(x))^2 /2 + 0・log(x) + 1}/x^2,
0535132人目の素数さん
垢版 |
2020/08/18(火) 00:41:35.53ID:DzjNlXfM
結局、ペアノの剰余項を使うと上手くいかないが、
ラグランジュの剰余項を使えば上手くいくって話かな
0536132人目の素数さん
垢版 |
2020/08/18(火) 01:34:40.92ID:diu24p4P
41000
14100
01410
00141
00014

のように、対角成分が4、それに隣り合う成分が1、他は0のn×nの対照行列の逆行列をnを使って表せ、というのがわかりません。
0539132人目の素数さん
垢版 |
2020/08/18(火) 07:33:33.22ID:kwUXr7cM
>>536
4の代わりにxってのが定番
0540132人目の素数さん
垢版 |
2020/08/18(火) 07:41:12.73ID:3nFsHLKc
>>536
問題の対称行列をAとする
ここで漸化式
f(0)=0, f(1)=1, f(n+2)+4f(n+1)+f(n)=0
を考えると、これは特性根α=-2+√3を使って
f(n)=(α^n-α^(-n))/2√3と解ける

n次行列Bをこのf(x)と階段関数θ(x)を使って
Bij= θ(i-j)f(i-j)-f(i)f(n+1-j)/f(n+1)
と定めるとこれがAの逆行列になっている

実際
恒等式f(n+1)f(i-j)=f(i)f(n+1-j)-f(n+1-i)f(j)からBは
Bij=θ(j-i)f(j-i)-f(j)f(n+1-i)/f(n+1)
とも書ける(対称行列である)が、この2通りの表示を使うことでAの逆行列であることが以下のように確かめられる
f(n)が漸化式を満たすことにより
縦ベクトル{f(i)}_iはAの第n行以外の横ベクトルたちと
縦ベクトル{θ(i-j)f(i-j)}_iはAの第j行と第n行以外の横ベクトルたちと
縦ベクトル{f(n+1-i)}_iはAの第1行以外の横ベクトルたちと
縦ベクトル{θ(j-i)f(j-i)}_iはAの第j行と第1行以外の横ベクトルたちと直交している
よって上の2通りのBの表示から縦ベクトル{Bij}_iはAの第j行以外の横ベクトルたち{Aik}_k(i≠j)と直交している
そして第j列の横ベクトル{Ajk}_kとの内積は1×(-θ(j+1-j)f(j+1-j))=-f(1)=1となる
これはAB=Eを意味している

うーん、もっといい表示と証明があるかも
まあ実際に行列を書いてみれば上の証明でやってることはとても単純な計算
0541132人目の素数さん
垢版 |
2020/08/18(火) 07:49:33.28ID:3nFsHLKc
>>540
訂正
誤 そして第j列の横ベクトル{Ajk}_kとの内積は1×(-θ(j+1-j)f(j+1-j))=-f(1)=1となる

正 そして第j行の横ベクトル{Ajk}_kとの内積は1×θ(j+1-j)f(j+1-j)=f(1)=1となる
0542132人目の素数さん
垢版 |
2020/08/18(火) 09:11:25.95ID:0OeuZ5oX
なぜ対数関数のテイラー展開はlog(1+x) = x - (1/2)*x^2 + …とlogの引数に1+xを渡した形の関数を展開しているのでしょうか?
log(x) = ∫_{1}^{x} 1/t dtと、積分範囲の下端が1なので確かに、xが1からプラスマイナスどれくらいかで考えるのは自然だとは思いますがなんか不思議です。
log(1+x)と引数を1+xにするとテイラー展開がシンプルになる深い理由はありますか?
0543132人目の素数さん
垢版 |
2020/08/18(火) 09:59:43.89ID:nwyPKwDv
半径の異なる交わらない二つの円A,Bがある。
A,Bに接する円Cの中心の軌跡はどのような曲線か?
0545132人目の素数さん
垢版 |
2020/08/18(火) 10:22:23.91ID:0OeuZ5oX
>>520
みなさんありがとうございました。

自分なりの解答を作りました。

f(x) = exp((1/x)*log(x) + 1/x^2 + o(1/x^2))
log(f(x))の各項はx->∞のときすべて無限小だからテイラー公式から

f(x) = 1 + ((1/x)*log(x) + 1/x^2 + o(1/x^2)) + (1/2)*((1/x)*log(x) + 1/x^2 + o(1/x^2))^2 + (1/6)*((1/x)*log(x) + 1/x^2 + o(1/x^2))^3 + o(((1/x)*log(x) + 1/x^2 + o(1/x^2))^3)

o(((1/x)*log(x) + 1/x^2 + o(1/x^2))^3) = o(1/x^2)である。
証明:
p(x) = o(((1/x)*log(x) + 1/x^2 + o(1/x^2))^3)とする。
p(x) / (1/x^2) = [p(x) / ((1/x)*log(x) + 1/x^2 + o(1/x^2))^3]*[((1/x)*log(x) + 1/x^2 + o(1/x^2))^3 / (1/x^2)]
p(x) / ((1/x)*log(x) + 1/x^2 + o(1/x^2))^3 -> 0 (x->∞)は仮定から成り立つ。
((1/x)*log(x) + 1/x^2 + o(1/x^2))^3 / (1/x^2) = (log(x))^3/x + (log(x))^2/x^2 + log(x)/x^3 + 1/x^4 + o(1/x^2)
(log(x))^3/x, (log(x))^2/x^2, log(x)/x^3, 1/x^4はすべて無限小であるから
((1/x)*log(x) + 1/x^2 + o(1/x^2))^3 / (1/x^2) -> 0 (x->∞)が成り立つ。
∴p(x) = o(1/x^2)である。

∴f(x) = 1 + ((1/x)*log(x) + 1/x^2 + o(1/x^2)) + (1/2)*((1/x)*log(x) + 1/x^2 + o(1/x^2))^2 + o(1/x^2)
0546132人目の素数さん
垢版 |
2020/08/18(火) 11:30:41.89ID:fCQwYtO8
>>542
それはテイラー展開ではなくマクローリン展開だ。
logはx=0で定義されないんだからx=1でテイラー展開するのは自然な話で、それをマクローリン展開で実現しようとすればlog(1+x)とするのは当然のこと。
0548132人目の素数さん
垢版 |
2020/08/18(火) 12:35:58.45ID:zQe0mW85
数ある正の数で選ぶなら1がきれいだとは思わんの?
どれかひとつ固定したら後は平行移動すればいいだけだし、わざわざ汚くなるような値を使う必要がない
0549132人目の素数さん
垢版 |
2020/08/18(火) 12:37:44.90ID:0OeuZ5oX
>>548
なぜx=1を選ぶと綺麗になるのでしょうか?その理由が知りたいです。
0550132人目の素数さん
垢版 |
2020/08/18(火) 12:38:04.14ID:DzjNlXfM
x = 0 における n 階の微分係数がきれいになるからくらいの理由じゃないの
0551132人目の素数さん
垢版 |
2020/08/18(火) 12:39:35.24ID:0OeuZ5oX
>>550
log(x) = ∫_{1}^{x} 1/t dtの積分範囲の下端が1であることと何か関係がありますか?
0552132人目の素数さん
垢版 |
2020/08/18(火) 12:43:06.67ID:DzjNlXfM
>>551
あんまり関係ない
強いて言うなら、 log(1) = 0 となるくらい
実際に log(a+x) の n 階微分を計算してみればいいんじゃね
0553132人目の素数さん
垢版 |
2020/08/18(火) 19:49:10.44ID:ymr8iYI5
>>540

問題のn次対称行列の行列式を f(n) とおくと
 f(0) = 1, f(1) = x, f(2) = xx-1,
(1行目、n行目、1列目、n列目のどれかを)展開すると、漸化式は
 f(n+2) - x f(n+1) + f(n) = 0,

一方、第二種チェビシェフ多項式 は
 U_0(x/2) = 1, U_1(x/2) = x, U_2(x/2) = xx-1,
また、和積公式
 sin((n+3)θ) - 2cosθ・sin((n+2)θ) + sin((n+1)θ) = 0,
から
 U_{n+2}(x/2) - x U_{n+1}(x) + U_n(x/2) = 0,

これらを見比べれば
 f(n) = U_n(x/2),
となることは明らか。
0554132人目の素数さん
垢版 |
2020/08/18(火) 19:53:08.45ID:ymr8iYI5
(訂正)
また、和積公式
 sinh((n+3)θ) - 2coshθ・sinh((n+2)θ) + sinh((n+1)θ) = 0,

チェビシェフ多項式そのものは両方に使えるので。。。
0555132人目の素数さん
垢版 |
2020/08/18(火) 20:07:12.93ID:ymr8iYI5
(続き)
x=4 だから
f(n) = U_n(x/2)
 = U_n(2)
 = sinh((n+1)α)/(sinh α)
 = {(2+√3)^{n+1} - (2-√3)^{n+1}) / (2√3),
ここに
 cosh α = 2, sinh α = √3,
0556132人目の素数さん
垢版 |
2020/08/18(火) 20:34:43.94ID:3nFsHLKc
三項間漸化式を解くのになぜそこまでするんだ
それに問題は行列式ではなく逆行列を求めること
0557132人目の素数さん
垢版 |
2020/08/18(火) 20:46:41.88ID:ymr8iYI5
>>543
円A,Bの外部から接する円Cの中心は、小さい方の円寄りの枝にあり
円A,Bを内包して接する円Cの中心は、大きい方の円寄りの枝にある。
0558132人目の素数さん
垢版 |
2020/08/18(火) 21:24:39.30ID:fCQwYtO8
>>543
ともに外接する円とともに内接する円で一対の双曲線(焦点間の距離が半径の差)
一方に外接し、他方に内接する円でもう一対の双曲線(焦点間の距離が半径の和)
で、計2対4本の双曲線
0560132人目の素数さん
垢版 |
2020/08/19(水) 00:07:45.14ID:uCfGWuf7
a,bをa>b>0の実数とする。
原点をOとするxy平面上の楕円C:(x^2/a^2)+(y^2/b^2)=1の焦点のうち、x座標が正であるものをFとする。
Cの周上を2点P,Qが∠POQ=θ(0<θ<π)であるように動くとき、∠PFQ=θとなるP,Qの座標をa,b,θで表せ。
0562132人目の素数さん
垢版 |
2020/08/19(水) 01:02:32.55ID:3QgWIytr
>>543
交わらないってのは一方の円がもう一方の円の内部にある場合ってのもある。
その場合は2円の中心を焦点とする楕円。2円の片方の半径を∞にすると直線になり、もう片方の半径をゼロにすると点になり
その場合は放物線になる。(ゼロにしなくてもいいが)
0563132人目の素数さん
垢版 |
2020/08/19(水) 03:12:55.16ID:igft8RME
>>560
離心率: e = √(1-b^2/a^2) と置いて
O=(0,0), F=(e*a,0) の2点を通る円 :
 (x - e*a/2)^2 + (y - √{ r^2 -(e*a/2)^2 })^2 = r^2
と 楕円C との交点 P, Q {rの関数} を求める。
正弦定理より 2r sinθ = |PQ|、これより逆算して r = .. {θの関数}
よって P=(.. , ..),  Q=(.. , ..) {θの関数}
を得る。
シンプルな式になる気配が感じられないけど、もしかして自作問題?
0564132人目の素数さん
垢版 |
2020/08/19(水) 16:54:00.14ID:sxH2M8gd
>>557 >>558

円Aの中心を A(d/2,0) 半径を r_a
円Bの中心を B(-d/2,0) 半径を r_b
円Cの中心を P(x,y) 半径を R
 r_a + r_b < d = AB
とする。

・CがA,Bの外部から接するとき
 R = AP - r_a = BP - r_b,
 x = (rb-ra)√{1/4 - yy/[dd-(rb-ra)^2]},
・CがA,Bを内包するとき
 R = AP + r_a = BP + r_b,
 x = (ra-rb)√{1/4 - yy/[dd-(ra-rb)^2]},
・CがAの外部から接し、Bを内包するとき
 R = AP - r_a = BP + r_b,
 x = -(ra+rb)√{1/4 - yy/[dd-(ra+rb)^2]},
・CがAを内包し、Bの外部から接するとき
 R = AP + r_a = BP - r_b,
 x = (ra+rb)√{1/4 - yy/[dd-(ra+rb)^2]}.
0565132人目の素数さん
垢版 |
2020/08/19(水) 17:00:25.25ID:sxH2M8gd
訂正 スマソ.

 x = (rb-ra)√{1/4 + yy/[dd-(rb-ra)^2]},

 x = (ra-rb)√{1/4 + yy/[dd-(ra-rb)^2]},

 x = -(ra+rb)√{1/4 + yy/[dd-(ra+rb)^2]},

 x = (ra+rb)√{1/4 + yy/[dd-(ra+rb)^2]}.
0566132人目の素数さん
垢版 |
2020/08/19(水) 17:35:33.17ID:sxH2M8gd
>>562

円Aの中心を A(d/2,0) 半径を r_a
円Bの中心を B(-d/2,0) 半径を r_b
円Cの中心を P(x,y) 半径を R
 d + r_a < r_b  (円Aが円Bに内包される)
とする。

・CがAの外部から接し、Bに内接するとき
 R = AP - r_a = r_b - BP,
 x = ±(ra+rb)√{1/4 − yy/[(ra+rb)^2 -dd]},

・CがAを内包し、Bに内接するとき
 R = AP + r_a = r_b - BP,
 x = ±(ra-rb)√{1/4 − yy/[(rb-ra)^2 -dd]}
0568132人目の素数さん
垢版 |
2020/08/19(水) 22:22:09.07ID:Fkw4mj3H
>>567
(a, b) = (1, 1), (2, 1) のみだと思われる
整数の問題というよりも不等式の問題だろうか
a ≦ 2 のときは、必要条件 b ≦ a より総当たりで上の解に限られることがわかる
a > 2 なら、証明はわからないが、不等式
a^a - (a+b)^(a-b) > a - b
が成立するはず
グラフを書けば明らかに思えるが、上手い証明が思いつかない
というかこれも自作問題か?
0569132人目の素数さん
垢版 |
2020/08/19(水) 23:59:41.32ID:G5+SSCRd
f(x)=x³-x²-x-1, g(x)=x²-x-1 とする

(1) xの方程式 f(x)=0 はただひとつの実数解αをもち、それは 1<α<2 であることを示せ
(2) xの方程式 g(x)=0 の正の解をβとして、αとβの大小を比較せよ
(3)α²とβ³の大小を比較せよ

(3)の一般に想定される解法はβの具体的な値を求めてから、その値を字数を下げた式
β³=β・β²=β(β+1)=β²+β=(β+1)+β=2β+1
に代入し、1<α²<4 と比較するものです

しかし、もっと鮮やかな解法がありそうな気がします。もし思いついた人があれば教えてください。
0570132人目の素数さん
垢版 |
2020/08/20(木) 00:33:32.18ID:z5KAMp1q
>>569
比較って大小関係がわかれば良いの?
それなら(2)から 1 < α < β がわかるから、
α^2 < β^2 < β^3
とするだけだと思うけど
逆にその「一般に想定される解法」は何がしたいのかわからない
0573132人目の素数さん
垢版 |
2020/08/20(木) 00:49:23.46ID:qQP5w1gK
xyz空間の原点Oを頂点、円C:x^2+y^2=r^2(z=H)を底円とする直円錐Vがあり、Vの0≦z≦h(0<h<H)の部分は水で満たされている。
いまC上の1点から、Cの接線方向に質量mの質点を射出すると、質点は円錐の内側面に沿って下降し始めた。
Vと質点の間に摩擦はないものとして、以下の問に答えよ。

(1)質点が円錐の側面上h<z<Hを動いているときの速さ|v|を求めよ。

(2)質点が水中に入った後も、円錐の内側面に張り付いたまま原点まで下降することは可能か。
0574132人目の素数さん
垢版 |
2020/08/20(木) 01:07:36.46ID:z5KAMp1q
>>569
β の具体的な値を使うのが嫌なら、
g(3/2) < 0 より β > 3/2 だから、
β^3 = 2β+1 > 4 > α^2
とかかな
0576132人目の素数さん
垢版 |
2020/08/20(木) 01:29:22.37ID:IC23kD8y
>>567-568
式の整数性よりa≧b
a-b=0のときa^a=1より(a,b)=(1,1)となる
a-b=1のときa^a=2aより(a,b)=(2,1)となる

以下、a-b≧2(自動的にa≧3)のときを考える
a=3のとき(a,b)=(3,1)は不適なのでa≧4となる
さてf(x)=(1+x)^(1/x-1)は対数微分を計算することで
(0<x<1)で単調減少(f(0)=e,f(1)=1)がわかるので
1<(1+b/a)^(a/b-1)<eとなっている
これを使うと
a^a-(a+b)^(a-b)
=a^(a-b)(a^b-((1+b/a)^(a/b-1))^b)
>a^2(a^b-e^b)
≧4a(4^b-e^b)
>4a>a-b
となり、(左辺)>(右辺)より解がないことがわかる
0577132人目の素数さん
垢版 |
2020/08/20(木) 01:37:32.64ID:RgKSKKaD
>>573
>質点は円錐の内側面に沿って下降し始めた。
の理由が不明でどうしようもない。接線方向に射出したら円錐を離れて明後日の方向に飛んでいくはずだろう。
問題文の設定には記述されていない未知の力が働いたり質点の動きになんらかの制約があるとしか思えない。
>Vと質点の間に摩擦はないものとして
働いてない力の情報なんかいらないから、働いている力はすべて記述してくれ。書かれていない条件は勝手に想定しないよ。物理じゃないんだから。

総じて数学の問題としては不備だらけで答えようもない。
0578132人目の素数さん
垢版 |
2020/08/20(木) 03:29:05.84ID:hteSSTrX
(1)
 α = {1 + (19-3√33)^(1/3) + (19+3√33)^(1/3)}/3
 = 1.839287 < 2,

(2)
 β = (1+√5)/2 = 1.618034 (黄金比)
 α > β

(3)
 β^3 = β(β+1)
  = (1/β +1)(β+1)
  > (1+1)^2   (コーシー)
  > α^2,
0580132人目の素数さん
垢版 |
2020/08/20(木) 13:34:50.77ID:z5KAMp1q
>>569
こんなやり方もある
β^2 = β + 1
の両辺を 3 乗すると、
β^6 = β^3 + 3β^2 + 3β + 1
= β^3 + 3β(β+1) + 1
= 4β^3 + 1
となるので、 h(x) = x^2 - 4x - 1 とすると、 β^3 は方程式 h(x) = 0 の解となる。
h(x) = x(x-4) - 1 より明らかに h(0) < 0 かつ h(4) < 0 であるので、
方程式 h(x) = 0 は 2 つの異なる実数解をもち、それらはそれぞれ x < 0, x > 4 の範囲にある。
β > 0 より β^3 > 0 であるから、したがって β^3 > 4
ゆえに β^3 > 4 > α^2
0582132人目の素数さん
垢版 |
2020/08/20(木) 19:23:46.12ID:hteSSTrX
>>569
(1)
f '(x) = 3xx -2x -1 =(x-1)(3x+1)
 f(-1/3) = -22/27 < 0, (極大)
 f(1) = -2 < 0,  (極小)
 f(5/3) = -22/27 < 0,
 f(2) = 1 > 0,
より
 5/3 < α < 2,

(2)
 g(1) = -1 < 0,
 g(5/3) = 1/9 > 0,
より
 1 < β < 5/3,

f(x) = x・g(x) - 1,
(α-β)(α+β-1) = g(α) - g(β) = {f(α)+1}/α - g(β) = 1/α > 0,
∴ α>β
0583132人目の素数さん
垢版 |
2020/08/21(金) 00:20:03.17ID:nVrIq2ZG
a,bをa>b>0の実数の定数とする。
xy平面の楕円C:(x/a)^2+(y/b)^2=1の周上にない点で、以下の条件を満たすもの全体からなる集合Sを考える。

『その点を通り、Cと直交する直線がちょうど4本存在する(ただし直線とCが直交するとは、直線とCの交点において、直線とCの接線が直交することを指す)』

Sが表すxy平面上の領域を求めよ。
0584132人目の素数さん
垢版 |
2020/08/21(金) 02:51:27.74ID:xwxyDQp9
>>560
θ=0 の時の P(=Q) の座標だけは解析解が得られたのでここに記す。

離心率: e=Sqrt[1-(b/a)^2] として
b < a ≦ 2b ⇒
 x = a*(3 - (Sqrt[3]*Sqrt[-1 + 9*e^2 + 4*e^4 - (1 + 4*e^2)^(3/2) + Sqrt[2]*Sqrt[1 + 26*e^4 +176*e^6 + 32*e^8 + (1 + 4*e^2)^(3/2) - 6*e^2*(1 + 4*e^2)^(3/2) - 16*e^4*(1 +4*e^2)^(3/2)]])/e)/(6*e)
 y = b*Sqrt[ 1- (x/a)^2 ]

a > 2b ⇒
 x = a*(3 + (Sqrt[3]*Sqrt[-1 + 9*e^2 + 4*e^4 - (1 + 4*e^2)^(3/2) - Sqrt[2]*Sqrt[1 + 26*e^4 +176*e^6 + 32*e^8 + (1 + 4*e^2)^(3/2) - 6*e^2*(1 + 4*e^2)^(3/2) - 16*e^4*(1 +4*e^2)^(3/2)]])/e)/(6*e)
 y = b*Sqrt[ 1- (x/a)^2 ]

接触円の中心点: H
|h| = a*( Sqrt[1-12e^2 + Sqrt[1+4e^2]^3] )/(Sqrt[8] Sqrt[1-e^2])
 b < a ≦ 2b ⇒ H=(e*a, +|h|)
 a > 2b ⇒ H=(e*a, -|h|)

https://imgur.com/a/RaAJa4r
GeoGebraで描いてみた。もちろん P点の位置決めに Intersect機能は使っていない。
0585563
垢版 |
2020/08/21(金) 03:03:34.65ID:xwxyDQp9
>>584
導出の方針
・楕円: (x/a)^2 + (y/b)^2 = 1
・接触円: (x-e*a/2)^2 + (y-h)^2 = (e*a/2)^2+h^2

(1) 楕円と円の連立式から xについての4次多項式が得られ、その判別式から、h^2 =... を得る。

(2) h^2=...を代入した4次多項式 {LHS} と  重根ありの4次多項式 (x-α)^2 (x^2 + βx + γ) {RHS}
 その係数関係から x = α を導く。 LHSの3次項が消えるようにズラすと簡単になる。
 表式中の符号選択に迷う箇所が2つあり、そこはグラフを動かして発見的に求めた。解の連続性を考えれば問題はないはず。

場合分けの境界点
・(x/a)^2 + (y/b)^2 = 1
・(x-e*a/2)^2 + y^2 = (e*a/2)^2
 → (1-(b/a)^2) x^2 - e*a*x + b^2 = 0
判別式より a=2b
hの表式より 1-12e^2 + Sqrt[1+4e^2]^3 = 0 となる。ここから逆算しても求められる。

境界での P座標は
  (x,y) = a * ( 1/Sqrt[3], 1/Sqrt[6] )
とシンプルになる。
0586132人目の素数さん
垢版 |
2020/08/21(金) 17:35:52.08ID:1oj4sGG5
D = {z ∈ C | |z - 1| < cosα, |Arg(1 - z)| < α}とする。z ∈ D ⇒ |z| < 1を証明せよ。
0587132人目の素数さん
垢版 |
2020/08/21(金) 18:28:16.63ID:GUw9f3Wz
f(x)=lim[n→∞] (x-a_1)(x-a_2)(x-a_3)....(x-a_n)  (a_iは実数)
が有界であるための数列{a_i}の必要十分条件はどうなるのでしょうか?
0590132人目の素数さん
垢版 |
2020/08/21(金) 19:22:49.31ID:eKSCCB4p
>>586
題意により
 z∈D ⇒
 (1-x) = |1-z| cos(Arg(1-z)) > |1-z| cosα > |1-z|^2, ・・・・ (*)
また
 |1-z|^2 = (1-x)^2 + y^2,
だから
 |z|^2 + |1-z|^2 = x^2 + (1-x)^2 + 2y^2
  = 1 - 2(1-x) + 2|1-z|^2
  < 1,  (← *)
0591132人目の素数さん
垢版 |
2020/08/21(金) 19:30:12.93ID:eKSCCB4p
違った
Π[n=1,∞] (1 + xx/(nπ)^2) = sinh(x)/x となるらしい。

(オイラの無限乗積表示)
0593132人目の素数さん
垢版 |
2020/08/21(金) 20:32:08.64ID:eKSCCB4p
|z|^2 + |1-z|^2 = 2|1/2 - z|^2 + 1/2,
より
|z|^2 + |1-z|^2 < 1  ⇔  |1/2 - z| < 1/2,
0594132人目の素数さん
垢版 |
2020/08/21(金) 20:49:44.05ID:Ate3H8P0
周の長さが2の△ABCにおいて、BC=2aとおく。

(1)実数aの値が変化するとき、その取りうる値の範囲を不等式で表せ。

(2)△ABCの内角∠Aの二等分線とBCとの交点をDとし、Aを通りADと直交する直線をlとする。またBを端点とする半直線BAの、Aに関してBと反対側に点Eをとる。このとき直線lが∠EACを2等分することを示せ。
0595132人目の素数さん
垢版 |
2020/08/21(金) 20:50:19.89ID:18CWivAF
>>587
簡単な条件に書くのは無理なんじゃないかな
有界数列は有界数列としかいえないのと同じで、強いて言うなら数列
P[n] := Π[k=1,n] (x-a_k)
が有界ということだが、これはただの言い換えでしかない
収束ならコーシーの条件があるけど、有界だけだとどうにも
例えば、 a_{2k} = x + 1/2, a_{2k+1} = x + 2 とすると、
P[n] は有界だが収束しない
一方、 a_k = x - (1 + 1/k) とすると、 a_k → x-1 (k→∞) だが、
lim[n→∞] P[n] = Π[n=1,∞] (1 + 1/n) = ∞
0596132人目の素数さん
垢版 |
2020/08/21(金) 21:06:25.94ID:8HBHIz0K
>>586
Dが空ならok
Dが非空なら0<cosαより-π/2<α<π/2
zは1を中心とする半径cosαの円内にあり
かつ中心1からみた偏角θはπ-α<θ<π+α
この領域は0,1,cosβe^(±iβ)を頂点とする菱形
の内部にあり(β=π/2-αとおいた)
よって領域は単位円内に存在する
0597イナ ◆/7jUdUKiSM
垢版 |
2020/08/21(金) 21:58:26.20ID:Ly2a2ymO
>>447
>>594
(1)2-2a>2a
2>4a
a<1/2
また2a>0
よって0<a<1/2
(2)角BAC+角EAC=180°
辺々を2で割ると角BAC/2+角EAC/2=90°
角CAD+角EACのC側の半分=90°
すなわち角CADを二分する直線と角EACを二分する直線はたがいに直交する。
∴示された。
0598132人目の素数さん
垢版 |
2020/08/21(金) 22:04:37.43ID:8HBHIz0K
f(x)が有界になるxの範囲が非空なためのa_iの条件
さらにそのようなxが一点でない場合の条件
とかなら考えても面白いかもね

xの範囲が一点でない場合、f(x)はその範囲のほとんどの点で0になりそうだけど、どうだろう
0599132人目の素数さん
垢版 |
2020/08/21(金) 22:53:02.29ID:1AD6M5we
xy平面をxy平面上のn本の異なる直線で分割してできる領域の数を考える。なおここで言う領域には、閉領域・開領域のいずれも含まれる。

(1)このようにしてできる領域の数の最大値をnで表せ。

(2)このようにしてできる領域の数は、n+1から(1)で求めた最大値まで複数通りある。それは何通りであるか、nで表せ。
0601132人目の素数さん
垢版 |
2020/08/21(金) 23:25:45.79ID:18CWivAF
>>598
>xの範囲が一点でない場合、f(x)はその範囲のほとんどの点で0になりそう

a_n が x に依存しない数列で、 f(x) が収束するならそうだろうね
無限積が 0 でない値に収束するなら項の数列は 1 に収束しなければならないから、
x-a_n → 1 (n→∞) なら、任意の実数 α に対して、x+α-a_n → α+1 (n→∞)
となるので、 f(x) が収束して ≠ 0 なら α = 0 でなければならない
0602132人目の素数さん
垢版 |
2020/08/21(金) 23:29:45.59ID:5qiPpY9M
異なるω_0, ..., ω_nに対してn + 1正方行列

(ω_i^j)_i,j

の行列式
0604132人目の素数さん
垢版 |
2020/08/22(土) 02:14:30.37ID:ymtf6p4s
>>586 を 改良・・・・
D = {z ∈ C | |z - 1| < cosα, |Arg(1 - z)| < α}とする。
z ∈ D ⇒ |z-1/2| < 1/2 を証明せよ。
0606132人目の素数さん
垢版 |
2020/08/22(土) 03:10:26.57ID:ymtf6p4s
>>599 (2)
n本の直線を、平行性によって分類する。
各類に含まれる直線の本数を m_i (i=1,…,k) とする。
 m_1 + m_2 + ・・・・ + m_k = n,
このときできる領域の総数は
 1 + n(n+1)/2 - Σ[i=1,k] m_i(m_1 - 1)/2,

ところで、
nをいくつかの自然数の和で表わす方法の数は分割数 p(n)
(2) の答え ≦ p(n)
n≦11 では p(n) ≦ (nn-n+2)/2 なので
 p(n) とおり以下のはず・・・・
0607132人目の素数さん
垢版 |
2020/08/22(土) 11:20:04.76ID:QJgAg0uf
n+1
Σ 1/i
i=3

よろしくお願いします。解答が間違っていたので。
0608132人目の素数さん
垢版 |
2020/08/22(土) 11:23:32.71ID:QJgAg0uf
各項を書き並べて書け。という問題です
0610132人目の素数さん
垢版 |
2020/08/22(土) 13:31:03.10ID:QJgAg0uf
1/3+1/4....+

の最後はどう書けばいいでしょうか?

nまでだったら
...+1/(n+2)

だと思うのですが、n+1までなので

1/3+1/4...+1/(n+2)+1(n+3)

となるのでしょうか
0612132人目の素数さん
垢版 |
2020/08/22(土) 13:39:08.47ID:hYx9dCgX
>>610
n+1
Σ 1/i
i=3

この記号は、1/iのiに3からn+1まで代入して合計するという意味です。
なので、1/3 + 1/4 + … + 1/(n+1)
が正解です。
0613132人目の素数さん
垢版 |
2020/08/22(土) 13:49:37.55ID:I9MUO/8V
1/iのiにn+1を代入したら1/(n+1)だろ

n+1
Σ
i=3
ってのは、i=3から始まる(n+1)項の和って意味じゃないぞ
i=3から始まってi=n+1までの和って意味だ
0615132人目の素数さん
垢版 |
2020/08/22(土) 13:53:29.67ID:tMe/q/9E
0189 132人目の素数さん 2020/08/22 12:05:05
n+1
Σ 1/i
i=3

各項を書き並べて書け、という問題。
解答がなかったのでお願いします
ID:QJgAg0uf

0190 132人目の素数さん 2020/08/22 12:36:01
マルチ
ID:TylVx0O5
0616132人目の素数さん
垢版 |
2020/08/22(土) 13:53:34.67ID:5Q5lMGyj
サイコロ6個振って目の合計が26を超える確率をマルコフ不等式とチェビシェフ不等式を使って評価せよ

マルコフ不等式が21/26以下だと思うのですが、チェビシェフ不等式がわかりません
よろしくお願いします
0617132人目の素数さん
垢版 |
2020/08/22(土) 13:56:04.99ID:QJgAg0uf
なるほど、基本が欠如していました。ありがとうございます
0618イナ ◆/7jUdUKiSM
垢版 |
2020/08/22(土) 15:59:59.40ID:uH5uOOQL
>>597
>>616
1+1+6+6+6+6=24超えない。
出目の数の合計が36
6回とも6が出る確率は1/6^6=1/46656
出目の数の合計が35
1回だけ5が出てあとは6の確率は5が何回目に出るか6通りあるから(1/6^6)×6=1/7776
出目の数の合計が34
5が2回出てあとは6の確率は5が何回目と何回目に出るか15通りあるから(1/6^6)×15=15/46656=5/15552
1回だけ4が出てあとは6の確率は4が何回目に出るか6通りあるから(1/6^6)×6=1/7776
あわせて5/15552+1/7776=7/15552
出目の数の合計が33
つづく……
0619132人目の素数さん
垢版 |
2020/08/22(土) 17:29:54.92ID:U0iCzex9
a^b+b^c=c^a
を満たす自然数は無数に存在するか?

という問題が解決できません。フェルマー予想を見て思いつきました。
解の1つとして(a,b,c)=(1,1,2)は分かりました。
x<yでは殆どの場合y^x<x^yなので、それを利用して、ごく一部の例外を除きmin(a,b,c)=cであることを示す→cの値を絞り込むという流れを考えたのですが、私の力では絞り込みがうまくできませんでした。
ご教授いただけますと幸いです。よろしくお願いします。
0620132人目の素数さん
垢版 |
2020/08/22(土) 18:36:16.88ID:SGuwN39p
>>616
1000万回シミュレーションした結果

> # サイコロ6個振って目の合計が26を超える確率
> sim <- function() sum(sample(6,6,replace = TRUE)) > 26
> mean(replicate(1e7,sim()))
[1] 0.0963488
0621イナ ◆/7jUdUKiSM
垢版 |
2020/08/22(土) 19:29:22.63ID:uH5uOOQL
>>618つづき。
>>616
出目の数の合計が33
3回5であとは6の確率は5の出方によって20通りあるから(1/6^6)×20=5/11664
4と5が1回ずつであとは6の確率は(1/6^6)×30=5/7776
1回だけ3であとは6の確率は(1/6^6)×6=1/7776
あわせると5/11664+5/7776+1/7776=1/11664+1/1296
=(1+9)/11664
=10/11664
=5/5832
出目の数の合計が32
4回5であとは6の確率は5が2回であとは6の確率と同じで(1/6^6)×15=5/15552
1回だけ4で5が2回であとは6の確率は、
1回目に4が出て5が何回目と何回目に出るか10通りあって4が何回目に出るか6通りあるから10×6=60通りあって(1/6^6)×60=10/6^5=10/7776=5/3888
4が2回であとは6の確率は5が2回であとは6の確率と同じで(1/6^6)×15=5/15552
3と5が1回ずつであとは6の確率は4と5が1回ずつであとは6の確率と同じで(1/6^6)×30=5/7776
1回だけ2であとは6の確率は(1/6^6)×6=1/7776
あわせると5/15552+5/3888+5/15552+5/7776+1/7776=(5+20+5+10+2)/15552
=42/15552
=21/7776
=7/2592
出目の数の合計が31
5回5で1回だけ6の確率は6/6^6=1/6^5=1/7776
1回だけ4で3回5であとは6の確率は1回だけ4で5が2回であとは6の確率と同じで5/3888
2回4で1回だけ5であとは6の確率も同じで5/3888
3と4が1回ずつであとは6の確率は何回も出てきてるのと同じで5/7776
2と5が1回ずつであとは6の確率も5/7776
1回だけ1であとは6の確率は1/7776
あわせると1/7776+5/3888+5/3888+5/7776+5/7776+1/7776=12/7776+10/3888
=(3+5)/1944
=1/243
出目の数の合計が30
出目の数の合計が29
出目の数の合計が28
出目の数の合計が27
すべて足すと出目の数の合計が26を超える確率は、
0622イナ ◆/7jUdUKiSM
垢版 |
2020/08/22(土) 21:56:37.44ID:uH5uOOQL
>>621つづき。
>>616
出目の数の合計が30
1+5+6+6+6+6=30確率5/7776
2+4+6+6+6+6=30確率5/7776
3+3+6+6+6+6=30確率5/15552
2+5+5+6+6+6=30確率5/3888
3+4+5+6+6+6=30確率(1/6^6)×6×5×4=5/1944
4+4+4+6+6+6=30確率5/11664
3+5+5+5+6+6=30確率5/3888
4+4+5+5+6+6=30確率(1/6^6)×
4+5+5+5+5+6=30確率5/15552
5+5+5+5+5+5=30確率1/46656
出目の数の合計が29
出目の数の合計が28
出目の数の合計が27
0623132人目の素数さん
垢版 |
2020/08/22(土) 23:47:46.59ID:kkVGJmRy
>>620
どうせPCに頼るなら 6^6 (= 46656) 通りの総チェックしたらええがな。
> 1000万回シミュレーション
そんだけやっておいて 0.096までしか合ってないってマヌケにもほどがある。
0625132人目の素数さん
垢版 |
2020/08/23(日) 07:43:14.97ID:qhSoFq1l
>>606
例) nが小さいとき

n, (nn-n+2)/2, p(n), q(n)=#M, M
------------------------------------------------
1, 1, 1, 1, {2}
2, 2, 2, 2, {3,4}
3, 4, 3, 3, {4,6,7}
4, 7, 5, 5, {5,8,9,10,11}
5, 11, 7, 7, {6,10,12,13,14,15,16}
6, 16, 11, 9, {7,12,15,16,18,19,20,21,22}
7, 22, 15, 13, {8,14,18,19,20,22,23,24,25,26,27,28,29}
8, 29, 22, 18, {9,16,21,22,24,25,26,27,28,29,30,31,32,33,34,35,36,37}
9, 37, 30, 21, {10,18,24,25,28,30,31,33,34,35,36,37,38,39,40,41,42,43,44,45,46}
10, 46, 42, 27, {11,20,27,28, ・・・・, 56}
---------------------------------------------
{q(n)} = {1,2,3,5,7,9,13,18,21,27,・・・・}
http://oeis.org/A069999


Mの最小元は n+1, 最大元は (nn+n+2)/2 だが 隙間や重複がある。
 とくに類数kが小さいところでは隙間が多い。
 (n) → n+1
 (n-1,1) → 2n,
 (n-2,2) → 3n-3,
 (n-2,1,1) → 3n-2,
 
 (2,1,・・・,1) → (nn+n)/2,
 (1,1,・・・・,1) → (nn+n+2)/2,
0627132人目の素数さん
垢版 |
2020/08/23(日) 08:40:25.72ID:qhSoFq1l
>>616
出目の合計, 確率
------------------------------
36, 1/6^6,  >618
35, 6/6^6,  >618
34, 21/6^6,  >618
33, 56/6^6,  >621
32, 126/6^6,  >621
31, 252/6^6,  >621(*)
30, 456/6^6,  >622(**)
29, 756/6^6,
28, 1161^6^6,
27, 1666/6^6,
--------------------------------
計, (1666+1161+756+456+252+126+56+21+6+1)/6^6
 = 4501/6^6 = 0.09647205

(*) (355666) が抜けてます。
(**) (455556) の確率が半分です。
0628132人目の素数さん
垢版 |
2020/08/23(日) 09:37:08.49ID:prx/bVbe
ある製品の寿命について、その累積分布関数は以下の通り与えられている。
F(x)=λexp(-λx) (0≦x)

(1)確率密度関数f(x)を求めよ。

(2)期待値、中央値、第一四分位数をそれぞれ求めよ。
0629132人目の素数さん
垢版 |
2020/08/23(日) 09:44:31.28ID:nKOizBja
>>623
111111〜666666を0から6^6−1を6進数で表示して
各桁の和が26−6より大きくなるようにプログラムして総当たりしてみた。

dec2n <- function(num, N, digit = 6){
r=num%%N
q=num%/%N
while(q > 0 | digit > 1){
r=append(q%%N,r)
q=q%/%N
digit=digit-1
}
return(r)
}
pm=sapply(0:(6^6-1),function(x) dec2n(x,6))
f<-function(x) sum(x) > 26-6
s=sum(apply(pm,2,f))
S=ncol(pm)
cat(s,'/',S,'\n')
s/S


> cat(s,'/',S,'\n')
4501 / 46656
> s/S
[1] 0.09647205
0632132人目の素数さん
垢版 |
2020/08/23(日) 09:58:27.42ID:nKOizBja
ソフトに微分してもらったら、>630でいいみたいだな。

> D(expression(λ*exp(-λ*x)),'x')
-(λ * (exp(-λ * x) * λ))
0635132人目の素数さん
垢版 |
2020/08/23(日) 10:26:05.70ID:jONCOSgj
>>627
数を数えるのもプログラムにさせてみた。

table(x)
x
6 7 8 9 10 11 12 13 14 15 16 17 18 19 20 21
1 6 21 56 126 252 456 756 1161 1666 2247 2856 3431 3906 4221 4332
22 23 24 25 26 27 28 29 30 31 32 33 34 35 36
4221 3906 3431 2856 2247 1666 1161 756 456 252 126 56 21 6 1
0640イナ ◆/7jUdUKiSM
垢版 |
2020/08/23(日) 12:03:47.05ID:Q5A4PXq6
>>622
>>626
出目の合計31は漏れがあるかな、とは思った。
30は、4,4,5,5,6,6がわからなくなった。
26超えは1割弱、そんなもんだろう。
0641132人目の素数さん
垢版 |
2020/08/23(日) 14:04:07.26ID:Rpa/qv/i
集合Xがただ一つの要素だけからなるというのを正確に言うと、「x∈Xとなるようなxが存在する。x≠y⇒y∈Xでない」ということですか?

|X| = 1の定義は、Xから{x∈Z_{+} | x < 1 + 1}への全単射が存在するということだと思います。
1∈{x∈Z_{+} | x < 1 + 1}であり、x∈Z_{+}かつ1 < x < 1 + 1となるxは存在しないため、x≠1⇒「x∈{x∈Z_{+} | x < 1 + 1}でない」が成り立ちます。
{x∈Z_{+} | x < 1 + 1}からXへの全単射が存在するため、1→x∈Xとなるxが存在します。x≠yかつy∈Xとなるようなyが存在すると仮定すると、
n→yとなるようなn∈{x∈Z_{+} | x < 1 + 1}が存在することになります。n = 1でなければなりません。これは→が写像であることに反します。
よって、「x≠y⇒y∈Xでない」が成り立ちます。
0642132人目の素数さん
垢版 |
2020/08/23(日) 15:10:25.40ID:nKOizBja
>>639
定義に従って計算するだけ

微分して、確率密度関数  exp(-λ * x) * λ

確率密度関数*xを定義域で積分して期待値 1/λ

累積密度関数の逆関数を作って

中央値:log2/λ

第一四分位数:log(4/3)/λ
0643132人目の素数さん
垢版 |
2020/08/23(日) 21:05:51.88ID:nKOizBja
10個のサイコロを振って目の合計を当てる賭けをするときいくつにかけるのが最も有利か。その確率はいくらか。

目の合計の期待値は35というのはわかるけど、それが最頻値だというのはどうやれば確認できるだろう?
0644132人目の素数さん
垢版 |
2020/08/23(日) 22:06:38.36ID:nKOizBja
>>643
分布が左右対称なら期待値(平均値)と最頻値は一致するといえるか?という問題になるのかなぁ?
0645132人目の素数さん
垢版 |
2020/08/23(日) 22:10:18.01ID:nKOizBja
>>643
10個にサイコロを投げて目の合計が35になる確率のシミュレーション解
> sim <- function(n=35) sum(sample(6,10,rep=T))==n
> mean(replicate(1e7,sim()))
[1] 0.0726302

厳密解はイナ大先生が怒涛の計算力で数えてくれると思う。
0647132人目の素数さん
垢版 |
2020/08/23(日) 23:46:07.66ID:qhSoFq1l
p(35) = 4395456/(6^10) = 0.0726928

p(n) ≒ p(μ)・e^{-(n-μ)^2 /(2σ^2)},

μ=35, σ^2 =31.09
0648132人目の素数さん
垢版 |
2020/08/24(月) 00:08:08.58ID:CGsqsz2y
数値が一致したな。
data.frame(sum=10:60,pips=pips[10:60])
sum pips
1 10 1
2 11 10
3 12 55
4 13 220
5 14 715
6 15 2002
7 16 4995
8 17 11340
9 18 23760
10 19 46420
11 20 85228
12 21 147940
13 22 243925
14 23 383470
15 24 576565
16 25 831204
17 26 1151370
18 27 1535040
19 28 1972630
20 29 2446300
21 30 2930455
22 31 3393610
23 32 3801535
24 33 4121260
25 34 4325310
26 35 4395456
0649132人目の素数さん
垢版 |
2020/08/24(月) 00:20:41.38ID:CGsqsz2y
>>647
σ^2 =31.09はどういう計算ででてくるのでしょうか?
μの方は10*(1+2+3+4+5+6)/6=35とわかりますが。
0650132人目の素数さん
垢版 |
2020/08/24(月) 00:25:59.76ID:mTnDMiIJ
投げる回数の帰納法使えば密度関数の一峰性示せるけどな
それと平均での対称性で示せる
一発証明ないものか
0652132人目の素数さん
垢版 |
2020/08/24(月) 00:58:46.24ID:x7ua1b+N
>>650
サイをn回振って合計がmになる確率を p(n,m) とすると、
p(1,m) = if m ∈ {1,2,3,4,5,6} then 1/6 else 0 として、
p(n+1,m) = (1/6) Σ[k=1..6] p(n,m-k) なんだから、
p(n+1,m+1) = p(n+1,m) + (1/6)(p(n,m) - p(n,m-6))
つまり p(n,m) と p(n,m-6) との大小関係がそのまま p(n+1,m+1) と p(n+1,m) との大小関係になる
このことを利用したら、数学的帰納法で n が偶数のとき m = 7n/2 で p(n,m) が最大になることが言えるのでは
0655132人目の素数さん
垢版 |
2020/08/24(月) 02:07:39.12ID:r3SqnUKm
∫(sinx/x)^2dx (0→∞)

∫(lnx/x^2+1)^2dx (0→∞)
0656132人目の素数さん
垢版 |
2020/08/24(月) 02:13:28.02ID:HWPbccmb
>>637
一様分布
 p(n) = 1/6   (1≦n≦6)
   = 0    (n<1, n>6)
6個をたたみ込んだもの。

 P(n) = C(n-1,5) = (n-1)(n-2)(n-3)(n-4)(n-5)/5!,   (6≦n≦11)

 P(n) = 2771 - (18929/12)n + (2847/8)n^2 - (949/24)n^3 + (17/8)n^4 - (1/24)n^5,
   (11≦n≦16)

 P(n) = -90049 + (177851/6)n - (46017/12)n^2 + (2893/12)n^3 - (29/4)n^4 + (1/12)n^5
   (16≦n≦21)

 P(n) = 582931 - (787943/6)n + (46191/4)n^2 - (5917/12)n^3 + (41/4)n^4 - (1/12)n^5
   (21≦n≦26)

 P(n) = -1198394 + (2393525/12)n - (104043/8)n^2 + (10021/24)n^3 - (53/8)n^4 + (1/24)n^5,
   (26≦n≦31)

 P(n) = C(41-n,5) = (41-n)(40-n)(39-n)(38-n)(37-n)/5!,   (31≦n≦36)

 P(n) = 0,   (n<6, n>36)
0657132人目の素数さん
垢版 |
2020/08/24(月) 02:54:12.63ID:HWPbccmb
>>655
(上) は部分積分して
∫(sin(x)/x)^2 dx = -(sin(x)^2)/x + ∫sin(2x)/x dx
      = -(sin(x)^2)/x + Si(2x),
これより
∫[0,∞] (sin(x)/x)^2 dx = Si(∞) - Si(0) = π/2,

あるいは
 ∫[0,∞] {cos(ax) - cos(bx)}/x^2 = (π/2)(b-a),
を使う。
高木:「解析概論」改訂第三版、岩波書店 (1961)
 第5章 練習問題 (4), p.264


(下) は発散しそう・・・・
0658132人目の素数さん
垢版 |
2020/08/24(月) 05:33:08.87ID:CGsqsz2y
>>656
ありがとうございました。
場合分けから随分と複雑な式になることだけは理解できました。
0660イナ ◆/7jUdUKiSM
垢版 |
2020/08/24(月) 10:06:11.51ID:67dg6YFo
>>640
>>643
出目の合計が10になる確率と60になる確率は、
1/6^10
出目の合計が11になる確率は2が10通りで、
59になる確率は5が10通りで10/6^10=5/3×6^9
出目の合計が12に……
出目の合計が35になる確率は1割にも満たないんじゃないか?
0662132人目の素数さん
垢版 |
2020/08/24(月) 16:57:34.94ID:p3oV4QfX
初投稿失
これはコピー用紙の規格で、長い方の辺を半分にすると、もとの長方形と、たてと横の比が同じ長方形が得られるという性質があるっていう問題だ。
辺BCと折り目BEが重なることから、AB:BCを求める。
また、このコピー用紙を長い方のへんで半分にしたとき、もとの長方形と縦と横の比が同じになることを、式を立てて確かめる。
コピー用紙の画像https://dotup.org/uploda/dotup.org2237290.png.html
問題のわかりやすい画像https://dotup.org/uploda/dotup.org2237293.png.html
わかりにくいと思うがよろしくおねがいします
あと、問題の解き方もよろしく
0666132人目の素数さん
垢版 |
2020/08/24(月) 17:28:57.86ID:p3oV4QfX
俺数学全くといっていいほどできないんだ
しつもんしてごめんな
0667132人目の素数さん
垢版 |
2020/08/24(月) 17:37:55.43ID:+Fh3sbhA
うむ
勉強する前に問題を解こうとするのは未解決問題に挑むってことだからな
常人には無茶な行い
0668132人目の素数さん
垢版 |
2020/08/24(月) 17:40:03.29ID:p3oV4QfX
ありがとう
ちょっと教科書見直してきて考えてみるわ
0671132人目の素数さん
垢版 |
2020/08/24(月) 19:31:29.13ID:F6f8Pdh5
ワークの問題なのですがいまいち解き方がピンときません…
よろしくお願いいたします。

次の関数に最大値、最小値があればそれを求めよ。
y=(x^2-2x)^2+4(x^2-2x)+5
0672132人目の素数さん
垢版 |
2020/08/24(月) 20:19:22.31ID:HWPbccmb
 u = (x^2 -2x) +1,
とおくと
 u = (x-1)^2 ≧ 0,
y = (u-1)^2 + 4(u-1) +5
 = uu + 2u +2  (u≧0 で単調増加)
 ≧ 2, ・・・・ 最小値 (@ u=0, x=1)

x→±∞ のとき y→∞
 最大値なし。
0673イナ ◆/7jUdUKiSM
垢版 |
2020/08/24(月) 20:47:49.01ID:67dg6YFo
>>622
>>671
y=(x^2-2x)^2+4(x^2-2x)+5
y'=2(x^2-2x)(2x-2)+4(2x-2)
=4(x-1)(x^2-2x+2)
x^2-2x+2=(x-1)^2+1>0だから、
y'=0となるのはx=1のみ。
x→±∞のときy→+∞
x=1のときx^2-2x=1-2=-1
最小値y=(-1)^2+4(-1)+5=1-4+5=2
こうかなぁ?
0674132人目の素数さん
垢版 |
2020/08/24(月) 21:04:54.65ID:F6f8Pdh5
>>672
ありがとうございます

u = (x^2 -2x) +1とおくとの
+1の部分ってどこからでた数字でしょうか
0675132人目の素数さん
垢版 |
2020/08/24(月) 21:38:36.09ID:F6f8Pdh5
>>673
ありがとうございます
1行目にあった5が消えて2行目のy'=の後の2がでてきたのは
与式を展開して(2x-2)で整理したということでしょうか
アホ過ぎてすみません…
0676132人目の素数さん
垢版 |
2020/08/24(月) 21:42:32.65ID:2Nnha+ZH
ある本に、関数f : N -> Nを
f(0) = 1
f(1) = 1
f(n) = f(n-1) + f(n-2) (n ≧ 2)
と定義するというのはおかしいと書いてあります。その理由は、これから定義しようという関数f : N -> Nを使ってfの値を定義しようとしているというものです。
そのかわり、関数f : N -> Nで
f(0) = 1
f(1) = 1
f(n) = f(n-1) + f(n-2) (n ≧ 2)
を満たすものが存在することを証明しています。

なんだか分かったような分からないような気がするのですが、上に書いたことは正しいのでしょうか?
0677132人目の素数さん
垢版 |
2020/08/24(月) 22:01:54.25ID:7o179h52
1以上22以下の自然数の集合をSとする
Sの部分集合Tで、次の条件を満たすものを考える

[条件] Tに属する任意の2つの要素の差は4でも7でもない

Tの要素数の最大値はいくらか

1 5 9 13 17 21
2 6 10 14 18 22
3 7 11 15 19
4 8 12 16 20
0678132人目の素数さん
垢版 |
2020/08/24(月) 22:02:58.76ID:ulqLR7TI
>>676
この程度ならどっちでも
もっと複雑だと後者のがよいことあるかもな
0679132人目の素数さん
垢版 |
2020/08/24(月) 22:07:57.98ID:z0JBrlOX
前者は「そもそもこのようなfが存在するかどうかわからないから、存在することを示す必要がある」なら納得できるし実際後者がこの立場なんだけど、この文言ではちょっと混乱しそう
f(n)の値を定義するのに使ってるのはfそのもの(全てのnについて値f(n)の定まった対象)ではなく、そのnまでの値f(n-1),f(n-2)しか使っておらず、循環定義には該当しない
0680132人目の素数さん
垢版 |
2020/08/24(月) 22:09:12.82ID:3IiFO/88
>>676
そのような場合の「…と定義する」は「…を満たすものが一意的に存在するから、それを…と定義する」という意味だよ
一意的に存在するのが明らかな場合によく(無意識に?)使う
0681132人目の素数さん
垢版 |
2020/08/24(月) 22:40:51.35ID:QaYdhkfZ
>>671
まず、 y は x^4 の係数が正の 4 次関数だから、最大値が存在しないことはすぐにわかる
同様に最小値が存在することもわかる

次に、 y のグラフの形を調べるために y の導関数 dy/dx を計算する
u(x) = x^2 - 2x
と置くと、 y = f(u) と書けるから、合成関数の微分を使うと dy/dx の計算が楽になる
実際に計算すると、x についての 2 次方程式 dy/du = 0 は x について実数解をもたないので、
全ての実数 x に対して dy/du > 0 となることがわかる
したがって du/dx の符号によってグラフの形が決まり、 y の最小値は極小値でもあることがわかる
0682132人目の素数さん
垢版 |
2020/08/24(月) 22:54:59.66ID:QaYdhkfZ
>>676
再帰的定義の話かな
確かに厳密に言えばそのような関数 f が存在することは明らかではないから、
証明が必要だろうね
この場合は数学的帰納法で f(n) の値が一意に定まることを示せば十分だと思うけど
0684132人目の素数さん
垢版 |
2020/08/24(月) 23:23:19.73ID:QaYdhkfZ
>>683
n = 1, 2, 3, … に対し、
a[n] = b[n] + A(n-1)(n-2)(n-3)(n-4)(n-5)(n-6)(n-7)(n-8)
( A は任意の複素数)
b[n] = 0, 1, 4, 9, 20, 40, 78, 147, B, B, …
( B は任意の複素数)
としたときの a[9]
0686132人目の素数さん
垢版 |
2020/08/24(月) 23:33:50.68ID:wnScEK33
そうなるんですね。自分は解けませんでした。
例えば、回答するのは小学生として、この数列の○を答えにする文章題は作れますでしょうか?
スレ違いなら申し訳ありません。
0688 【末吉】
垢版 |
2020/08/25(火) 00:10:17.94ID:A/VRpGsl
>>673
>>675
y=(x^2-2x)^2+4(x^2-2x)+5
y'=2(x^2-2x)(2x-2)+4(2x-2)
なぜ5が消えたのか→微分したら定数項は消えるだろう。5じゃなくて3でも7でも-1/2でも消えるはず。なんでか。xと掛けあってへんで変化の割合に影響がないでやない? 知らんけど。
x^2-2xそのものを微分した2x-2を掛けておくんやったと思う。なんでか? なんでかは知らんことないけど。
なぜ2が出てきたか→2乗を微分したでやない?
2xを微分したら2やん。
0689132人目の素数さん
垢版 |
2020/08/25(火) 00:25:00.42ID:BYmXedij
>>671
もしかして微分を知らないまたは使えないのか
それなら u = x^2 - 2x と置いて y を u について平方完成して、
さらに u を x について平方完成すれば良いんじゃね
0690132人目の素数さん
垢版 |
2020/08/25(火) 00:40:12.56ID:7va++JjE
>>682
>この場合は数学的帰納法で f(n) の値が一意に定まることを示せば十分だと思うけど
この場合は帰納的定義だから証明するまでも無いよ
0691132人目の素数さん
垢版 |
2020/08/25(火) 00:44:46.86ID:BYmXedij
>>686
例えばこんなのはどうだろう

0 番目の数 = 0
1 番目の数 = 1
2 番目の数 = 4
として、
3 番目の数 = 3×((1 番目の数)÷1 + (2 番目の数)÷2)
4 番目の数 = 4×((2 番目の数)÷2 + (3 番目の数)÷3)
5 番目の数 = 5×((3 番目の数)÷3 + (4 番目の数)÷4)

とするとき、 8 番目の数を求めよ。
0692132人目の素数さん
垢版 |
2020/08/25(火) 00:50:07.73ID:BYmXedij
>>690
それだと>>676が言っている本のスタンスに合わないでしょ
「帰納的定義より明らか」が使えるなら>>676みたいなことは書かないはず
0693132人目の素数さん
垢版 |
2020/08/25(火) 01:13:07.74ID:7va++JjE
>>692
どうせその本でやってることって大したことじゃ無く
帰納的定期だから帰納的に決められる
程度だろうよ
もっと本質的に必要な場面はあろうが
フィボナッチ数列の定義程度では萎えさせるだけ
0694132人目の素数さん
垢版 |
2020/08/25(火) 02:08:50.48ID:BYmXedij
>>693
さすがにそんなトートロジーを書く数学の本はないと思うが
フィボナッチ数列の定義は導入か何かで、その後に一般に帰納的定義がwell-definedであることを示すとか、
逆にwell-definedでない定義の例を示すとか、そういう流れじゃないの?
>>676の本が何の本で、どんな流れで書いてあったのかわからないとわからないが
0695132人目の素数さん
垢版 |
2020/08/25(火) 02:19:26.21ID:qIfbhl+s
x''+2x=tcostの解を求めてく段階で
(1/(D^2+1))te^(it)の計算がわからないです
0696132人目の素数さん
垢版 |
2020/08/25(火) 02:31:36.99ID:BYmXedij
>>695
ヘヴィサイドの演算子法?
(1/(D+a))f(x) = e^(-ax) ∫ f(x)e^(ax) dx

D^2 + 1 = (D+i)(D-i)
を使うだけじゃないの?
0697132人目の素数さん
垢版 |
2020/08/25(火) 02:32:15.46ID:N/HR40kY
>>689
今高1なのでまだ微分は習ってないんです
すみません…
平方完成の方法でやってみます!
ありがとうございます
0698132人目の素数さん
垢版 |
2020/08/25(火) 02:38:52.84ID:qIfbhl+s
>>696
すみません。D^2+1じゃなくてD^2+2でした
√2が出てきてよくわかんなくなってしまいました
0700132人目の素数さん
垢版 |
2020/08/25(火) 03:29:11.78ID:qIfbhl+s
>>699
できました。ありがとうございました
0701132人目の素数さん
垢版 |
2020/08/25(火) 05:19:08.15ID:RvbSTotP
D = d/dt,

{1/(D+i√2)} t e^(it) = (√2 -1)(-it + √2 -1)e^(it),
{1/(D-i√2)} t e^(it) = (√2 +1)(it + √2 +1)e^(it),
{1/(D+i√2)} t e^(-it) = (√2 +1)(-it + √2 +1)e^(-it),
{1/(D-i√2)} t e^(-it) = (√2 -1)(it + √2 -1)e^(-it),

{1/(DD+2)} t e^(it) = (t-2i) e^(it),
{1/(DD+2)} t e^(-it) = (t+2i) e^(-it),

{1/(DD+2)} t cos(t) = t cos(t) + 2 sin(t),
これが強制振動部分。
これと自由振動(固有振動)を足して
 y(t) = t cos(t) + 2 sin(t) + c1 cos(t√2) + c2 sin(t√2),
0702132人目の素数さん
垢版 |
2020/08/25(火) 05:30:06.20ID:7jaUCBry
>>691
わかりやすいです。
ただイメージとしてはもっと文章的にしたいので、参考に工夫してみます。
ありがとうございました。
0705132人目の素数さん
垢版 |
2020/08/25(火) 08:19:44.74ID:WzhAMzCC
>>677
10個が最大だな

具体的に1,3,6,9,11,12,14,17,20,22の10個がとれる
以下、11個はとれないことを証明する
もし11個とれたとすると、1〜11か12〜22のどちらかに6個以上が含まれる
11個の連続した数字をZ/11Zで考えると条件はnが入っているときにn±4が入っていないこととなる
しかしZ/11Zを0,4,8,…,32,36,40と言う形で書いたとき
この中から6個以上選ぼうとすると必ず隣あった数字を選ぶ必要があり、矛盾する
0707132人目の素数さん
垢版 |
2020/08/25(火) 08:27:50.39ID:WzhAMzCC
>>705
補足
11と4は互いに素なのでZ/11Zにおいて0,4,8,…,36,40は全ての元をちょうど1度ずつ代表する
円形に並べて、40と0(=44)も隣あうと考える
0708132人目の素数さん
垢版 |
2020/08/25(火) 09:52:01.77ID:A/VRpGsl
>>688
>>704
(x^5+x^4+x^3)+(x^4+x^3)+x^3+6(1+x+x^2)+(x+x^2)+x^2=x^3(x^2+x+1)+x^2(x^2+x+1)+x(x^2+x+1)+(x^2+x+1)=(x^2+x+1)(x^3+x^2+x+1)=0
x^2+x+1=(x+1/2)^2+3/4>0
x^3+x^2+x+1=0
(x+1)(x^2-x+1)=0
x^2-x+1=(x-1/2)^2+3/4>0
∴x=-1
0710132人目の素数さん
垢版 |
2020/08/25(火) 10:25:10.27ID:RvbSTotP
0 = x^5 +2x^4 +3x^3 +8x^2 +7x +6
 = (x+2)(xx-x+3)(xx+x+1),

xx+x+1 = (x+1/2)^2 + 3/4 ≧ 3/4,
まで合ってる。
あと
xx-x+3 = (x-1/2)^2 + 11/4 ≧ 11/4,
から
x=-2
0712132人目の素数さん
垢版 |
2020/08/25(火) 12:42:20.58ID:RvbSTotP
>>649
10個のサイコロを投げるとして
 μ = (7/2)n = 35,
 p(μ) = 4395456/(6^n) = 0.0726928
 p(m) = p(μ){1 - 0.0160823・(m-μ)^2 + 1.2415E-4・(m-μ)^4
   - 6.1676E-7・(m-μ)^6 + 2.05E-9・(m-μ)^8 - ・・・・}

 σ^2 = 1/(2・0.0160823) = 31.09

(大意)
サイコロ1個の場合の分散は 35/12,
サイコロn個の場合の分布は、これをn個たたみ込んだものだから
 分散は (35/12)n,
しかし正規分布から外れているので
ピーク曲率から決めた σ^2 とは一致しない。いまの場合は
 σ^2 ≒ (35/12)n + 1.923
0713132人目の素数さん
垢版 |
2020/08/25(火) 14:48:39.67ID:BYmXedij
>>704
この方程式が整数解をもつことは明らかではないと思うけど、どうやって見つけるんだろう
適当に代入していったら x = -2 で 0 になる?これはちょっと無理がある気がする
(x^5+x^4+x^3)+(x^4+x^3)+x^3+6(1+x+x^2)+(x+x^2)+x^2
= x^3(1+x+x^2) + x^2(1+x+x^2) + x(1+x+x^2) + 6(1+x+x^2)
= (x^3 + x^2 + x + 6)(1+x+x^2)
までなら気が付けるだろうか
問題は g(x) = x^3 + x^2 + x + 6 の因数分解だが、これをどうやるか
g(x) = 0 は整数係数の 3 次方程式で x^3 の係数が 1 だから整数解が存在すると仮定して、
定数項 6 の約数から絞り込めばまあ見つかるけど
偶然簡単に解けるってだけなのかな
例えばもしこの定数項が 6 じゃなくて 8 とかだったら簡単に解ける気がしない
どうせ因数定理を使えば簡単に解ける問題なんだろって感じ?
0716132人目の素数さん
垢版 |
2020/08/25(火) 18:37:06.33ID:LqiSh/C2
Aを可換なネーター環で(1)(2)をみたすとします

(1) Aの冪零根基は(0)
(2) 任意の極大イデアルmに対して、局所化A_mは有限環

このときAは有限である

自然な準同型A→A_mで単射になるものがあると言おうとしたけど、A = Z/6Zとかで既にだめだったわ
0717132人目の素数さん
垢版 |
2020/08/25(火) 19:06:57.46ID:XiNDC1KH
累積分布関数
F(x)=4x(1-x) (0≦x≦1)
とする。

(1)確率密度関数f(x)を求めよ。

(2)メディアン、モード、ミーンを小さい順に並べよ。

(3)キュムラントを求めよ。
0721132人目の素数さん
垢版 |
2020/08/25(火) 21:08:34.18ID:lpRHuZYO
>>705
11個の式は作れないようだ

Table[2n-b-a+{(n+a)mod4}+4C(0,n-8+a),{a,0,1},{b,0,2},{n,1,10}]
0723132人目の素数さん
垢版 |
2020/08/25(火) 22:34:37.45ID:Cxs29Z8K
>>716
radA = p1‥pnとしてA→ΠA_piのkernelがradAになる気がする
A/radAが有限環ならradA^i/radA^(i+1)が全部有限加群になるからそれでいける希ガス
0726132人目の素数さん
垢版 |
2020/08/26(水) 01:24:23.05ID:xATZmZn/
累積分布関数が
F(x)=√(x^2+1) (0<x≦a)
で与えられている。

(1)実数aの値を求め、また確率密度関数f(x)を求めよ。

(2)E(a) = ∫[0→a] xf(x) dxを求めよ。

(3)V(a) = ∫[0→a] f(x){E(a)-x}^2 dxを求めよ。

(4)∫[0→a] √{E(x)V(x)} dxと標準偏差との関係について論ぜよ。
0728132人目の素数さん
垢版 |
2020/08/26(水) 03:48:26.09ID:E7gw9mcx
>>712
n=8個のサイコロを投げるとして
 μ = (7/2)n = 28,
 p(μ) = 135954/(6^n) = 0.0809435
 p(m) = p(μ){1 - 0.0197967・(m-μ)^2 + 1.8862E-4・(m-μ)^4
   - 12.736E-7・(m-μ)^6 + 7.43E-9・(m-μ)^8 - ・・・・}

 σ^2 = 1/(2・0.0197967) = 25.2567

n=12個のサイコロを投げるとして
 μ = (7/2)n = 42,
 p(μ) = 144840476/(6^n) = 0.0665388
 p(m) = p(μ){1 - 0.0135455・(m-μ)^2 + 0.8858E-4・(m-μ)^4
   - 3.7175E-7・(m-μ)^6 + 1.025E-9・(m-μ)^8 - ・・・・}

 σ^2 = 1/(2・0.0135455) = 36.9127
0729132人目の素数さん
垢版 |
2020/08/26(水) 05:28:18.51ID:E7gw9mcx
>>705
{1,2,・・・・,22} を下図にように並べる。

01 - 05 - 09 - 13 - 17 - 21
   | | | |
04 - 08 - 12 - 16 - 20
| | |
03 - 07 - 11 - 15 - 19
| | | |
02 - 06 - 10 - 14 - 18 - 22
| | | |
01 - 05 - 09 - 13 - 17 - 21

これは周期 (7,4)をもつ。同じ数が11つ隣にある。
隣り合うものが(白,黒)となるように彩色することはできない
そのように11個ずつに分けることはできない。

なお、上記のような分け方ができる炭化水素を
「交互炭化水素」というらしい。
0730132人目の素数さん
垢版 |
2020/08/26(水) 05:31:47.73ID:E7gw9mcx
                 01 - 05 - 09 - 13 - 17 - 21
                  |  |  |  |
               04 - 08 - 12 - 16 - 20
               |  |  |
          03 - 07 - 11 - 15 - 19
          |  |  |  |
     02 - 06 - 10 - 14 - 18 - 22
     |  |  |  |
01 - 05 - 09 - 13 - 17 - 21
0731132人目の素数さん
垢版 |
2020/08/26(水) 05:46:27.71ID:E7gw9mcx
1つ右に行くと+4
1つ下に行くと+7
a_{i,j} = 7i + 4j = a_{i-4,j+7} = a_{i+4,j-7}
0732132人目の素数さん
垢版 |
2020/08/26(水) 06:39:50.16ID:+5D/ly7R
>>728

n=8個のサイコロを投げるとして
 μ = (7/2)n = 28,
 p(μ) = 135954/(6^n) = 0.0809435
 p(m) = p(μ){1 - 0.0197967・(m-μ)^2 + 1.8862E-4・(m-μ)^4...

0.0197967や1.8862E-4はどこから出た数字なのでしょうか?
0733132人目の素数さん
垢版 |
2020/08/26(水) 10:10:01.59ID:8ae+cQFx
有限体上の特殊線形群の位数って、決定できますか?
0735132人目の素数さん
垢版 |
2020/08/26(水) 11:13:14.95ID:fEj1xUS4
>>734
これって1次独立なベクトルを数えてるのか
頭良いな
0736132人目の素数さん
垢版 |
2020/08/26(水) 14:27:14.39ID:Juzzfk9N
累積分布関数が
F(x)={√(x^2+x)}/2 (0≦x≦a)
で与えられている。

(1)F(x)=1となる正の実数aの値を求め、また確率密度関数f(x)を求めよ。

(2)E(a) = ∫[0→a] xf(x) dxを求めよ。

(3)V(a) = ∫[0→a] f(x){E(a)-x}^2 dxを求めよ。

(4)∫[0→a] √{E(x)V(x)} dxと標準偏差との関係について論ぜよ。
0737132人目の素数さん
垢版 |
2020/08/26(水) 14:36:21.84ID:VSkeehob
01 02
03 04 05
06 07 08 09
10 11 12 13 14
15 16 17 18 19 20
21 22 23 24 25 26 27
28 29 30 31 32 33 34 35
36 37 38 39 40 41 42 43 44

44 36
43 35 28
42 34 27 21
41 33 26 20 15
40 32 25 19 14 10
39 31 24 18 13 09 06
38 30 23 17 12 08 05 03
37 29 22 16 11 07 04 02 01

上の数列を下の数列に変換する
アルゴリズムを見つけてくれ(^_^)ノ
0739132人目の素数さん
垢版 |
2020/08/26(水) 17:34:24.13ID:XvaNrpWd
n≧3とする。縦2*nマス、横2*nマスのチェス盤から白、黒のマス目を1つずつ抜き取った欠損チェス盤で、
ドミノ牌で敷き詰められないものが存在するか。また、白、黒2個ずつ抜き取ったらどうか。

この問題ですが、2部グラフの完全マッチングの問題と考えていいでしょうか?
0740132人目の素数さん
垢版 |
2020/08/27(木) 03:58:44.01ID:dl/YlIUZ
>>737
Mathematicaでは、
In[21]:= Table[{k,p=Floor[(1/2)(Sqrt[8k+1]-1)],q=k+1-p(p+1)/2,(9-q)(10-q)/2+9-p},{k,1,44}]
で、下のような出力を得られます。上の三角形で、pは何行目か、qは何列目かを表し、それに対応する値に変換する式が、(9-q)(10-q)/2+9-pです。
Out[21]= {{1, 1, 1, 44}, {2, 1, 2, 36}, {3, 2, 1, 43}, {4, 2, 2, 35}, {5, 2, 3, 28}, {6, 3, 1, 42}, {7, 3, 2, 34}, {8, 3, 3, 27}, {9, 3, 4, 21},
> {10, 4, 1, 41}, {11, 4, 2, 33}, {12, 4, 3, 26}, {13, 4, 4, 20}, {14, 4, 5, 15}, {15, 5, 1, 40}, {16, 5, 2, 32}, {17, 5, 3, 25}, {18, 5, 4, 19},
> {19, 5, 5, 14}, {20, 5, 6, 10}, {21, 6, 1, 39}, {22, 6, 2, 31}, {23, 6, 3, 24}, {24, 6, 4, 18}, {25, 6, 5, 13}, {26, 6, 6, 9}, {27, 6, 7, 6},
> {28, 7, 1, 38}, {29, 7, 2, 30}, {30, 7, 3, 23}, {31, 7, 4, 17}, {32, 7, 5, 12}, {33, 7, 6, 8}, {34, 7, 7, 5}, {35, 7, 8, 3}, {36, 8, 1, 37},
> {37, 8, 2, 29}, {38, 8, 3, 22}, {39, 8, 4, 16}, {40, 8, 5, 11}, {41, 8, 6, 7}, {42, 8, 7, 4}, {43, 8, 8, 2}, {44, 8, 9, 1}}
0741132人目の素数さん
垢版 |
2020/08/27(木) 11:20:19.71ID:17pR8ej1
>>739
下のように○と●のところを抜き取ると明らかにドミノ牌で敷き詰められません。
解答を見ると、「白、黒1つずつのときは存在しないが、2個ずつのときは存在する。しかし、その例は省略。」となっていました。
簡単な例があるのに省略しているのがなぜなのか分かりません。
このような自明な例以外に、白、黒2個ずつ抜き取った場合に、ドミノ牌が敷き詰められない例はありますか?
与えられた2部グラフが完全マッチングを持つかどうかを計算するプログラムを作って、調べてみましたが、
n = 3, 4, 5のときには自明な抜き取り方以外の抜き取り方をした場合、すべて敷き詰め可能でした。
多分、nが大きくなっても、白、黒2個ずつ抜き取るだけなので、直感的に考えて状況は変わらないと思いますが、どうでしょうか?

□■□■○■
■□■□■○
□■□■□■
■□■□■□
□■□■□●
■□■□●□
0743132人目の素数さん
垢版 |
2020/08/27(木) 11:43:17.41ID:17pR8ej1
>>742
解答は、上に書いたように、「白、黒1つずつのときは存在しないが、2個ずつのときは存在する。しかし、その例は省略。」です。
ラスロウ・ロバース他著(秋山仁+ピーター・フランクル翻案)『入門組合せ論』という本に載っている問題です。
0744132人目の素数さん
垢版 |
2020/08/27(木) 11:51:03.33ID:+ZYod5S8
>>742
自明に無理でこんなの自分で見つけられないようなら本書を読む資格ない
位のノリなのかも
まぁ一個見つけられたんだからいいんじゃないの?
0745132人目の素数さん
垢版 |
2020/08/27(木) 11:59:28.08ID:17pR8ej1
>>744
ありがとうございます。なんか気持ちが悪いですが、気にせず先に進もうと思います。
0747132人目の素数さん
垢版 |
2020/08/27(木) 13:45:18.43ID:17pR8ej1
平面にn個の円を描くと、平面はいくつかの領域に分割される。“境界弧を共有して隣り合う”領域が同じ色にならないように、平面全体を2色
で色分けできる。

証明:
n = 1のとき、縁の外側を白、内側を黒で塗る。
“円の個数がnのとき正しい色分けが可能である”と仮定する。円の個数がn+1のときを考える。いま、n個の円で領域が分割されている場合を
想定しよう。帰納法の仮定より、これは正しく色分けできる。次に、n+1番目の円を加えて、その円の外側の領域の色はそのままにして、内側の
領域をいままで塗ってあった色と反対の色にする。この色分けの仕方は正しい色分けを与えること以下で確認しよう。
上述の色分けの仕方は、n+1番目の円以外のn個の円の弧だけを境界とする領域については仮定により正しい。なぜならば、弧の両側の領域の
色は、その領域がn+1番目の円の内側にあれば両方とも変わり、またその領域が外側にあれば両方とも変わらないからである。また、n+1番目の
円の弧が領域の境界になるときは、その前に1つだった領域が弧を描いた後に2つに分割され、このうちの内側は色を変え、外側は不変だからである。
これで証明は完了した。

以下の部分の日本語が分かりません。一体何が言いたいのでしょうか?

「上述の色分けの仕方は、n+1番目の円以外のn個の円の弧だけを境界とする領域については仮定により正しい。なぜならば、弧の両側の領域の
色は、その領域がn+1番目の円の内側にあれば両方とも変わり、またその領域が外側にあれば両方とも変わらないからである。また、n+1番目の
円の弧が領域の境界になるときは、その前に1つだった領域が弧を描いた後に2つに分割され、このうちの内側は色を変え、外側は不変だからである。」
0749132人目の素数さん
垢版 |
2020/08/27(木) 14:24:30.16ID:17pR8ej1
>>747
分かりました。以下のように自分で言葉を補いました。
「上述の色分けの仕方は、n+1番目の円以外のn個の円の弧だけを境界とする領域については仮定により正しい。なぜならば、弧の両側の領域の
色は、その両側の領域がn+1番目の円の内側にあれば両方とも変わり、またその両側の領域が外側にあれば両方とも変わらないからである。また、n+1番目の
円の弧が領域の境界になるときは、その前に1つだった領域が弧を描いた後に2つに分割され、このうちの内側は色を変え、外側は不変だからである。」
0750132人目の素数さん
垢版 |
2020/08/27(木) 14:41:10.91ID:npSkWuIN
何が聞きたいのかよくわからん質問だな。「何が言いたいのか?」と聞かれても、書いてある通りのことが言いたいのだろうとしか。
0751132人目の素数さん
垢版 |
2020/08/27(木) 14:49:11.10ID:17pR8ej1
組合せ論の天才ロバース(数学オリンピック3回金メダル)の本だけあって面白い問題がありました。

□□□□□□□□
□□□□□□□□
□□□□□□□□
□□□□□□□□
□□□□□□□□
□■□□□□□□
□□□□□□□□
□□□□□□□□

2^n × 2^nのチェス盤から1つのマス目だけ取り除いた欠損チェス盤は以下のL字牌で敷き詰められることを証明せよ。


□□
0752132人目の素数さん
垢版 |
2020/08/27(木) 14:50:29.27ID:s3GY++rV
ジューコフスキー変換の逆変換をやってみようと思って、
ド・モアブルの定理で計算できないかなと試みたのですが、
ジューコフスキー変換で算出した座標に適用しても元の座標に戻りませんでした。
ジューコフスキー変換の逆変換について詳しく説明してるウェブなど、
何らかのアドバイスがあったらレスください。
0755132人目の素数さん
垢版 |
2020/08/27(木) 16:58:10.63ID:iR9F5nK1
集合Sは相異なる(2^n+1)個の自然数a[1],...,a[2^n]を要素とし、同様に集合Tも相異なる(2^n+1)個の自然数b[1],...,b[2^n]を要素とする。
またa[1]=b[1]=2である。
いずれの集合についても、すべての要素の和は2の累乗の形で表せる。
集合S,Tからそれぞれ任意に1つの要素を選び交換しても、交換後のS及びTのすべての要素の和は2の累乗の形である。
集合S,Tとして考えられるものをすべて決定せよ。
0757132人目の素数さん
垢版 |
2020/08/27(木) 21:25:59.58ID:uwydrJ8F
2^n個に直したとしても
S = T = {2} しかないことが簡単に証明できるな
0758132人目の素数さん
垢版 |
2020/08/27(木) 21:50:49.79ID:pzbetnOc
マセマティカを使ってクラドニ図形の3Dの振動の図形のシミュレーションをしようとしていて、それについて論文を書くために、高校レベルに簡略化された式でシミュレーションができないかやっています。先生にはSin[mπx]Sin[nπy]Cos[t]という式を使えと言われたのですが、どこを調べてもどうしてこの式が導かれたのかがわかりません・・。そもそもの問題をどれくらい私が理解できているかもわからないところですが、だれかなんとか助けていただけないでしょうか・・・。
0759132人目の素数さん
垢版 |
2020/08/28(金) 07:02:13.47ID:mKzga76c
AB=a,AC=4a,∠BAC=60°の△ABCにおいて、∠BACの二等分線とBCとの交点をD、ACの中点をM、Cから直線ABに下ろした垂線の足をHとする。
面積比△DMH:△ABCをaで表せ。
0760132人目の素数さん
垢版 |
2020/08/28(金) 07:16:37.35ID:mKzga76c
Σ[k=0,n] 3^k が2の累乗となるような1以上の整数nを全て求めよ。
ただし1以上の整数aに対しa^0=1とする。
0762132人目の素数さん
垢版 |
2020/08/28(金) 12:26:53.37ID:vOxVc6EM
>>761
ヤフー知恵袋でも聞かれているので有名問題かと思ったら一橋大学に似たような入試問題があったようです。
それにしてもよく見つけてこられましたね
0763132人目の素数さん
垢版 |
2020/08/28(金) 12:28:12.42ID:2EjgpYll
n^2 + 1個の相異なる整数からなる数列には、長さn+1の増加部分列があるか、あるいは長さn+1の減少部分列があることを証明せよ。

証明:
s : a_1, a_2, …, a_{n^2+1}を相異なる整数からなる列とする。sは長さn+1の増加部分列を含まないとしよう。ここで、各a_k(k = 1, …, n^2+1)に
ラベルl_kを次の規則によってつける:各l_kはa_kから始まるsの最長増加部分列の長さとする。このとき、あるラベルl_jが存在して、sのn+1項以上が
このラベルl_jをもつことが鳩の巣原理よりわかる。ラベルl_jをもつsのn+1項以上は、減少部分列を与える。


「このとき、あるラベルl_jが存在して、sのn+1項以上がこのラベルl_jをもつことが鳩の巣原理よりわかる。」がなぜそう言えるのか分かりません。
0764132人目の素数さん
垢版 |
2020/08/28(金) 12:33:03.33ID:2EjgpYll
>>763
ラスロウ・ロバース他著(秋山仁+ピーター・フランクル翻案)『入門組合せ論』という本に載っている問題です。
0767イナ ◆/7jUdUKiSM
垢版 |
2020/08/28(金) 15:19:28.28ID:GS8HrOsp
>>725
>>759
メネラウスの定理、チェバの定理、ピタゴラスの定理より△AHC内の辺の比を確定し、DからHMに引いた垂線DNをxとおくと、
Aを起点にメネラウスの定理より、
(AB/BH){2a√3/(2a√3-2a/√3)}(a/√3+x)/(a√3-x)=1
x=a√3/5
△ABC=a^2√3
△DMH=ax/2=a^2√3/10
∴△DMH:△ABC=1:10
0768132人目の素数さん
垢版 |
2020/08/28(金) 16:54:47.12ID:sIUBKqp2
>>763
長さn+1の増大列はないと仮定したんだから全てのl_kはn以下のはず
(l_kは{a_1,a_2,…,a_(n^2+1)}から{1,2,…,n}への写像となる)
つまりn^2+1個のものにn以下の数を割り当てていくんだから、
鳩の巣原理によってどこかでn+1個が被った数bを割り当てられてしまう
0770132人目の素数さん
垢版 |
2020/08/28(金) 17:36:28.76ID:Ji3HI3EV
>>759
作図してプログラムに計算させてみた。

https://i.imgur.com/yFv7F0m.png

sim <- function(a=1){
plot(NULL,xlim=c(0,5*a),ylim=c(0,2*a),asp=1,ann=F,axes=F)
A=0i ; pt(A,'A')
C=4*a+0i ; pt(C,'C')
B=a*cos(pi/3)+1i*a*sin(pi/3) ; pt(B,'B')
seg(A,B) ; seg(B,C) ; seg(C,A)
d=cos(pi/6)+1i*sin(pi/6)
D=intsect(B,C,A,d)
pt(D,'D')
seg(A,D,lty=3)
M=1/2*(A+C) ; pt(M,'M')
# y-0=tan(pi/3+pi/2)*(x-4*a)
y0=-4*a*tan(pi/3+pi/2)
H=intsect(A,B,C,y0*1i)
pt(H,'H')
seg(B,H,lty=3)
seg(C,H,lty=3)
# △DMH:△ABC
seg(D,M,col=2)
seg(M,H,col=2)
seg(D,H,col=2)

Tri <- function(a,b,c){
s=(a+b+c)/2
S=sqrt(s*(s-a)*(s-b)*(s-c))
return(S)
}
DMH=Tri(abs(D-H),abs(H-M),abs(M-D))
ABC=Tri(abs(A-B),abs(B-C),abs(C-A))
DMH/ABC
}

実行結果
> sim(runif(1))
[1] 0.2
0771132人目の素数さん
垢版 |
2020/08/28(金) 17:39:24.20ID:W4G73UvR
先週毎日新聞に載っていた確率の問題がどうしても納得できないんだけど解説してくれる人お願い!
【問題】
川へ洗濯に行ったおばあさんは、流れてきた桃を10個拾って家に持ち帰りました。そのうち、後で自分たちが食べる分を1つだけ残し、9個をお客に出しました。お客に出した桃は1個だけ残りました。
その後おばあさんは、隣人に、残った桃をおすそ分けすることを約束しました。
しかしそのすぐ後で、おじいさんから『川から流れてきた桃の1つに桃太郎が入っていたらしい』という話を聞きました。
お客に出した桃には、桃太郎は入っていませんでした。是非とも桃太郎が入った桃が欲しい2人ですが、どちらか1つはお隣におすそ分けしなければなりません。
さて、どちらの桃を残せばいいでしょう?また、残した桃に桃太郎が入っている確率は何%でしょう?

【新聞記載の答え】
10個の桃を1個と9個に分けた時、それぞれのグループに桃太郎が入っている確率は10%と90%となります。お客に出した9個のうち8個には桃太郎は入っていなかったので、残りの1個に入っている確率は90%。よって、お客に出して残った桃を残すべきです。

客が桃太郎の有無で桃を選んだわけじゃないんだし俺は50%じゃないんかな?と思うんだけどどう?
0773132人目の素数さん
垢版 |
2020/08/28(金) 18:16:13.78ID:mDb6M5mP
モンティホールの亜種を狙ったっぽいな。
お客がレントゲンか何かで中身をチェックできたと考えれば新聞の答えでいいと思う。
0775132人目の素数さん
垢版 |
2020/08/28(金) 18:42:50.26ID:CiJT+a8W
8個は既に切り分けたのに桃太郎が入ってなかったということでしょう
なので90%
0776132人目の素数さん
垢版 |
2020/08/28(金) 18:55:05.14ID:sIUBKqp2
客が桃太郎の入ってない桃を選んだわけではないから1/2だよ
9人の客に9個の桃を渡して、8番目の客までが食べ、9番目の客がまだ食べてない状況と考えて、自分が10番目の客だと思えばいい
9番目と10番目に違いはない

モンティホール的にするには客に透視能力があって桃太郎の入ってる桃は選ばない、というような設定が必要
0777イナ ◆/7jUdUKiSM
垢版 |
2020/08/28(金) 19:58:00.95ID:GS8HrOsp
>>767
>>771
桃10個のうち1個に桃太郎が入っているからその桃に桃太郎が入っている確率は(1/10)×100%=10%
お客に出して残った桃もお客に出さなかった桃もその桃に桃太郎が入っている確率は10%
∴どちらを残してもいい。
その桃に桃太郎が入っている確率は10%
俺は惑わされない。
0778132人目の素数さん
垢版 |
2020/08/28(金) 20:31:44.07ID:oBaL3ybI
この手のやつはどのようにしてハズレだと判明したのかが問題なのにそこを理解していないやつが記事書いたってことかな
0780イナ ◆/7jUdUKiSM
垢版 |
2020/08/28(金) 21:39:05.43ID:GS8HrOsp
>>777
>>779
流れてきた桃をお婆さんがぜんぶ拾ったら10%になると思う👵🍑
川に流してしまったら10%未満。
お爺さんが聞いてきた話が100%ほんとなら10%の確率でどの桃も桃太郎が入っている可能性がある。
お爺さんが聞いた話の信憑性によっては10%未満。
お客に出して残ったら桃が桃太郎になるなら、お爺さんもお婆さんも交替制でお客に桃を振る舞うと思う。
0781132人目の素数さん
垢版 |
2020/08/28(金) 21:42:32.20ID:5u8NPnD9
>>771
客は無作為に桃を選んだと仮定してシミュレーションしてみた。

m=c(rep(0,9),1) # 桃10個 1が桃太郎入りのもも
sim <- function(){
i=sample(10,1)
a=m[i] # a:自分達用に残した桃
j=sample(9,1)
b=m[-i][j] # b:客が残した桃
c=m[-i][-j] # c:客が食べた桃8個
c(a,b,c)
}
momo=t(replicate(1e6,sim()))
fa <- function(x){ # 客が食べた桃cに桃太郎がいなくて
sum(x[3:10]==0) & x[1]==1 # かつ、自分達用に残した桃aに桃太郎がいればTRUEを返す
}

fb <- function(x){ # 客が食べた桃cに桃太郎がいなくて
sum(x[3:10]==0) & x[2]==1 # かつ、客が残した桃bに桃太郎がいればTRUEを返す
}

mean(apply(momo,1,fa)) # 自分達用の桃に桃太郎
mean(apply(momo,1,fb)) # 客の残した桃に桃太郎

> mean(apply(momo,1,fa))
[1] 0.100156
> mean(apply(momo,1,fb))
[1] 0.099983

当然ながら、どちらの確率も同等というシミュレーション結果になった。
0782132人目の素数さん
垢版 |
2020/08/28(金) 21:47:05.48ID:/8vUoX3h
1/n! * ∫[t=2n, ∞] dt t^n e^(-t) → 0 ( n→ ∞ )
を示してください。
簡単そうに思えたのですがどうしたらいいか分からなくなりました。

https://imgur.com/a/Kyf8LdB
鞍点法によるスターリング公式の導出過程で現れたので、
できればスターリング公式を使わないルートでお願いします。
0783132人目の素数さん
垢版 |
2020/08/28(金) 21:56:50.52ID:5u8NPnD9
当然ながら、厳密解は

> u
[,1] [,2] [,3] [,4] [,5] [,6] [,7] [,8] [,9] [,10]
[1,] 0 0 0 0 0 0 0 0 0 1
[2,] 0 0 0 0 0 0 0 0 1 0
[3,] 0 0 0 0 0 0 0 1 0 0
[4,] 0 0 0 0 0 0 1 0 0 0
[5,] 0 0 0 0 0 1 0 0 0 0
[6,] 0 0 0 0 1 0 0 0 0 0
[7,] 0 0 0 1 0 0 0 0 0 0
[8,] 0 0 1 0 0 0 0 0 0 0
[9,] 0 1 0 0 0 0 0 0 0 0
[10,] 1 0 0 0 0 0 0 0 0 0
> mean(apply(u,1,fa))
[1] 0.1
> mean(apply(u,1,fb))
[1] 0.1

でどちらの桃を選んでも同じ。
0784771
垢版 |
2020/08/29(土) 00:00:16.61ID:ZmqVm7kK
桃太郎の問題、みんなありがとう!
変わらないって意見が多くて安心しました。スッキリです!

ちなみに問題は会員じゃないから確認できないけど多分この記事の中にあるはず
https://mainichi.jp/articles/20200816/ddv/010/070/019000c
0785132人目の素数さん
垢版 |
2020/08/29(土) 01:26:18.98ID:gICDV3If
>>782
多分、 n > 0 のとき、不等式
Σ[k=0,n] (2n)^k/k! < (1/n)*Σ[k=0,2n] (2n)^k/k! < e^(2n)/n
が成立すると思うんだけど、証明がわからない
0786132人目の素数さん
垢版 |
2020/08/29(土) 07:54:19.70ID:nEvr3uHf
>>782
(1/n!)∫[t=2n,∞] (t^n) e^(-t)dt
 = e^(-2n) Σ[k=0,n] (2n)^k / k!
 〜 √(2/πn) (2/e)^n,
スターリングを使わずに示すのは
簡単とは思えないが・・・・
0787132人目の素数さん
垢版 |
2020/08/29(土) 07:59:00.42ID:nEvr3uHf
>>782
参考までに、別の本から概略を引用すると、

log(n) に ∫[n-1/2,n+1/2] log(x)dx を代用すれば、
 log((n-1)!) = (n-1/2)log(n) -n +1 -δ,
従って
 Γ(n) = (n-1)!
   = n^(n-1/2) e^(-n+1-δ)
   = a n^(n-1/2) e^{-n+μ(n)},   (1)
 lim(n→∞) μ(n) = 0.        (2)
定数aはWallisの公式(253頁,(9))
 √π = lim(n→∞) (n!)^2・2^(2n) / ((2n)!・√n),
から簡単に求められる。
すなわち (1) から代入して, (2) を用いれば、
 √π = lim(n→∞) ・・・・ = a/√2.
故に
 a = √(2π).
よって (1) から, nを掛けて
 n! 〜 √(2π) n^(n+1/2) e^(-n).
これが Stirling の公式である。
 Stirlingの公式は簡単に得られたが、・・・・

高木貞治:「解析概論」改訂第三版, 岩波書店 (1961)
 第5章 解析函数 §69. p.258-259
0788132人目の素数さん
垢版 |
2020/08/29(土) 08:50:52.62ID:nEvr3uHf
>>785
 Σ[k=1,2n] (2n)^k / k!
 = Σ[k=1,n] {(2n)^k / k! + (2n)^{k+n} / (k+n)!}
 = Σ[k=1,n] {(2n)^k / k!} {1 + (2n)^n / [(k+1)(k+2)・・・・(k+n)]}
 ≧ Σ[k=1,n] {(2n)^k / k!} {1 + (n-1)}    (←*)
 = n Σ[k=1,n] (2n)^k / k!,
かな?

(*)
 (2n)^n /(k+1)(k+2)・・・・(k+n) ≧ (2n)^n / [(n+1)(n+2)・・・・(2n)]
 = (2n)^{n-1} / [(n+1)(n+2)・・・・(2n-1)]
 > (2n)^{n-1} / (3n/2)^{n-1}     (GM-AM)
 = (4/3)^{n-1}
 > n-1,    (n≧8)
0789132人目の素数さん
垢版 |
2020/08/29(土) 09:01:06.05ID:nEvr3uHf
>>771
 近所のスーパーかコンヴィニで1つ買って
それをお隣にあげればいいんぢゃね?
0790132人目の素数さん
垢版 |
2020/08/29(土) 09:06:38.47ID:c31TQaq/
実数a,b,cは1≦a≦2,1≦b≦2,1≦c≦2を動く。
p,q,rをp=bc/(a+b),q=ca/(b+c),r=ab/(c+a)と定め、xyz空間の点(p,q,r)が動いてできる立体Kを考える。

(1)rの取りうる値の範囲を求めよ。

(2)実数tを(1)の範囲内の値とする。Kを平面z=tで切った切り口の概形を図示せよ。

(3)Kの体積を求めよ。
0791782
垢版 |
2020/08/29(土) 10:31:35.00ID:MuxeN7Uy
>>785, >>788 これ分かり易いですね! ありがとうございます。

スターリング使わないで...とは言ったものの
>>786
e^(-2n) Σ[k=0,n] (2n)^k / k! 〜 √(2/πn) (2/e)^n
どうやったらこの式が導出できるのか教えてほしいです。
0792132人目の素数さん
垢版 |
2020/08/29(土) 11:46:39.49ID:pjypKnM7
ラスロウ・ロバース他著(秋山仁+ピーター・フランクル翻案)『入門組合せ論』という本に以下の定理が書いてあります。
なぜ、「n以外のnの約数」と書いているのでしょうか?
n = 7のとき7以外の7の約数で素数であるものは存在しませんが
φ(7) = 6 = 7 * (1 - 1/7)
と定理を適用できます。

φ(n)をn以下の自然数でnと素であるものの個数とする。
n以外のnの約数で素数であるものをp_1, …, p_kとする。このとき
φ(n) = n * (1 - 1/p_1) * … * (1 - 1/p_k)
が成り立つ。
0793132人目の素数さん
垢版 |
2020/08/29(土) 11:54:18.56ID:nEvr3uHf
t≒2n では
 e^{t/2n} ≒ (e/2n)t,
 t^n ≒ (2n/e)^n・e^{t/2},
t >>2t では 小さいから
∫[t=2n,∞] (t^n) e^{-t} dt
 ≒ (2n/e)^n ∫[t=2n,∞] e^{-t/2} dt
 = (2n/e)^n [ 2e^{-t/2} ](t=2n,∞)
 = (2n/e)^n・2e^{-n}
 = 2(n/e)^n・(2/e)^n
 〜 2{n!/√(2πn)} (2/e)^n, (スターリング)
0794132人目の素数さん
垢版 |
2020/08/29(土) 12:02:30.60ID:a4jrTFKD
>>792
たしかに意味がないように思える
nが素数でないときは「n以外のnの約数で素数であるもの」と「nの約数で素数であるもの」は同じことになるし、
nが素数のときは、「nの約数で素数であるもの」はnだけなのでその計算式はn(1-1/n)=n-1となり、「n以下の自然数でnと素であるものの個数」と一致する
0795132人目の素数さん
垢版 |
2020/08/29(土) 12:17:16.24ID:nEvr3uHf

e^x ≧ e・x (等号成立はx=1) を使った。
t> >2n では小さいから誤差を無視して…
0797132人目の素数さん
垢版 |
2020/08/29(土) 12:46:22.59ID:MuxeN7Uy
>>793
初見では少し?な近似でしたが、
∫[2n,2n+M] dt t^n e^{-t}
= ∫[0,M]dt (t+2n)^n e^{-t-2n}
= e^{-2n} (2n)^n ∫[0,M]dt (1+t/2n)^n e^{-t}
≒ e^{-2n} (2n)^n ∫[0,M]dt e^{+t/2} e^{-t} (t≦ M << n)
= e^{-2n} (2n)^n ∫[0,M]dt e^{-t/2}
= ... (M→∞)
なるほど納得しました。ありがとうございます。
0798132人目の素数さん
垢版 |
2020/08/29(土) 13:14:08.64ID:c31TQaq/
>>790
(1)
a,b,cは独立に動くことに留意する。
ab/(c+a)=b[1-{c/(c+a)}]
=f(b|a,c)
cを固定してc/(c+a)=g(a|c)とaの関数と見る。ここでg(x|y)はxが変数、yを定数とみなした表記である。
c>0よりg(a|c)は単調減少。よってcの値に関わらず、
Max{g(a|c)}=g(1|c),min{g(a|c)}=f(2|c)
次にcの固定を外し、g(a|c)をcの関数g(c|a)とみる。aを固定した上でg(c|a)の増減を調べる。
g(c|a)=c/(c+a)=(c+a-a)/(c+a)=1-a/(c+a)
a>0より、g(c|a)は単調増加。
よってg(a|c)はcに関して単調増加かつaに関して単調減少で、
Max{g(a|c)}=g(1|2),min{g(a|c)}=g(2|1)
したがって1/3≦g(a|c)≦2/3であるから、
b/3≦f(b|a,c)≦2b/3
min{f(b|a,c)}=f(1|1,2)=1/3
Max{f(b|a,c)}=f(2|2,1)=4/3
以上より、1/3≦ab/(c+a)≦4/3
0799132人目の素数さん
垢版 |
2020/08/29(土) 13:39:47.67ID:c31TQaq/
>>790
(2)
1/3≦t≦4/3なるtを1つ固定する。
t=ab/(c+a)より、b=t(c+a)/a
よって
p=bc/(a+b)=tc(c+a)/[t(c+a)+a^2]
q=ca/(b+c)=(c*a^2)/[t(c+a)+ca]

ここから先で2変数a,cをどう扱えば良いか分かりません。固定されたtの値によってa,cの範囲も変わってくるので場合分けが必要だと思いますが、どうしたら良いでしょうか。
0800132人目の素数さん
垢版 |
2020/08/29(土) 16:17:07.46ID:pjypKnM7
>>787
解析概論を読み込んでいるんですか?
0801132人目の素数さん
垢版 |
2020/08/29(土) 16:55:20.16ID:pjypKnM7
>>787
lim(n→∞) ・・・・ = a/√2.

これはなぜですか?
0802132人目の素数さん
垢版 |
2020/08/29(土) 16:59:08.21ID:pjypKnM7
>>801
うまい具合に消えてくれるんですね。
0803イナ ◆/7jUdUKiSM
垢版 |
2020/08/29(土) 17:54:54.71ID:fW6yRWVP
>>780
>>790(1)
a=b=1,c=2のときr=ab/(c+b)=1/(2+1)=1/3
a=√2,b=2,c=1のときr=ab/(c+a)=2√2/(1+√2)=2√2(√2-1)=4-2√2
∴1/3≦r≦4-2√2
1/3≦r≦4-2√2
0804イナ ◆/7jUdUKiSM
垢版 |
2020/08/29(土) 18:00:14.85ID:fW6yRWVP
>>790(1)訂正。
a=b=1,c=2のときr=ab/(c+b)=1/(2+1)=1/3
a=b=2,c=1のときr=ab/(c+a)=4/(1+2)=4/3
∴1/3≦r≦4/3
0806イナ ◆/7jUdUKiSM
垢版 |
2020/08/29(土) 19:02:11.28ID:fW6yRWVP
>>804
>>790(2)
z座標がab/(c+a)=tのとき、
x座標はbc/(a+b)=ab(b-t)/(a+b)t
y座標はca/(b+c)=(b-t)a^2/(ab-at+bt)
1/3≦t≦4/3だから、
ab(3b-1)/(a+b)≦x≦ab(3b-4)/4(a+b)
(3b-1)a^2/(3ab-a+b)≦y≦(3b-4)a^2/(3ab-4a+4b)
0807132人目の素数さん
垢版 |
2020/08/29(土) 19:30:50.30ID:nEvr3uHf
>>797
蛇足だけど
マクローリン展開
log(1+x) = ∫[0,x] 1/(1+y) dy = x - (1/2)x^2 + (1/3)x^3 - ・・・・,
で x = e^{t'/2n} -1 とおいて
t'/2n = [e^{t'/2n} -1] - (1/2)[e^{t'/2n} -1]^2 + ・・・・,
2n+t' = 2n (e^{t'/2n} - (1/2)[e^{t'/2n} -1]^2 + ・・・・),
(2n+t')^n
 ≒ (2n)^n (e^{t'/2} - (n/2)[e^{(n+1)t'/2n} - 2e^{t'/2} + e^{(n-1)t'/2n}] ),
(2n+t')^n e^{-t'}
 ≒ (2n)^n (e^{-t'/2} - (n/2)[e^{-(n-1)t'/2n} - 2e^{-t'/2} + e^{-(n+1)t'/2n}] ),
∫[2n,∞] (t^n) e^{-t} dt
 = e^{-2n}∫[0,∞] (2n+t')^n e^{-t'} dt'
 ≒ e^{-2n} (2n)^n {2 - n[n/(n-1) -2 +n/(n+1)]}
 = (2/e)^n (n/e)^n {2 - 2n/(nn-1)}
 ≒ (2/e)^n n! √(2/πn) {1 - n/(nn-1)},
0809132人目の素数さん
垢版 |
2020/08/29(土) 20:21:40.53ID:MSAwsPl+
>>799
どなたかこれをお願いします。
定義域が複雑に変わる2変数の処理が難しくて思いつきません。対称性を上手く使えないかとも思いましたが、思いつきません…
0810132人目の素数さん
垢版 |
2020/08/30(日) 04:27:47.99ID:2u9lqC6Z
(数列の)無限小について質問です。

o(a_n)と書いた時、o(*)は関数の記号をイメージさせるように思います。
小平邦彦著『解析入門』には、o(a_n)という記号のイメージはo×a_nであると書いてあります。
どっちをイメージするのが普通でしょうか?
0811132人目の素数さん
垢版 |
2020/08/30(日) 04:31:14.64ID:2u9lqC6Z
o(a_n)が関数の記号をイメージさせるというのを説明しますと、数列a_nにa_nよりも高次の無限小である数列の集合を対応させる関数o(a_n)という
イメージです。
一方、掛け算のイメージを説明しますと、b_n = o(a_n)というのをb_n = o × a_n(oは無限小である、ある数列)と考えるというイメージです。
0812132人目の素数さん
垢版 |
2020/08/30(日) 06:33:27.00ID:8qkie79e
>>782
これ面白いな
何が起こってるのか自分なりに整理してみたら
ポアソン分布の中心極限定理=スターリングの公式
ってことが分かった
コンビネーションあるいは指数関数の性質のおかげで
ポアソン分布e^(-λ)λ^k/k!は再生性を持っている(合成系が再び同分布になる)から
中心極限定理によってN合成系のポアソン分布
e^(-N)N^k/k! (平均N、偏差√N)
はNが大きくなっていくと同平均同偏差の正規分布
1/√(2πN)e^(-(k-N)^2/2N) (平均N、偏差√N)
に近づいていく
k=Nを代入すると
スターリングの公式N!〜√(2πN)N^N/e^N
が得られる
また偏差が√NであることからNが大きくなったとき
中心(=平均)Nから±N/2(←√Nに比べて非常に大きい)の範囲にほとんどの確率が入ってくる
逆に言えばk=1〜N/2という裾野に入る確率はゼロになっていく
これはまさにe^(-N)Σ[k=1,N/2] N^k/k!→0(N→∞)を意味している
0813132人目の素数さん
垢版 |
2020/08/30(日) 08:40:20.57ID:wS+E/Mk9
数値例
I_n = (1/n!)∫[2n,∞] (t^n)e^{-t} dt,
J_n = (1/n!)(2n/ee)^n *2(1-n/(nn-1)), >>807
K_n = (1/n!)(2n/ee)^n *2,      >>793

n, I_n, J_n, K_n
-------------------------------------------------
4, 0.0996324005, 0.0839700231, 0.114504577
8, 0.02198725355, 0.02093050770, 0.02397494519,
16, 0.001391667287, 0.001371653306, 0.001463479469,
32, 7.4429824464E-6, 7.4125663716E-6, 7.6519227025E-6,
64, 2.90400556165E-10, 2.9007904985E-10, 2.9468462147E-10,
128, 6.1223407676E-19, 6.12056673216E-19, 6.1687631358E-19,
0814132人目の素数さん
垢版 |
2020/08/30(日) 09:34:02.53ID:8qkie79e
>>782
ガンマ分布とポアソン分布の双対性
∫[t=0,t'] t^s'e^(-t)/s'! + Σ[s=0,s'] t'^se^(-t')/s! =1
とも思えるな

この式、すごく不思議だ
0815782
垢版 |
2020/08/30(日) 09:53:42.39ID:NMQHWWIV
せっかくなので >>782 の式の証明を整理してみました。
https://imgur.com/a/Hi8iXiB

https://imgur.com/a/Kyf8LdB
と合わせて、スターリング公式の証明が完成しました。

偉い人が見たら こんなの証明とは言えん!
と怒られそうですが、鞍点法は手軽さが売りみたいなものですからね。
0817132人目の素数さん
垢版 |
2020/08/30(日) 11:27:29.18ID:wS+E/Mk9
>>788
 e^{2n} > Σ[k=1,2n] (2n)^k / k!
 = ・・・・
 > Σ[k=1,n] {(2n)^k / k!} (1 + (4/3)^{n-1})
 > (4/3)^{n-1} Σ[k=0,n] (2n)^k / k!,
だから
 e^{-2n} Σ[k=0,n] (2n)^k / k! < (3/4)^{n-1} = 0.75^n (4/3),
これは
 (左辺) ≒ (2/e)^n √(2/πn) = 0.73576^n √(2/πn)
より少し大きい。
0818132人目の素数さん
垢版 |
2020/08/30(日) 12:07:17.74ID:5BeLtRJT
>>811
どっちも同じことじゃないの?
迷ったら定義に戻ればいい
b_n = o(a_n) の定義は b_n/a_n → 0 でしょ?
だから ε_n := b_n/a_n とすれば ε_n → 0 かつ b_n = (ε_n)*(a_n)
0819132人目の素数さん
垢版 |
2020/08/30(日) 12:26:19.90ID:5BeLtRJT
>>811
強いて言うなら

>a_nよりも高次の無限小

という表現はイマイチかな
b_n = o(a_n) でも b_n → 0 とは限らないからね
例えば、 a_n := n^2 のとき、
b_n := n とすれば b_n = o(a_n) as (n→∞) だが b_n → ∞ (n→∞)
0820132人目の素数さん
垢版 |
2020/08/30(日) 14:00:06.34ID:UYEbc3ly
>>790
これの(2)をどなたかお願いします
何時間か考えましたが解けませんでした
(1)は1/3≦r≦4/3です
0821イナ ◆/7jUdUKiSM
垢版 |
2020/08/30(日) 15:31:40.52ID:upD++ZyF
>>806
>>790(2)
ab(4b-3)/4(a+b)≦x≦ab(3b-1)/(a+b)
a^2(3b-4)/(3ab-4a+4b)≦y≦a^2(3b-1)/(3ab-a+b)
(a+b)+x=ab(b-t)
y(ab-at+bt)=a^2(b-t)
a(a+b)+ax=by(ab-at+bt)
a(a+b)+ax-ab^2y=-by(a-b)t
t={ab^2y-ax-a(a+b)}/by(a-b)
ここまではできた。
領域の形がわからない。
0823132人目の素数さん
垢版 |
2020/08/30(日) 17:44:45.74ID:wS+E/Mk9
>>814
マクローリン展開
 e^{t'} = Σ[s=0,∞] (t'^s) /(s!),
から
 e^{-t'} Σ[s=0,∞] (t'^s) /(s!) = 1,   (ポワソン分布)

 e^{-t'} Σ[s=0,s'] (t'^s) /(s!) = 1 - e^{-t'} Σ[s=s'+1,∞] (t'^s) /(s!)
 = 1 - ∫[t=0,t'] (t^s') e^{-t} dt / (s'!),
0824132人目の素数さん
垢版 |
2020/08/30(日) 18:30:40.16ID:wS+E/Mk9
>>797
をチョト改良。
e = lim(N→∞) (1+1/N)^{N +1/2},
∴ (1+t'/2n)^{2n/t' +1/2} ≒ e,

(1+t'/2n)^n ≒ (1+t'/2n)^{-t'/4} e^{-t'/2}
 ≒ (1 - t't'/8n) e^{-t'/2},

∫[0,∞] (1 +t'/2n)^n e^{-t'} dt'
 ≒ ∫[0,∞] (1 - t't'/8n) e^{-t'/2} dt'
 = [ -{2 - (8+4t'+t't')/4n} e^{-t'/2} ](0→∞)
 = 2(1 - 1/n),     (>>807 を参照)
0825132人目の素数さん
垢版 |
2020/08/30(日) 19:00:24.05ID:2u9lqC6Z
以下の問題の解答がわかりません。スペルナーの補題を利用しているのですが、分からないところがあります。
スペルナーの補題および以下の問題の解答をアップロードすれば一緒に考えていただけますか?

ラスロウ・ロバース他『入門 組合せ論』共立出版 (1985)
(秋山 仁+ピーター・フランクル翻案) 136p.2090円
http://www.kyoritsu-pub.co.jp/bookdetail/9784320013407

多分、難しい問題だと思います。aとcを結ぶ線分およびbとdを結ぶ線分は長方形の対角線になります:

長方形の板があり、その4頂点a, b, c, dに釘が打ってある。aとcは+の釘、bとdは-の釘である。そして、どの3本の釘も一直線上に並ばないようにかってに
+と-の釘を何本でも打ち込み、釘以外で意図が交差することがないように釘の間を糸で結び、もうこれ以上は交差せずには結べないところまで結ぶ(この
とき、長方形の内部の領域はすべて三角形であることに注意せよ)。このとき、+どうしを結ぶ糸だけを通ってaからcまで到達できるか、または-どうしを結ぶ
糸だけを通ってbからdまで到達できる。
0826132人目の素数さん
垢版 |
2020/08/30(日) 19:24:56.49ID:wS+E/Mk9
>>824 (訂正)

(1+t'/2n)^n ≒ (1+t'/2n)^{-t'/4} e^{t'/2}
 ≒ (1 - t't'/8n) e^{t'/2},
これに e^{-t'} を掛ける。
0827132人目の素数さん
垢版 |
2020/08/30(日) 21:27:53.18ID:HbO/9OyJ
>>825
以下二本の+釘を結ぶ辺を+辺、二本の−釘を結ぶ辺を−辺と呼ぶ
+釘と+辺で囲われた領域の内部は全て+釘と+辺としてよい
−についても同様に仮定する
+釘と+辺からなるグラフを+グラフと呼ぶ
−についても同様に−グラフと呼ぶ
ε>0を十分小さくとる
プラスグラフのε近傍を除いて長方形が分断される時は+辺を辿ってaとcが結ばれるから+グラフのε近傍を除いても残った領域は連結である
そこでbからdに至るパスを+グラフのε近傍をb,d以外は釘を通らないものにとる
このパスが通過する三角形の頂点は−釘は高々一つである
そこで各三角形に入る部分と出る部分を止めたままパスをずらして、全てグラフ上で−グラフのε近傍を通らないようにできる
この時得られたグラフは−辺を通らないパスなのでさらに変形してプラス辺しか通らないようにできる
0828132人目の素数さん
垢版 |
2020/08/30(日) 21:29:48.66ID:HbO/9OyJ
>>827

後半修正
+と−逆になった

> このパスが通過する三角形の頂点は+釘は高々一つである
> そこで各三角形に入る部分と出る部分を止めたままパスをずらして、全てグラフ上で+グラフのε近傍を通らないようにできる
> この時得られたグラフは+辺を通らないパスなのでさらに変形して−辺しか通らないようにできる
0829イナ ◆/7jUdUKiSM
垢版 |
2020/08/30(日) 22:18:27.24ID:upD++ZyF
>>822
>>825
acとbdの交点をeとすると、
△cdeの内部に+の釘を打つことでaからcに2本の糸で行ける。
-の釘を打てばbからdに2本の糸で行ける。
かならずしも+も-も打つ必要はなく、
直接aとcを結べば1本の糸で行けるし、
bとdを結べば-どうし1本の糸で結ぶこともできる。
∴示された。
0832132人目の素数さん
垢版 |
2020/08/30(日) 23:10:46.49ID:wS+E/Mk9
>>807
更に蛇足だが・・・・

(1+t'/2n)^n = {1 - (1/8n)t'^2 + (1/24n^2)t'^3 + {(n-2)/(128n^3)}t'^4 - {(5n-6)/(960n^4)}t'^5 - ・・・・} e^{t'/2},
を使えば
∫[0,∞] (1+t'/2n)^n e^{-t'} dt'
 = ∫[0,∞] {1 - (1/8n)t'^2 + (1/24n^2)t'^3 + ・・・・} e^{-t'/2} dt'
 = 2(1 - 1/n + 5/n^2 - 41/n^3 + ・・・・),
0833132人目の素数さん
垢版 |
2020/08/31(月) 00:41:32.75ID:SpqiBffY
>>831
r=4/3のとき断面は1点
1≦r<4/3のとき断面は3角形
2/3<r<1のとき断面は5角形
r=2/3のとき断面は4角形
1/2<r<2/3のとき断面は5角形
1/3<r≦1/2のとき断面は3角形
r=1/3のとき断面は1点
0834132人目の素数さん
垢版 |
2020/08/31(月) 00:53:34.63ID:/Gn5tqYJ
(端を除いて)切り口は相似にならない?
0835132人目の素数さん
垢版 |
2020/08/31(月) 10:14:42.63ID:5D4+y8sX
>>807
指数関数列に展開するなら(ラプラス展開?)
 x = e^{t'/2n} -1 とおいて
1 + t'/2n
 = {1 - (1/2)x^2 + (1/2+1/3)x^3 -(1/2+1/3+1/4)x^4 + ・・・・} e^{-t'/2n},

(1 ⊹ t'/2n)^n
 = {1 - (1/2!)nx^2 + (5/3!)nx^3 + (1/4!)n(3n-29)x^4 - (2/5!)n(25n-102)x^5 - (5/6!)n(n-1)(3n-134)x^6 + (7/7!)n(n-1)(75n-1262)x^7 + ・・・・} e^{-t'/2},

∫[0,∞] x^k e^{-t'/2} dt'
 = ∫[0,∞] (e^{t'/2n} -1)^k e^{-t'/2} dt'
 = 2/C(n-1,k)
 = 2(k!)/{(n-1)・・・・(n-k)},

∫[0,∞] (1 + t'/2n)^n e^{-t'} dt'
 = 2(1 - n/{(n-1)(n-2)} + 5n/{(n-1)(n-2)(n-3)} + n(3n-29)/{(n-1)・・・・(n-4)} - 2n(25n-102)/{(n-1)…(n-5)} - 5n(3n-134)/{(n-2)…(n-6)} + 7n(75n-1262)/{(n-2)・・・・(n-7)} + ・・・・)
 = 2(1 -1/n +5/n^2 -41/n^3 +364/n^4 - ・・・・)
0836132人目の素数さん
垢版 |
2020/08/31(月) 17:34:49.13ID:5D4+y8sX
(訂正)
1 + t'/2n
 = {1 - (1/2)x^2 + (5/6)x^3 -(13/12)x^4 +(77/60)x^5 -(29/20)x^6 -(223/140)x^7 +(481/280)x^8 - ・・・・} e^{-t'/2n},

(1 ⊹ t'/2n)^n
 = {1 - (1/2!)nx^2 + (1/3!)5nx^3 + (1/4!)n(3n-29)x^4 - (1/5!)n(50n-204)x^5 - (1/6!)n(15nn-685n+1714)x^6 + (1/7!)n(525nn-9359n+16862)x^7 + ・・・・} e^{-t'/2},

2(1 -1/n +5/n^2 -41/n^3 +469/n^4 - ・・・・),
0837132人目の素数さん
垢版 |
2020/08/31(月) 17:41:59.20ID:5D4+y8sX
>>832
マクローリン展開の方は
(1+t'/2n)^n
 = {1 - (1/8n)t'^2 + {1/(24n^2)}t'^3 + {(n-2)/(128n^3)}t'^4
- {(5n-6)/(960n^4)}t'^5 - (3nn-26n+24)/(9216n^5)}t'^6
+ {(35nn-154n+120)/(107520n^6)}t'^7 + ・・・・} e^{t'/2},

∫[0,∞] (1+t'/2n)^n e^{-t'} dt'
= 2(1 -1/n +5/n^2 -41/n^3 +469/n^4 - ・・・・),
0838132人目の素数さん
垢版 |
2020/09/01(火) 15:23:30.02ID:fpY624Bn
f(x + y) = f(x) * f(y)
f(1) = 10
を満たす関数はg(x) = 10^x以外に、無数に存在することを示せ。
0839132人目の素数さん
垢版 |
2020/09/01(火) 18:35:21.38ID:wPQGNFIu
>>638
RのQベクトル空間としての基底(vi)_i∈card(R)をv0=1のようにとる
自然数nに対しQ線形写像gn:R→Rをgn(v0)=v0,gn(vi)=n(i>0)ととる
fn(x)=10^gn(x)
は全て条件を満たす
0840132人目の素数さん
垢版 |
2020/09/01(火) 19:12:28.67ID:2qjbTlF5
1230
学コン・宿題ボイコット実行委員会@gakkon_boycott 9月1日
#拡散希望
#みんなで学コン・宿題をボイコットしよう
雑誌「大学への数学」の誌上で毎月開催されている学力コンテスト(学コン)と宿題は、添削が雑で採点ミスが多く、訂正をお願いしても応じてもらえない悪質なコンテストです。(私も7月号の宿題でその被害に遭いました。)このようなコンテストに参加するのは時間と努力の無駄であり、参加する価値はありません。そこで私は、これ以上の被害者を出さないようにするため、また、出版社に反省と改善を促すために、学コン・宿題のボイコットを呼び掛けることにしました。少しでも多くの方がこの活動にご賛同頂き、このツイートを拡散して頂ければ幸いです。
https://twitter.com/gakkon_boycott/status/1300459618326388737
https://twitter.com/5chan_nel (5ch newer account)
0841132人目の素数さん
垢版 |
2020/09/01(火) 19:50:01.76ID:YyZ51xqq
誰か教えてください(´;ω;`)

単一換字暗号で暗号化されている文字列XXYがある。このとき、最頻出文字の推定が失敗する確率を求めなさい。
ただし、平文は英語のアルファベット3文字x, y, zだけで書かれている。尚、各アルファベットの出現する確率は、大きい方から順に0.5, 0.3, 0.2である。
0842132人目の素数さん
垢版 |
2020/09/01(火) 21:15:25.68ID:avnwFqkU
>>841
x0.5y0.3z0.2で
xxyの確率は75/1000
x0.5y0.2z0.3で50/1000
同様に他のは45/1000, 18/1000, 20/1000, 12/1000
これらの内x0.5なのは(75+50)/(75+50+45+18+20+12)=125/230=25/46
0843132人目の素数さん
垢版 |
2020/09/01(火) 21:16:30.91ID:avnwFqkU
失敗だから21/46か
0844132人目の素数さん
垢版 |
2020/09/01(火) 22:47:29.11ID:YyZ51xqq
>>843
ありがとう(´;ω;`)
0845132人目の素数さん
垢版 |
2020/09/02(水) 00:09:32.35ID:8HGOiuSr
>>844
説明は適当だから適当に適切にしてな
0846132人目の素数さん
垢版 |
2020/09/02(水) 01:46:36.51ID:oLFPYAp6
>>829
>>833場合分けか。
分子に来るか、分母に来るか、両方に来るか。
分子、分母が1より大きいか、小さいか。
r=ab/(c+a)=1/3となるのは(a,b,c)=(1,1,2)のとき。
1/3<r<1/2
r=ab/(c+a)=1/2となるのは(a,b,c)=(2,1,2)のとき。
1/2<r<5/9=0.555……
r=ab/(c+a)=5/9となるのは(a,b,c)=(5/3,1,4/3)
5/9<r<1/√3
1/√3=0.5773502……
r=ab/(c+a)=1/√3となるのは(a,b,c)=(√3,2√3/3,√3)のとき。
1/√3<r<3/5=0.6

2/3=0.666……
r=ab/(c+a)=2/3となるのは(a,b,c)=(1,2,2)のとき。
(a,b,c)=(7/5,10/7,8/5)のときもr=2/3
2/3<r<1/√2
r=ab/(c+a)=1/√2となるのは(a,b,c)=(√2,√2,√2)
いくらでも刻めるじゃないか。
場合分けなんか意味あんのか?
0847132人目の素数さん
垢版 |
2020/09/02(水) 05:02:59.82ID:jqJOY8zO
xyz空間の点(x,y,z)は以下を満たしながら動く。
x=ca/(a+b)
y=a+b
z=ab/(b+c)

ただしa,b,cは以下を満たす実数である。
・a,b,cは独立であり、1≦a≦2、1≦b≦2、1≦c≦2の範囲を変化する。

点(x,y,z)が動いて出来る立体の体積を求めよ。
0848132人目の素数さん
垢版 |
2020/09/02(水) 11:20:27.69ID:aS8SLhhP
平面上では存在しない不適切問題とされた問題

AB = 8, BC = 12, ∠B = 60°, ∠C = 40° のとき、
三角形ABCに外接している円の半径を求めよ。

http://suseum.jp/gq/question/3187

こう改題したらどうだろう?

球面上で
AB = 8, BC = 12, ∠B = 60°, ∠C = 40° という三角形ABCが存在するならその球の半径と∠A及びCAの長さを求めよ

という問題なら答があるだろうか?
0850イナ103
垢版 |
2020/09/02(水) 15:17:18.75ID:oLFPYAp6
>>846
>>847
y軸を縦にとるせいか見やすいですね。
0851132人目の素数さん
垢版 |
2020/09/02(水) 15:41:10.87ID:zrYT2aew
小さい方から数えてn番目の素数をP[n]と書く。

(1)素数pで、P[p]+pが素数となるものをすべて求めよ。

(2)素数qで、P[q]+q!が素数となるものは無数に存在することを示せ。
0852132人目の素数さん
垢版 |
2020/09/02(水) 16:05:47.03ID:i2k4QLcN
https://i.imgur.com/CXXqApj.png

上の組合せ論の公式の証明が意味不明です。証明の最初の2文が特によく分かりません。
0853132人目の素数さん
垢版 |
2020/09/02(水) 16:21:31.88ID:i2k4QLcN
最初の2文ですが、以下のようであれば分かるのですが。。。

{1, …, u+v+1}からu+v-m+1個の整数を取り出して、昇順に一列に並べる。
このとき、左からu+1番目の整数はu+k+1(0≦k≦m)と書ける。
0854132人目の素数さん
垢版 |
2020/09/02(水) 16:25:15.26ID:i2k4QLcN
>>852
ラスロウ・ロバース他著(秋山仁+ピーター・フランクル翻案)『入門組合せ論』という本に載っている公式です。
おそらく秋山仁さんが大幅に原著を翻案してほぼ一人で書いていると推測します。
0855132人目の素数さん
垢版 |
2020/09/02(水) 16:26:28.40ID:i2k4QLcN
>>853
公式自体が成り立つことは、以下から簡単に分かります。

{1, …, u+v+1}からu+v-m+1個の整数を取り出して、昇順に一列に並べる。
このとき、左からu+1番目の整数はu+k+1(0≦k≦m)と書ける。
0856132人目の素数さん
垢版 |
2020/09/02(水) 16:29:22.56ID:i2k4QLcN
arcsin(x/sin(x))^log(sin(e^x))を微分せよ。
0859イナ ◆/7jUdUKiSM
垢版 |
2020/09/02(水) 19:11:15.57ID:oLFPYAp6
>>850
>>848
∠A=180°-60°-40°=80°
AからBCに垂線AHを下ろすとその長さは4√3
BH=4,HC=8
ピタゴラスの定理よりAC=√{(4√3)^2+8^2}
=√(48+64)
=√112
=4√7
0860132人目の素数さん
垢版 |
2020/09/02(水) 20:20:13.04ID:i1KE1wMl
確かに
三辺が8:12:4√7のとき外接円半径は(4/3)√21
これ数字の並びが綺麗だけどね

ただこの場合、角度がきれいじゃないんだなあ
0862132人目の素数さん
垢版 |
2020/09/02(水) 20:44:37.11ID:55IWteQv
 iを虚数単位とする。
 互いに素な自然数の組(m,k)が与えられている。
 自然数nに対し、複素数α[n]をα[n]={(m+ki)/|m+ki|}^nにより定義する。

(1)
 任意の素数pに対し、α[p]は実数でないことを証明せよ。

 以下、自然数s,t,uに対してα[s],α[t],α[u]が複素数平面上の三角形の3頂点をなすとき、その面積をS(s,t,u)と表す。

(2)
 相異なる3つの素数の組(p_1,p_2,p_3)を選び、複素数平面上の3点α[p_1],α[p_2],α[p_3]を頂点とする三角形を作る。
 このとき、組(p_1,p_2,p_3)をどのように選んでも、それとは異なる、相異なる3つの素数の組(p_4,p_5,p_6)で、
  S(p_4,p_5,p_6) > S(p_1,p_2,p_3)
となるものが存在することを示せ。
 ただし組(A,B,C)と組(D,E,F)が異なるとは、{A,B,C}≠{D,E,F}であることを指す。

(3)
 S(x,y,z)には上限が存在することを示せ。またその上限を求めよ。
0863イナ ◆/7jUdUKiSM
垢版 |
2020/09/02(水) 21:02:17.21ID:oLFPYAp6
>>859
>>848
8/4√7=0.75592894601……
cos40°=0.76604444311……
わずかだが4√7はACとして長いので弛ませる必要がある。
すなわち△ABCが球体の表面にあればよい。
弧⌒AC=4√7に対する中心角を2θ,
球体の半径をrとすると、
sinθ=4/rcos40°=4θ/(2√7×cos40°)
2θ/sinθ=(cos 40°)√7
r=2√7/θ
一意に決まりそうなんだが。
0868132人目の素数さん
垢版 |
2020/09/03(木) 12:28:07.30ID:WJg5dk4Y
f_n(x)=x^2-nx+1とする。

(1)方程式f_4(x)=0の2解をα,β(α>β)とおく。α^2020の一の位の数字を求めよ。

(2)方程式f_3(x)=0の2解をγ,δ(γ>δ)とおく。γ^2020の一の位の数字を求めよ。
0870132人目の素数さん
垢版 |
2020/09/03(木) 13:36:28.55ID:7p7EW6Y9
>>868
a(t)=α^t+β^tとおいて
a(t)=na(t-1)-a(t-2), a(0)=2, a(1)=n
a(n) の法10での類は多くとも周期10でループ
α^nの1の位≡a(t)-1(mod10)
0873132人目の素数さん
垢版 |
2020/09/03(木) 20:54:11.59ID:PGJ1gE8Y
>>868
 f_n(x) = xx -nx +1 = (x-n/2)^2 - (nn-4)/4,
 2解 {n±√(nn-4)}/2

 f_n(0) = 1,
 f_n(1) = 2-n < 0,
 f_n(n) = 1,
∴ 0 < β(δ) < 1 < α(γ)
0874132人目の素数さん
垢版 |
2020/09/03(木) 21:48:28.44ID:19dRHONM
組み合わせの問題で教えて下さい。

@〜Iの名前のついた箱があります。
一つの箱には最低1以上〜91以下の数字が入ります。

このとき、10個の合計が必ず100になる条件とした場の組み合わせを
求める方法はあるでしょうか。


    パターン名
     ↓
箱の名前 A   B    C    X
 @  91  90   20    …
 A   1   2   10    …
 B   1   1    2    …
 C   1   1    5    …
 D   1   1   13    …
 E   1   1   22    …
 F   1   1    7    …
 G   1   1    8    …
 H   1   1   14    …
 I   1   1    4    …
−−−−−−−−−−−−−−−−−−−−−−−−−−−−−−
合計 100 100  100  100  合計は必ず100
                      最低は必ず1
同種の質問がありましたら、ここを見ろでもOKです。

最終的な何パターンあるが計算で求められれば良いですが、
実際にはプログラムで全パターンを回してみたいので、その方法も
わかれば嬉しいです。

何卒、よろしくおねがいします。
0875132人目の素数さん
垢版 |
2020/09/03(木) 22:02:21.36ID:QHxsSkOO
>>874

Binomial(89, 9)
0877132人目の素数さん
垢版 |
2020/09/03(木) 22:06:20.20ID:QHxsSkOO
>>874

Binomial(100-1, 10-1) = 1731030945644
0878132人目の素数さん
垢版 |
2020/09/03(木) 22:20:31.89ID:QHxsSkOO
>>876
なぜBinomial(n + k - 1, k - 1)のことを重複組合せというんですか?
0879132人目の素数さん
垢版 |
2020/09/03(木) 22:24:22.77ID:QHxsSkOO
普通の組合せとは何の関係もないですよね?重複組合せと書くと何か普通の組合せの類似物のように錯覚しますが。
0880132人目の素数さん
垢版 |
2020/09/03(木) 22:52:40.33ID:FNYVyrwP
>>879
1個ずつ玉の入った名前付きの箱を10個用意して、その箱にあと90個の玉を入れるのと同じことだから、
10個の箱から重複を許して90回選ぶ選び方と同じ
なので重複組合せ
0882132人目の素数さん
垢版 |
2020/09/04(金) 06:57:46.73ID:a8/P402N
>874をWolframに数えてもらおうと
 Coefficient[Sum[x^k, {k, 1, 90}]^10, x^100]
と入れたけどエラーが返ってきた。

箱が9個の時は計算してくれたんだが、
Coefficient[Sum[x^k, {k, 1, 90}]^9, x^100]
171200862675
0884132人目の素数さん
垢版 |
2020/09/04(金) 08:31:49.03ID:gPAmKfac
>>874
列挙プログラムを書いてみた。

f <- function(Box,Sum){ # Box:箱の数,Sum:入った数字の合計の値
# 重複を許してn個からr個を選ぶ組み合わせを列挙する
H <- function(n, r, v=1:n) {
if (r == 0)
NULL
else if (r == 1)
matrix(v, n, 1)
else if (n == 1)
matrix(v, 1, r)
else rbind(cbind(v[1], H(n, r - 1, v)), H(n - 1, r, v[-1]))
}
h=H(Box,Sum-Box) # Box個の中から重複を許してSum-Box個を選ぶ
nh=nrow(h) # その組み合わせ数
re=matrix(rep(NA,Box*nh),nrow=nh,ncol=Box) # メモリ確保
a=numeric(Box)
for(i in 1:nh){ # 各々の選び方iに対して
for(j in 1:Box) a[j]=sum(j==h[i,]) # どの箱が何個選択されたかを
re[i,] =a # reのi行に入れる
}
return(re+1) # 1から始まる分を補正
}
箱3個合計9の場合
> f(Box=3,Sum=9)
[,1] [,2] [,3]
[1,] 7 1 1
[2,] 6 2 1
[3,] 6 1 2
[4,] 5 3 1
[5,] 5 2 2
[6,] 5 1 3
[7,] 4 4 1
[8,] 4 3 2
[9,] 4 2 3
[10,] 4 1 4
[11,] 3 5 1
[12,] 3 4 2
[13,] 3 3 3
[14,] 3 2 4
[15,] 3 1 5
[16,] 2 6 1
[17,] 2 5 2
[18,] 2 4 3
[19,] 2 3 4
[20,] 2 2 5
[21,] 2 1 6
[22,] 1 7 1
[23,] 1 6 2
[24,] 1 5 3
[25,] 1 4 4
[26,] 1 3 5
[27,] 1 2 6
[28,] 1 1 7
0886132人目の素数さん
垢版 |
2020/09/04(金) 17:51:15.59ID:4rR4gXNg
>>848
 O (0, 0, 0)
 A (r cos(8/r), r sin(8/r)cos(B), r sin(8/r)sin(B))
 B (r, 0, 0)
 C (r cos(12/r), r sin(12/r), 0)
 ∠B = 60°
とおく。
∠C は平面OACと平面OBCの二面角。
平面OACの法線ヴェクトル ↑OA×↑OC の
 xy-平面成分は rr sin(8/r) sin(B),
 z成分は rr [sin(12/r)cos(8/r) - cos(12/r)・sin(8/r)cos(B)],
一方、平面OBCはxy-平面で、法線ヴェクトルは (0,0,1)
これらが∠Cをなすことから、↑OA×↑OC の成分比
 sin(8/r) sin(B):[sin(12/r)cos(8/r) - cos(12/r)・sin(8/r)cos(B)]
 = sin(C):cos(C),
 = ±sin(40°):cos(40°),
 CA = r arccos[cos(12/r)cos(8/r) + sin(12/r)・sin(8/r)cos(B)],

C = +40° から
 r = 3.27225023635408
 CA = 3.4176589202447

C = -40° から
 r = 2.08421016388903
 CA = 4.0289210164475
0887132人目の素数さん
垢版 |
2020/09/04(金) 17:58:47.73ID:43QR7q9z
(グラフ理論の)グラフが同型でないことを示す一般的な方法ってありますか?
0888132人目の素数さん
垢版 |
2020/09/04(金) 18:04:18.16ID:CIoeKwyr
>>887
しらみつぶしぐらいジャね?
0889132人目の素数さん
垢版 |
2020/09/04(金) 18:39:47.74ID:hSfz/m5F
複素平面上の点P(z)とPを通る定円Cがあるとき、PをC上の別の点に移すある変換fを考えます。
変換fを施すことを→で表します。
P→P_1→P_2→...と、PとPが移った点にfを施し続けるとき、任意のP_iがC上にあって、どのP_iも全て異なるようなfは一般にどのような形をしているかご教授ください。
0890イナ ◆/7jUdUKiSM
垢版 |
2020/09/04(金) 21:08:38.08ID:SjJnJuPh
>>869
>>886
AC=4.0と3.4の2つ出たの、すごいね。
楕円球にするしかないと思とった。
0891874
垢版 |
2020/09/04(金) 21:17:22.72ID:JgMbxafq
874です。

みなさん、コメント・解答ありがとうございました。
なるほど、重複組み合わせというのですね。

Webでも調べてみて理解を深めることができました。
ありがとうございます。

>> 884 様
プログラムありがとうございます。
さらっとこんなプログラムを書ける、羨ましいです。
使われてる関数などで調べましたが、R言語なんでょうか。
https://cran.ism.ac.jp/ からWいwindows版をダウンロードして
インストしてみましたが、未だ動作に至らず....

もしよろしければ、動作方法など教えていただければ幸いです。

皆様、改めましてありがとうございました。m(_ _)m
0892132人目の素数さん
垢版 |
2020/09/04(金) 21:32:45.62ID:HUputvkD
>>889
Cの任意可算無限集合の任意順序付けをP_iとし
P_iはP_(i+1)に移し、それ以外の点は任意の変換
0893132人目の素数さん
垢版 |
2020/09/04(金) 21:59:36.05ID:huVAl7qX
>>889
正則関数だと例えば
z₀ : 円Cの中心点、α : 任意の無理数として
 f(z) = z₀ + e^{i2πα} (z - z₀)
他のタイプの解があるのかは分からん。
非正則を含めた一般解となると >>892 みたいになる。
0894132人目の素数さん
垢版 |
2020/09/05(土) 00:44:18.59ID:r6wBA3+u
>>891
言語はRです。
ここで実行できます。
https://www.tutorialspoint.com/execute_r_online.php
日本語のコメントのままでも実行できました。
箱3個、合計数9でしかやってませんが。

箱10個、合計数100はメモリ不足で実行できないと思います。
重複組み合わせの列挙に再帰関数を使っているのでネストが深すぎてエラーになると思います。
0895132人目の素数さん
垢版 |
2020/09/05(土) 06:35:50.35ID:HlUk1qZS
実数θが0≦θ≦π/4を動くとき、極座標系においてθ|sin(8πθ)|≦r≦θで表される領域の面積を求めよ。
0896132人目の素数さん
垢版 |
2020/09/05(土) 06:44:40.90ID:dkJd7U2T
以下の条件を満たす自然数mを全て決定せよ。

【条件】
0≦n≦mを満たす全ての整数nについて、二項係数(m,n)は奇数となる。
0898132人目の素数さん
垢版 |
2020/09/05(土) 09:47:46.51ID:xcauWUrR
一般化して
パスカルの三角形で横一列の全てがpの倍数でない段はm=ap^k-1(1≦a≦p-1)段目だな
このようなmはp進表示で最高桁以外でp-1の並ぶ数で、和がmになるどのような2つの数n,(m-n)に分けてもその足し算で繰り上がりが発生しない数にとして特徴付けられる
こういう性質と二項係数の関係は前スレでも出た
https://rio2016.5ch.net/test/read.cgi/math/1594131967/460
0900132人目の素数さん
垢版 |
2020/09/05(土) 12:52:12.18ID:sjSgt5Lc
>>895
S = (1/2)∫θ^2 dθ - (1/2)∫{θ sin(8πθ)}^2 dθ
 = (1/2)∫θ^2 cos(8πθ)^2 dθ
 = (1/4)∫θ^2 {1+cos(16πθ)} dθ
 = (1/4)∫θ^2 dθ + (1/4)∫θ^2・cos(16πθ) dθ
 = [ (1/12)θ^3 + θ^2・sin(16πθ)/(64π) ] - ∫θ・sin(16πθ)/(32π) dθ
 = [ (1/12)θ^3 + θ^2・sin(16πθ)/(64π) + θ・cos(16πθ)/(512π^2) ]
  - ∫cos(16πθ)/(512π^2) dθ
 = [ (1/12)θ^3 + θ^2・sin(16πθ)/(64π) + θ・cos(16πθ)/(512π^2)
  - sin(16πθ)/(8192π^3) ](θ=0,π/4)
 = (π^3)/768 + π・sin(4π^2)/1024 + cos(4π^2)/(2048π) - sin(4π^2)/(8192π^3)
 = 0.04333824242
0901132人目の素数さん
垢版 |
2020/09/05(土) 15:16:16.70ID:vQ+eQauY
 iを虚数単位とする。
 互いに素な自然数の組(m,k)が与えられている。
 自然数nに対し、複素数α[n]をα[n]={(m+ki)/|m+ki|}^nにより定義する。

(1)
 任意の素数pに対し、α[p]は実数でないことを証明せよ。

 以下、素数p_i,p_j,p_kに対してα[p_i],α[p_j],α[p_k]が複素数平面上の三角形の3頂点をなすとき、その面積をS(p_i,p_j,p_k)と表す。

(2)
 相異なる3つの素数の組(p_1,p_2,p_3)を選び、複素数平面上の3点α[p_1],α[p_2],α[p_3]を頂点とする三角形を作る。
 このとき、組(p_1,p_2,p_3)をどのように選んでも、それとは異なる、相異なる3つの素数の組(p_4,p_5,p_6)で、
  S(p_4,p_5,p_6) > S(p_1,p_2,p_3)
となるものが存在することを示せ。
 ただし組(A,B,C)と組(D,E,F)が異なるとは、{A,B,C}≠{D,E,F}であることを指す。

(3)
 S(p_i,p_j,p_k)には上限が存在し、その上限値が(3√3)/4であることを示せ。
0902132人目の素数さん
垢版 |
2020/09/05(土) 15:53:54.67ID:8Trxkoe5
8.33%の確率のものを17回当てたくて1回当たり30000コイン必要な場合、コイン何枚必要か式と答えを教えてくださいm(_ _)m
0903132人目の素数さん
垢版 |
2020/09/05(土) 16:23:32.93ID:Air1TPt6
>>902
必ず17回当たるということなら無限に必要
当たり回数の期待値が17回を超えるのは17÷0.0833=204.08……なので205回以上だから615万コイン必要
ただし、205回抽選した場合18回以上当選する場合がある代わりに16回以下しか当選しない場合もある
0904132人目の素数さん
垢版 |
2020/09/05(土) 16:38:49.29ID:liKukSbn
>>902

>>903の訂正

必ず17回当たるということなら無限に必要
当たり回数の期待値が17回を超えるのは17÷0.0833÷2=102.04……なので103回以上だから309万コイン必要
0906132人目の素数さん
垢版 |
2020/09/05(土) 16:45:03.05ID:r6wBA3+u
>>902
負の二項分布
ここでは成功確率が0.083である事象が17回成功するまでの失敗の数の分布

p=0.083
s=17 # 成功数
s*(1-p)/p # 成功数sになるまでの失敗数の期待値 負の二項分布の期待値の公式

s*(1-p)/p + s = s/p # 成功数と失敗数の合計

s/p*30000 = 6144578
0907132人目の素数さん
垢版 |
2020/09/05(土) 17:10:36.90ID:r6wBA3+u
>>905
0.0833だったので、17回成功するまでに必要なコインの期待値は

p=0.0833
s=17 # 成功数
s*(1-p)/p # 成功数sになるまでの失敗数の期待値
s*(1-p)/p + s # = s/p 成功数と失敗数の合計
s/p
s/p*30000 # 6122449
0908132人目の素数さん
垢版 |
2020/09/05(土) 17:21:16.20ID:r6wBA3+u
10万回シミュレーションして平均値(期待値の近似値)を出してみた

sim <- function(p=0.0833,s=17){
Su=0 # 成功数
i=0 # 試行数
while(Su<17){
Su=Su+rbinom(1,1,p)
i=i+1
}
return(i)
}
mean(replicate(1e5,sim()))*30000

> mean(replicate(1e5,sim()))*30000
[1] 6117982
0910132人目の素数さん
垢版 |
2020/09/05(土) 19:13:46.27ID:sjSgt5Lc
17回成功に到達するのがn回目の抽選である確率は
 q_n = C(n-1,17-1)・p^17・(1-p)^(n-17) (n≧17)
   = 0   (n<17)
Σ[n=17,∞] q_n = 1,
E[n] = Σ[n=17,∞] n・q_n = 17/p = 204.08163265

Σ[n=17,199] q_n = 0.4935182680
Σ[n=17,200] q_n = 0.5020417085
Σ[n=17,204] q_n = 0.5357946506
Σ[n=17,205] q_n = 0.5441305558

median 〜 199.76
200回抽選するとして 600万コイン必要
0911132人目の素数さん
垢版 |
2020/09/05(土) 19:18:57.28ID:BtVdvkls
定理4.4.2の証明中の「つまり、どの辺も1度だけ使われる。」の言っていることが分かりません。
解説をお願いします。

定理4.4.1
木のどの2点もちょうど1本の道で連結している。

定理4.4.2
どんな位数nの木もn-1本の辺をもつ。
証明:
電話のネットワークを例にとって証明しよう。ある町で事件が起こり、他の町にメッセージを電話で送ろうとしたとする。
まず、その町の人は直接回線がつながっている町で電話する。電話を受けた町は直接つながっている町へ電話する。
電話を受けた町は、直接つながっている町でまだ電話を受けていない町へ電話する。…、グラフは連結なのでメッセージは
どの町へも伝わる。定理4.4.1より、どの町も事件のあった町とは1通りの道で結ばれている。つまり、どの辺も1度だけ
使われる。したがって、電話の回数は辺の本数と1対1に対応する。電話をかけないときに事件を知っている町はその町1つ
だけで、1回電話するたびに事件を知る町が1つずつ増える。したがって、点(町)の個数は辺(交信)の本数よりもちょう
ど1つ多い。
0913132人目の素数さん
垢版 |
2020/09/05(土) 20:07:20.57ID:lbP0o9nI
>>901
それホントに答え持ってんの?
答えない自作問題かどうかわからないと考える気にならん
0914132人目の素数さん
垢版 |
2020/09/05(土) 20:41:15.76ID:BtVdvkls
>>911
ちなみに、英語で出版されている同著者らによる本の該当箇所には以下のように書かれています:

Theorem 8.2.3 Every tree on n nodes has n - 1 edges.

8.2.2 Let G be a tree, which we consider as the network of roads in a medieval
country, with castles as nodes. The king lives at node r. On a certain day, the
lord of each castle sets out to visit the king. Argue carefully that soon after they
have left their castles, there will be exactly one lord on each edge. Give a proof
of Theorem 8.2.3 based on this.

8.2.2. Any edge has only one lord, since if there were two, they would
have to start from different ends, and they would have then two ways to
get to the king: either continuing as they started, or waiting for the other
and walking together. Similarly, an edge with no lord would have to lead
to two different ways of walking.
0915132人目の素数さん
垢版 |
2020/09/05(土) 20:42:15.03ID:BtVdvkls
>>911
は秋山仁・ピーター・フランクルによるハンガリー語からの翻訳です。
0916132人目の素数さん
垢版 |
2020/09/05(土) 20:49:22.46ID:BtVdvkls
>>911
ルートノードRから各ノード(例えばAとする)への一意的な道R→…→B→AとB→Aが一対一に対応するということを言おうとしているようにもかすかながら思えますが、
何を言っているのか正確には分かりません。
0917132人目の素数さん
垢版 |
2020/09/05(土) 20:53:02.21ID:sjSgt5Lc
>>910
スターリングの公式で
log(q_n) = log(C(n-1,16)) + 17・log(p) + (n-17)・log(1-p)
 = log(1-p)・n + 16・log(n) + 17・log(p/(1-p)) - log(16!) - 136/n - 748/nn - ・・・・
 = log(1-p)・n + 16・log(n) - 71.4435 - 136/n - 748/nn - 18496/(3n^3) - ・・・・
nで微分すると
 (d/dn){log(q_n)} = log(1-p) + 16/n + 136/nn +1496/n^3 + 18496/n^4 + ・・・・

 n = 192.576 で最大(ピーク)となる。 q_n = 0.00863234
0918132人目の素数さん
垢版 |
2020/09/05(土) 21:47:12.18ID:Wa2CzuyX
>>913
当然、どの小問も結論は正しいです。
(2)が難しくて分かりません
(3)は正三角形に限りなく近い例を構成すれば良いので(2)が解決すれば難しくはないと思います
(1)は素数乗なので何とかなりそうです
0919132人目の素数さん
垢版 |
2020/09/05(土) 22:01:13.65ID:BtVdvkls
Rを木Tの任意のノードとする。

RからRを除くn-1個の各ノードへは一意的な道が存在する。

AをRとは異なる任意のノードとする。

R→…→A''→A'→Aという一意的な道が存在する。Aに辺A'-Aを対応させる写像φ : V(T) - {R} → E(T)を考える。
φは単射である。なぜなら、仮に、φ(A) = φ(B)となるような異なる2点A, Bが存在したとすると、
B = A'でなければならないが、道R→…→A''→A'→AのR→…→A''→A'がRからA' = Bへの一意的な道で
あるからφ(B) = φ(A') = A''-A' ≠ A'-A = φ(B)となって矛盾が発生するからである。
φは全射である。仮に、φ(A) = A'-AとなるようなノードA∈V(T) - {R}が存在しないような辺A'-Aが存在したとする。
RからA'、RからAへの一意的な道がそれぞれ存在する。これらの道には辺A'-Aは含まれていないことは明らかである。
RからA'への道、辺A'-A、AからRへの道を考えれば明らかなように、Tに閉路が存在することになってしまうが、これは
矛盾である。∴φは全射である。以上より、n-1 = #(V(T) - {R}) = #E(T)である。
0921132人目の素数さん
垢版 |
2020/09/05(土) 22:07:00.70ID:BtVdvkls
>>911は多分>>919のようなことを言いたいのだろうと推測しましたが、どなたか>>911の文章を解読できる方いますか?
0922132人目の素数さん
垢版 |
2020/09/06(日) 00:10:11.22ID:nVWP7zTC
>>916
どのnodeにも領主の城があり(一つだけ領主ではなく王の城がある)
ある時一斉に、しかも同時に領主達が王の城を目指して旅を始める瞬間を考えよ。
動き始めた瞬間、領主達が進む王の城に向かう道の上にいる領主の人数はキッチリ一人であり
また領主がいない道は存在しない。
だから道の数と王以外の領主の数は一致する。

一度だけ使われる、というのはそういう意味。
0923132人目の素数さん
垢版 |
2020/09/06(日) 05:55:19.05ID:Tact+p1h
>>902
応用問題

成功確率8.33%のベルヌーイ試行を17回成功させるのに必要な試行回数の95%信頼区間は何回から何回か?
0924132人目の素数さん
垢版 |
2020/09/06(日) 09:27:29.72ID:tN16f4S+
> # 分位数でのCI
> qnbinom(0.025,17,0.0833)+17 ; qnbinom(0.975,17,0.0833)+17
[1] 123
[1] 307
> # Highest Probability Density IntervalでのCI
> HDInterval::hdi(qnbinom,size=17,prob=0.0833)+17
lower upper
115 297
attr(,"credMass")
[1] 0.95
0925132人目の素数さん
垢版 |
2020/09/06(日) 09:31:01.55ID:tN16f4S+
>>924
10万回のシミュレーション結果と照合

> sim <- function(p=0.0833,s=17){
+ Su=0 # 成功数
+ i=0 # 試行数
+ while(Su<17){
+ Su=Su+rbinom(1,1,p)
+ i=i+1
+ }
+ return(i)
+ }
> re=replicate(1e5,sim())
> quantile(re,c(0.025,0.975))
2.5% 97.5%
122 308
> HDInterval::hdi(re)
lower upper
115 297
attr(,"credMass")
[1] 0.95
0927132人目の素数さん
垢版 |
2020/09/06(日) 14:53:42.83ID:HYDaJwjZ
>>901
(1)
p=2のとき
 α[2] = {(m+ki)/|m+ki|}^2 = {(mm-kk)+(2mk)i}/|m+ki|^2,
 題意より (m,k)は自然数だから mk≧1,
 α[2] は実数でない。
pが奇数のとき
 Re{ (m+ki)^p } = mΣ[j=0,(p-1)/2] C(p,2j) m^(p-1-2j) (-kk)^j
は mの倍数。
一方 題意より (m,k) は互いに素だから
 |m+ki|^{2p} = (mm+kk)^p ≡ k^{2p} ≠ 0 (mod m)
∴ |m+ki|^p はmの倍数ではない。
したがって
 α[p] = {(m+ki)/|m+ki|}^p ≠ ±1.
0928132人目の素数さん
垢版 |
2020/09/06(日) 19:26:53.94ID:HYDaJwjZ
>>923
n = 122.025 までに17回達成する確率が 2.5%
n = 306.839 まで17回未達の確率が 2.5%
∴ 求める区間は
 122.025 < n < 306.839

(参考)
  18/p = 216.086
相加平均 214.432
E[n] = 17/p = 204.816  (p=0.0833)
メジアン 199.76
相乗平均 193.499
  16/p = 192.077

Σ[n=17,121] q_n = 0.02322393181
Σ[n=17,122] q_n = 0.02495533764
Σ[n=17,123] q_n = 0.02678209166

Σ[n=17,306] q_n = 0.97420660647
Σ[n=17,307] q_n = 0.97514963127
Σ[n=17,308] q_n = 0.97606163314
0930132人目の素数さん
垢版 |
2020/09/07(月) 01:58:33.58ID:X63VHU9J
>>930
あ、ダメだ
間違ってる
予想としてはワイルの一様分布定理の素数版

αを無理数、0<a<b<1とするとき
lim #{ p ≦ x | p/α - [p/α] ∈ (a,b)}/(x/log(x)) = b-a

が成立しそうな気はするんだけど
それには

Σ[p≦x] exp(2πi/α p)/p = o(x/log(x))

が示せれば十分
自分が示せたと思ったのは

Σ[p≦x] exp(2πi/α log(p))/p = o(x/log(x))

だった
0931132人目の素数さん
垢版 |
2020/09/07(月) 02:03:56.04ID:X63VHU9J
訂正

Σ[p≦x] exp(2πi/α p) = o(x/log(x))

が示せれば十分
自分が示せたと思ったのは

Σ[p≦x] exp(2πi/α log(p)) = o(x/log(x))

まぁ要するに対数ζのs=1+2πi/αのところだけどlog(p)といういらん因子がある
0932132人目の素数さん
垢版 |
2020/09/07(月) 02:10:14.10ID:I+kGemqZ
あ、イヤ、いいのかな?
いわば「log(p)の“modα”の類」が一様に分布するからどのみち無限にあるからいいのか
0933132人目の素数さん
垢版 |
2020/09/07(月) 07:15:28.29ID:x5YdqZiz
行列Aを以下で定める。
[a b]
[c d]
a^2+c^2=1,b^2+d^2=1
この行列Aと列ベクトルv=(x,y)^Tによる連立方程式Av=0が|x|≦1かつ|y|≦1の実数解を持つとき、自然数nに対してA^nをnで表せ。

(※記号『^T』で行ベクトル(a,b)の転置を表す)
0934132人目の素数さん
垢版 |
2020/09/07(月) 07:24:10.99ID:5r3avP6+
vの条件が謎だな
常に(x,y)=(0,0)を解に持つし、それを除いて考えても解の定数倍も解だから常に絶対値を1以下に出来るのでは
0935132人目の素数さん
垢版 |
2020/09/07(月) 08:32:01.11ID:mXO8E5CX
メネラウスの定理とチェバの定理は同値な双対定理とみなせないのですか?
0937132人目の素数さん
垢版 |
2020/09/07(月) 10:38:45.91ID:c1fkzuff
高卒で数学Vまでなら大体の入試問題を解けます
今から1年でどれくらいの数学を身につけることが可能ですか
土日に8時間、平日3時間を予定しています
初等微積分、線形代数、微分方程式、複素関数論、確率統計、を考えています
0938132人目の素数さん
垢版 |
2020/09/07(月) 12:38:30.89ID:ejy1pQjv
入試用勉強の悪影響からどれだけ抜けられるかで決まる
小学生の素直さがあれば楽勝なんだけどな
0939132人目の素数さん
垢版 |
2020/09/07(月) 13:25:50.73ID:bE/6WhUJ
>>937
そんな無駄な質問する前に勉強したら?
いやならやめれば?
0940132人目の素数さん
垢版 |
2020/09/07(月) 14:57:12.89ID:Lx7mxSXy
グラフGには2つの異なる閉路が存在していて、そのどちらの閉路も辺aを含むとする。
また一方の閉路は辺bを含むが他方の閉路は辺bを含まないとする。
このとき、グラフGから辺bを除去したグラフG'には閉路が存在することを証明せよ。
0941132人目の素数さん
垢版 |
2020/09/07(月) 15:03:54.05ID:Lx7mxSXy
>>940
間違えました。訂正します。

グラフGには2つの異なる閉路が存在していて、そのどちらの閉路も辺aを含むとする。
また一方の閉路は辺bを含むが他方の閉路は辺bを含まないとする。
このとき、グラフGから辺aを除去したグラフG'には閉路が存在することを証明せよ。
0942132人目の素数さん
垢版 |
2020/09/07(月) 15:41:00.01ID:/YNSU6EH
>>941
Gからbの内部を抜いたグラフをG"としてMayer-Vietoris列

0→H1(pt∪pt)→H1(G")+H1(b)→H1G)
. →H0(pt∪pt)→H0(G")+H0(b)→H0(G)

からβ1(G)=β1((G")+1でG"は少なくともひとつの閉路を持つからβ1(G")≧1
∴ β1(G)≧2
同様にしてβ1(G)=β1(G')+1であるからβ1(G')≧1
0943132人目の素数さん
垢版 |
2020/09/07(月) 15:41:03.65ID:c1fkzuff
>>939
えっ
何かお気に障りましたか?
そのレスは時間の無駄ですよね(笑)
勉強法の質問はダメでしたか
0944132人目の素数さん
垢版 |
2020/09/07(月) 19:24:53.45ID:aYAtIge6
>>937
大学の講義は、1単位あたり45時間の学習が目安である。建前かもしれないが。なので大体1.5週間で1単位の勉強量。
大学のカリキュラムを参考に、単位数を、初等微積分4、線形代数4、微分方程式2、複素関数論2、確率統計2とすると、14単位なので21週間。
演習も考えるともっと時間かかるし、サクサク勉強できれば短縮出来る。
0945132人目の素数さん
垢版 |
2020/09/07(月) 19:28:02.41ID:0mPqgBlS
>>933
Cayley-Hamilton の定理より
 AA - (a+d)A + |A|E = O, 
ここに |A| = ad - bc,
∴ n≧1 のとき
 A^n = t_n A - t_{n-1}|A|E,
ここに
 t_0 = 0
 t_1 = 1,
 t_2 = a+d,
 ・・・・
漸化式
 t_{n+1} = (a+d)t_n - |A|t_{n-1},
0947132人目の素数さん
垢版 |
2020/09/08(火) 09:57:50.36ID:uFw/N5vZ
位数5の完全グラフK_5が平面的なグラフではないことの証明ですが、以下のように考えました。
模範解答と違うのですが、どこかおかしいところはありますか?

K_5が平面に辺が交差することなく描けたと仮定する。
n = 5, e = 10であるから、オイラーの公式より領域の数f = e - 5 + 2 = 7である。
一方、一番外側の領域を除く他の領域は三角形のはずであるから、Binomial(5, 3) = 10個の3角形領域のうち、少なくとも9個は
一番外側の領域ではない。f = 7 < 9だからこれは矛盾である。
0948132人目の素数さん
垢版 |
2020/09/08(火) 10:07:31.90ID:kbnSLIZb
>>945
 特性根をα≠βとおくと
 t_n = (α^n - β^n)/(α-β),

 これは αとβの対称多項式だから、
 基本対称式 α+β=a+d, αβ=|A| の多項式。
0949132人目の素数さん
垢版 |
2020/09/08(火) 10:45:22.40ID:uFw/N5vZ
「位数vの平面グラフは最大何本の辺をもつことができるか?」という問題の答えに以下のような記述があります。
「Gを平面性を保つ範囲では、これ以上辺を加えることができないグラフとする。このとき、Gのどの領域も3辺だけで囲まれている」
Gの一番外側の領域も3辺で囲まれていることはどうやって分かるのでしょうか?
0950132人目の素数さん
垢版 |
2020/09/08(火) 11:18:25.92ID:kbnSLIZb
>>917
ピーク付近では
 μ = 192.57637716855732
 σ = 45.975784
 q_max = 0.008632336568

log(q_n) = log(q_max) - (n-μ)^2 /(2σ^2) + 8.583E-7・(n-μ)^3 - 4E-9・(n-μ)^4 + …

>>928
 122.025 = μ - 1.75204σ
 306.839 = μ + 2.48528σ
かなり非対称である。対数正規分布に近いのかも。
0952132人目の素数さん
垢版 |
2020/09/08(火) 12:59:09.60ID:kbnSLIZb
ピーク位置  n=192.576 >>917  と
95%CIの下限・上限の相乗平均 n=193.499 >>928
が近いことから、対数正規分布に近いと推測される。

 μ_g = log(μ) = 5.2604928395496
 σ_g = 0.2387405
 q_max = 0.008632336568

log(q_n) = log(q_max) - {log(n/μ)}^2 /{2(σ_g)^2}
  - 2.64254{log(n/μ)}^3 - 0.697889{log(n/μ)}^4
  - 0.139578{log(n/μ)}^5 - …
95%CIの下限・上限は
 log(122.025) = μ_g - 1.91114σ_g,
 log(306.839) = μ_g + 1.95120σ_g,
これは正規分布の場合 (μ±1.960σ) に近い。

対数正規分布を仮定したときの 95% CI は
 μ_g - 1.960σ_g = log(120.611)
 μ_g + 1.960σ_g = log(307.481)
0953132人目の素数さん
垢版 |
2020/09/08(火) 14:08:39.26ID:xIu482fc
>>947
グラフの周長が3のサイクルが三角形の領域を作るとは限らない
ex K5から一辺除いたグラフは球面グラフであるが、周長3のサイクルは7個あるが、平面グラフの三角形領域は6個しかできない
0954132人目の素数さん
垢版 |
2020/09/08(火) 17:50:18.73ID:UIkXrCax
n次元空間に原点Oを置き、n個のベクトル
↑OA_1=(1,0,0,...,0,0)
↑OA_2=(0,1,0,...,0,0)
...
↑OA_n=(0,0,0,...,0,1)
によって張られる直交座標系(x_1,x_2,...,x_n)を考えます。(数学的に正しくない表現ですいません)
このとき原点からのユークリッド距離がちょうどr(r>0)である点の集合は
(x_1)^2+(x_2)^2+...+(x_n)^2=r^2…(ア)
で表される全体だと思うのですが、

@(ア)で正しいでしょうか
A(ア)はなんと表現すればいいでしょうか。超曲面、超立体、言い方が分かりません
B(ア)をrとn-1個の角で極座標表示することは可能でしょうか

よろしくお願いします
0956132人目の素数さん
垢版 |
2020/09/08(火) 20:38:11.64ID:P6Fyzolp
極座標は
x_1 = r cosθ_1, x_2 = r sinθ_1 cosθ_2, x_3 = r sinθ_1 sinθ_2 cosθ_3, …
x_(n-1) = r sinθ_1 sinθ_2 … sinθ_(n-2) cosθ_(n-1)
x_n = r sinθ_1 sinθ_2 … sinθ_(n-2) sinθ_(n-1)
とすりゃいいのさ
0957132人目の素数さん
垢版 |
2020/09/08(火) 20:54:12.40ID:P6Fyzolp
極座標の解説:
まず x_1 座標と残りの n-1 次元超平面を考えて x_1 座標とベクトルの角をθ_1とすれば
x_1 成分は r cosθ_1 で n-1 次元成分は r sinθ_1
同様に x_2 座標と残りの n-2 次元を考えて… と言う感じに角θを定義して行く
0958132人目の素数さん
垢版 |
2020/09/09(水) 01:02:25.72ID:yGOW4YIU
最近どつかのスレでπ(x;4,1)とπ(x;4,3) (4で割って1余るx以下の素数の数と3のそれ)で増大速度に差があるって話がでてた記憶あるんですけどどこでしたっけ?
0959132人目の素数さん
垢版 |
2020/09/09(水) 01:28:58.83ID:YDbS9Hgz
あんまり超は使わないなあ
n次元多様体とか
n次元球面とか
n次元立方体とか
0960132人目の素数さん
垢版 |
2020/09/09(水) 06:03:09.03ID:hDTCHuTp
実数xについての関数f_[k](x)を、
f_[1](x) = x^2+x
f_[n+1](x)=log{f_[n](e^x)}
と帰納的に定義する。
このとき極限 lim[n→+∞] f_[n](0) および lim[n→+∞] f_[n](1) を求めよ。
0963132人目の素数さん
垢版 |
2020/09/09(水) 16:03:10.42ID:9PTvu2Ea
f_1(x) = x^3 - 3x
f_(n+1)(x) = {f_n(x)}^3 - 3{f_n(x)}
とする。
nを3以上の整数とするとき、xの方程式f_n(x)=0の実数解の個数をnで表せ。
0964132人目の素数さん
垢版 |
2020/09/09(水) 18:13:32.20ID:ayHPVyw3
>>963
実解の個数を a[n] とする。
y=x^3 - 3x と y=x のグラフを描くと
有限回の繰り返し写像で0になりうる点は -2 < x < +2 かつ x≠±1 の範囲に存在していて、
この範囲にある1点の逆像は 重複無しの3点となる事が分かる。
よって 0点の逆像を n 回繰り返し求めれば...
a[n] = 1 * 3 * ... * 3 = 3^n となる。
0965132人目の素数さん
垢版 |
2020/09/09(水) 18:22:09.85ID:ayHPVyw3
こんなのグラフを描けば分かるっしょ?
これを教科書的な厳密さで示すのは面倒かもしれない。
0966132人目の素数さん
垢版 |
2020/09/09(水) 18:42:34.71ID:QiJM7dSD
>>965
東大の入試問題でやや難しいとされた問題です
誘導の(1)(2)を消して、この(3)部分だけにしたら、どういう解答を作ればいいか分かりませんでした
ありがとうございました
論述を頑張ってみます
0969132人目の素数さん
垢版 |
2020/09/09(水) 20:40:14.85ID:2WpbIfaQ
>>963
f_(n+1)(x) = f_1(f_n(x))
だから、 f_n(x) の性質は f_1(x) の性質によって決まる
x > 2 なら f_1(x) > 2 であり、 x < -2 なら f_1(x) < -2
また、 x が -2 → 2 と動くとき、 f_1(x) の値は -2 → 2 → -2 → 2 と動くから、
帰納的に
f_n(x) の値が単調に -2 → 2 となる x の範囲において、 f_(n+1)(x) の値は -2 → 2 → -2 → 2 と動く
ことがわかる
2 → -2 のときも同様
よって、 f_n(x) の値が単調に変動する x の範囲において、 f_(n+1)(x) は 3 回 0 になる
そのような範囲は f_n(x) に対して、帰納的に、 3^n 個あることがわかる
0971132人目の素数さん
垢版 |
2020/09/09(水) 21:34:23.30ID:ayHPVyw3
>>970
なるほどありがとう。 てか問の文で 3^n まで提示してるとか甘やかしすぎなんじゃ....
しかも解答例は面倒臭く考えすぎだと思う。そりゃ「やや難しい」になるわけだ。
出題側は >>964 をもっと丁寧に書けくらいの気持ちだと思う。
あんまり詳しくないけど東大数学の "難問" て、こんなもんじゃないでしょ。
0973132人目の素数さん
垢版 |
2020/09/09(水) 22:20:18.48ID:Ncax73dV
あるサイトが参加者にポイントを配るとします。
ポイントはランダムな量がランダムなタイミングで掲載され、取得は早いもの勝ちです。
参加者はページをリフレッシュして掲載されているポイントをクリックして獲得します。
サイト管理者はトラフィック量を増加させたくないので、参加者それぞれに
キャッシュを設け、60秒毎に情報を更新します。
キャッシュの更新が60秒毎なのは全参加者に共通ですが、バラバラのタイミングで
更新されます。(つまりある参加者のキャッシュでは掲載されているポイントが
別の参加者のキャッシュでは未掲載ということがある)
参加者は60秒間に2回だけリフレッシュすることが許可されています。
ポイントの配布は永久に続くものとします。
この時参加者にとってもっとも期待値が高くなる戦略を教えてください。
0974132人目の素数さん
垢版 |
2020/09/09(水) 22:23:25.60ID:Ncax73dV
追記
参加者はポイント掲載のタイミングも、自分のキャッシュが更新されるタイミングも分からないものとします。
0975132人目の素数さん
垢版 |
2020/09/10(木) 16:21:04.46ID:Gfqgi8U+
>>673
要はキャッシュクリアのタイミングをどれだけ効率よく推定するか?でいい?
最初のフェーズでは1/2分間隔でリフレッシュする。
例えばn,n+1/2分のタイミング
すると最初のページ更新のときに自分のキャッシュクリアのタイミングが[n,n+1/2]なのか[n+1/2,n+1]なのかがわかる
前者の場合
今度はnとn+1/4でキャッシュクリアする
するとn,n+1/4]なのか[n+1/4,n+1/2]なのかがわかる
一般にk回目のページ更新のときに[n+a,n+a+1/2^k]に絞られるから次のフェーズではn+a,n+a+1/2^(k+1)でリフレッシュする
細かいチューニングでさらに良くできるかもしれないけど大筋コレがベストな伊予柑
0976132人目の素数さん
垢版 |
2020/09/10(木) 22:53:17.09ID:wl4xf8iO
誰か教えてください。
積分の問題で、
0から∞まで積分で
e^(-st)tdt
なのですが、回答が
1/s^2

です。どなたか教えていただけませんか、、、。
0978132人目の素数さん
垢版 |
2020/09/11(金) 00:43:45.47ID:V//8CgLy
0 < m < n であるような
定数 m,n があるとする

関数の集合 A があるとして、n 個の任意の
異なる入力 x1,x2,...,xn について、
少なくとも m 個の等式 A(xk) = yk が真となるように、n 組の数字 y1,y2,...,yn を計算できるだろうか? 
0979132人目の素数さん
垢版 |
2020/09/11(金) 07:23:45.69ID:FXcn8PzE
xyz空間に半径1の円が2020個配置されており、どの2つの円もちょうど2点で交わっている。
これら2020個の円の位置関係を述べよ。
0980132人目の素数さん
垢版 |
2020/09/11(金) 07:58:16.82ID:+0o5IpbM
それだけで位置関係決まらんだろ
一つの円を少しだけずらしながらコピーするように配置すれば条件を満たすんだから、一点を固定してその充分近い近傍に残り2019個の点を取りさえすれば位置関係は自由じゃ?
それとも中心位置の距離の上限を調べろという糞問題かな
0982132人目の素数さん
垢版 |
2020/09/11(金) 08:29:21.92ID:hs1QHGjt
>>980で終わってるな
問いでは3次元空間に2次元図形の円を配置していることに注意して
すべての円は同一平面または
(半径が円と同じかより大きい)同一球面上にある
とすればよい
0983132人目の素数さん
垢版 |
2020/09/11(金) 08:42:58.37ID:hs1QHGjt
円の中心の分布の条件は
任意の2点の距離<元の円の直径
なので、分布は1点を中心とする円とは限らず
ルーローの三角形のような定幅図形でもよい

等号は含まないので、最大値はなし
0985132人目の素数さん
垢版 |
2020/09/11(金) 09:07:54.34ID:hs1QHGjt
>>973
キャッシュは普通、ユーザが使うブラウザやアプリに持たせるもの
サーバが全ユーザ分のキャッシュを保持するのは非現実的だが…
問題の通りの条件で、かつ1分ごとのページの更新が判別できるなら
戦略はおおむね>>975でOK

アクセス1分に2回の条件が毎分0秒にリセットと決まっているなら
分割探索で前半が確定すれば、最後の結果と次回の1回目の結果も60秒未満で
比較可能となるので、次回は前半を3分割、後半を2分割とできる
分割数は2の累乗より大きくでき、1、2、5、13、…と
フィボナッチ数を1つ飛ばしにした値になる
分割のタイミングは、黄金比 φ=1.618… を用いて
2分割は φ:1、3分割は φ:φ:1 とすれば最適化できる
0986132人目の素数さん
垢版 |
2020/09/11(金) 09:18:53.15ID:hs1QHGjt
>>985の続き
>>974で更新時刻は判別不可とあるが、現実的には
「配布中」⇔「配布されていません」
の画面の違いが長期的にみると出現するので利用できる
この場合は、断続的に2分探索を行うので>>975がそのまま使える

これを無視して、画面の更新を戦略に使わないとすれば
1分2回のアクセスをランダムに割り振るのが最善
キャッシュがランダムではなく、最初のアクセスから60秒であれば
ユーザも60秒ごとにアクセスするのが最善

早いもの勝ちのポイントサイトというと
ガッチャモールのローソン無料商品券配布祭りとかがあったな
0987132人目の素数さん
垢版 |
2020/09/11(金) 09:20:41.28ID:hs1QHGjt
>>984
異なる2点とは書いてないので、確かにそれもありですねー
0988イナ ◆/7jUdUKiSM
垢版 |
2020/09/11(金) 12:14:17.73ID:AaAozqQu
>>890
>>979
蜘蛛は蜘蛛の巣に捕まったコガネムシを任意の2点を固定してクルクルクルクル高速で回してコガネムシを身動きとれないストレスにより弱らせる。
0989イナ ◆/7jUdUKiSM
垢版 |
2020/09/11(金) 12:16:58.95ID:AaAozqQu
>>988訂正。
>>979
蜘蛛は蜘蛛の巣に捕まったコガネムシを任意の2点を固定してクルクルクルクル高速で回して身動きとれないストレスによりあるいは毒を注入して弱らせる。
0990132人目の素数さん
垢版 |
2020/09/11(金) 12:35:07.25ID:xcUymbow
「グラフGが2-因子分解可能であるための必要十分条件は、あるn≧1に対して、Gが2*n-正則であることである。」とロバースらの本に書いてあるのですが、
Gが連結でないと成り立たないと思いますがいかがでしょうか?
0991132人目の素数さん
垢版 |
2020/09/11(金) 12:48:40.71ID:bm+WDsM1
>>2因子分解可能とは?
ググっても出てこないけど?
グラフ理論みたいなマイナーな話ふりたいならグクっても出てこないような単語は載せんとダメだよ
0992132人目の素数さん
垢版 |
2020/09/11(金) 14:53:03.63ID:E85RL8Qh
(1) 同一平面上にあり、
  どの2つの円の中心の距離dも 0<d<2,   >>980

(2) 同一球面上にあり、(半径R≧1)
  どの2つの円の中心を球の中心から見た角θも   >>982
   0 < R sin(θ/2) < 1,

(3) z軸上の2点 (0,0,c) (0,0,-c) を通るように回す。0<c≦1.
   {x cosφ + y sinφ ± √(1-cc)}^2 + zz = 1,  >>984
   y/x = tanφ,
0993132人目の素数さん
垢版 |
2020/09/11(金) 15:04:40.34ID:E85RL8Qh
>>979
いまのところ
(1) 同一平面上にあり、
  どの2つの円の中心間距離dも 0<d<2,   >>980

(2) 同一球面上にあり (半径R≧1)
  どの2つの円の中心を球の中心から見た角θも  >>982
   0 < R sin(θ/2) < 1,

(3) z軸上の2点 (0,0,c) (0,0,-c) を通り、0<c≦1.
   {x cosφ + y sinφ ± √(1-cc))}^2 + zz = 1,  >>984
   - x sinφ + y cosφ = 0, 0≦φ<2π
0995132人目の素数さん
垢版 |
2020/09/11(金) 19:54:26.90ID:QjMckGWj
面積1の閉領域Dの周上または内部の点P(x,y)に対して点Q(x+y,xy)を考えます。
Pが動くとき、Qの存在領域の面積はDの何倍から何倍の間にあるでしょうか。
よろしくお願いします。
0996132人目の素数さん
垢版 |
2020/09/11(金) 22:03:59.50ID:SzpHTH85
>>995
x+y=u
x-y=v
とおいて
x+y=u
xy=(u^2-v^2/4
uv平面上の面積2の図形が変換
w=(u^2-v)^2/4
によってuw平面上の領域として移される場合の面積の値域と考えればよい
∴ 0〜∞
0997132人目の素数さん
垢版 |
2020/09/12(土) 08:32:45.01ID:zrYwMlIY
>>996
x+yとxyが有限の値になるので、てっきり有限の定数a,bでa倍〜b倍と表せると思っていました。
xy平面の単位円をこのように変換して面積を求める入試問題から、一般化を考えました。
しかし例えば「この変換で面積k倍になる元の領域全体はどのような集合か」でも、要素のパターンが無数にあって決定しきれない感じてしょうか
ありがとうございました
0998132人目の素数さん
垢版 |
2020/09/12(土) 15:47:07.57ID:n7twx+Wx
k≧1 の例ですが
正方形 (面積1)
 (x,y) = (k,0) (k+1,0) (k,1) (k+1,1)
は四角形 (面積k)
 (x+y, xy) = (k,0) (k+1,0) (k+1,k) (k+2,k+1)
に移るので、上限は無いようです。
下限は有るかも?
0999132人目の素数さん
垢版 |
2020/09/12(土) 16:39:36.66ID:n7twx+Wx
k=1/6 の例ですが
a≧0 として
正方形 (面積1)
 (x,y) = (a,a) (a+1,a) (a,a+1) (a+1,a+1)
は放物線とその接線の隙間 (面積 1/6)
 a(u-a) ≦ v ≦ (u/2)^2,  (2a≦u≦2a+1)
 (a+1)(u-a-1) ≦ v ≦ (u/2)^2,  (2a+1≦u≦2a+2)
に移る。
 (u, v) = (x+y, xy) とした。
1000132人目の素数さん
垢版 |
2020/09/13(日) 12:45:55.03ID:aLRApFcX
k>0 の例ですが
b>a≧2k として
斜め長方形 (面積1)
 (a-2k, a+2k) (a+2k, a-2k) (b-2k, b+2k) (b+2k, b-2k)
 ただし k = 1/{8(b-a)},
は2本の放物線の間 (面積k)
 v = (u/2)^2,
 v = (u/2)^2 - (2k)^2,
 2a≦u≦2b,
に移る。
b-a → ∞ のとき k → 0
∴下限も無い。

>>996 が正解。
10011001
垢版 |
Over 1000Thread
このスレッドは1000を超えました。
新しいスレッドを立ててください。
life time: 40日 13時間 20分 49秒
10021002
垢版 |
Over 1000Thread
5ちゃんねるの運営はプレミアム会員の皆さまに支えられています。
運営にご協力お願いいたします。


───────────────────
《プレミアム会員の主な特典》
★ 5ちゃんねる専用ブラウザからの広告除去
★ 5ちゃんねるの過去ログを取得
★ 書き込み規制の緩和
───────────────────

会員登録には個人情報は一切必要ありません。
月300円から匿名でご購入いただけます。

▼ プレミアム会員登録はこちら ▼
https://premium.5ch.net/

▼ 浪人ログインはこちら ▼
https://login.5ch.net/login.php
レス数が1000を超えています。これ以上書き込みはできません。

ニューススポーツなんでも実況